Sei sulla pagina 1di 217
THE MATHEMATIE€S TEAGHER SUB-JUNIOR-II 2 am Unrae Sele q oe 0 4 MATHEMATICS TEACHERSIOF us GEMS From THE MATHEMATICS TEACHER SUB-JUNIOR -II Edited by Sri. G. Gnanasundaram THE ASSOCATION OF MATHEMATICS TEACHERS OF INDIA B-19, Vijay Avenue, 85/37, Venkatarangam Street, Triplicane, Chennai t 600 005. Tel: (044)-28441523 E-mail : amti@vsnl.com Web site: amtionline.com reneneonncernonwonne ly BETWEEN US Dear Reader, The demand for our books based on NMTC questions had been increasing steadily. Hence it is decided to bring all the problems and solutions of NMTC every five or ten years without disturbing the earlier editions. To satisfy the needs of various age groups they are being brought out in five different books ~ Primary, Sub-Junior, Junior, Inter and RMO and INMO. The Present period covered by these books are from 2004 to 2012. In the case of RMO and INMO it is 2005 to 2013. We hope and trust the beneficiaries of our service will continue to encourage us to help aspirants of excellence in Mathematics Education. We are grateful to Smt. K. — Maheswari, Sri G. Gnanasundaram, Dr. M. Palanivasan and Sri V. Seshan respectively for having gone through the pages and edited these books in record time. With best wishes. Yours sincerely, At (M. MAHADEVAN) General Secretary, AMTI 1. Year 2004 2. Year 2005 3. Year 2006 4. Year 2007 5. Year 2008 6. Year 2009 7. Year 2010 8. Year 2011 9. Year 2012 CONTENTS Screening Test Final Test Screening Test .. Final Test . Screening Test...... Final Test...... Screening Test .. Final Test. Screening Test .. Final Test. Screening Test Final Test Screening Test . Final Test . Screening Test .... Final Test... Screening Test Final Test KAPREKAR CONTEST Sub-juuion Loved Screening Test-2004 1, A four digit number of the form abaa (a's and b’s are the digits of the four digit number) is divisible by 33. The number of such four digit numbers is (A) 36 (B)6 (0) 3 (D)1 Solution: As abaa is divisible by 33, it is divisible by 11 as well as 3. Hence (i) (a +a) ~ (6+) or a—d is divisible by 11 (ii) 3a + b is divisible by 3. As a and b are single digited integers (i) implies a — b = 0 or a= and therefore (ii) gives da is a multiple of 3. Hence there are three numbers 33336666 and 9999 satisfying the conditions of the problem. Answer: (C) Nn . a+ 2a+ 3a +--+ 10000 = 26+ 4b + 6b +... + 20005 = 3c+ 6c-+9c++++3000c then a:b: cis as (A) 1:2:3 (B) 3:2:1 (Oy 2:3:6 (D) 6:3:2 Solution: Cancelling off the common factor 1+2+3-+- +1000 from the three equal sums given, we get a= 2b = Se or & a:b:c=6:3:2, Answer: (D) 6 Problems and Solutions 3. ax =atland sa =a-—1, then 1t—#1+2*—#2+34—« 3-4... 1000 * — * 1000%is equal to (A) Y000 (B) —1000 (C) 2000 (D) -2000 Solution: Asax = a-+1, x2 =a—1 we get ax—+a-= 2. This the given sum is 2+ 2-+--++ 1000 terms= 2000. Answer: (C) 4. ‘a’ and ‘BD’ are two natural numbers with a+6 = 8. Ifa>b6 and-a? + 8? has. minimum value, then a and 6 are given-by (A) 71 (B) 6,2 (C) 44 (Dy 5,2 Solution: ; a? +b? = a?+(8—a)? = 2a? — 160+ 64 2(a? — Ba + 16) +32 = 2(a~ 4)? + 32 i ‘This is minimum when a = 4, Then b +4. Answer: (C) 5. In the adjoining figure, AABC is, right angled at A; mZB = mZ0.~ The bisectors of angles B and C meet at I, Then mZBIC is (A).135° (B) 115° (C) 100°.(D) 90° a Solution: ABC is a right angled isosceles triangle. mZABC = mZACB = (180° = 90°) = 45° Since BI,CI are internal bisectors of ZABC and 4ACB, mZIBC = mZICB = 223°. mZBIC = 180° — 2 x 224° = 135° Answer: (A) . The number of isosceles triangles in which one angle is 4 times another angle is 3 (A) 2 (B)1 (C) Infinitely many (D)4 ae Problems and Solutions Solution: If tho base angles are four times the vertical angle, then exch of the base angles is 80° and the vertical angle is 20°. On the other hand, if the vertical angle is four times either of the base angles, then the base angles are 30°, 30° and the vertical angle is 120°, These are the only two possibilities. Answer: (A) Nine dots are arranged such’ that they are =) yy equally spaced horizontally and vertically asin |) the figure: The number of triangles which are |), not right angled triangles that can be formed with the above dots as vertices is (A) 18 (B) 21 (c) 40 (D) 24 Solution: Dots belong to the same row or the same column or the same diagonal do not form a triangle. Name the dots as A, B,C, D, E, F,G,H,J as shown ‘Yaking A, B of the first row and joining them to the six points of the second and third rows, we can get six triangles. Of these AABD, AABE, AAGB and AABH are right angled. Hence only AABF and AABI satisfy the given condition. Similarly with B,C we have ABDC and «Q Ce o> et tte et en me eQ * ABGC. With C,A we have only AAHC, since AAEC is right angled. (Note that if AB = BC = a, then AC? = 4a? and AB? = CE? = 2a? so that AC? = AE? + CE*). Thus every row, except the second, contributes 5 triangles. Similarly every: column except the second will contribute 5 triangles. The second row and the second column will give only 4. Thus on the whole we get 28 triangles. Besides thses there are four more SAFH,ADCH, AIBD and AGBF. (These triangles have a Problems and Solutions vertex from each row and also form each column) Thus the total number of non-right-angled triangles that can be formed is 32. Thus none of the answers given is correct. Answer: 32 . The number of non negative integers which are less than 1000 and end with only one zero is (A) 90 (B) 99 (C) 91 (D) 100 Solution: Required type of numbers will be nine in each of the ten subsets {1,2,---99}, {100, 101, ---199}--- {900, 9012, '--999}. This accounts for 90. Including 0 we have, therefore, 91 such numbers. Answer: (C) . The terith term of the sequence- (2,5), (3,7), (5,11), (7,18), (11,17), «++ is * (A) (23,29) (B) (19,18) (C) (20,04) (D) (29,31) Solution: The sequence is formed by pairs of prime numbers PirDea2 where py is the kth prime. (k = 1,2---). Hence the tenth pair is (pio, 712). From the sequence of primes 2, 3, 5, 7, 11, 13, 17, 19, 23, 29, 31, 37, --». We find that pip = 29 and Piz = 37. Thus the answer is (29,37). None of the options given is correct. Aliter: In the given sequence both members are primes and the first member is smaller than the second. Hence options (B) and (C) are-ruled out. Again the two members of the pait are separated by middle prime, (A) and (D) give successive primes. Hence these two are also ruled out. No option gives the answer. Note: Perhaps option (D) is a misprint and should read (29,37)- Problems and Solutions 9 10. The sum of the digits of the number 10" ~ 1 is 3798. The value of n is (A) 431 (B) 673 (C) 422 (D) 501 Solution: 10" — 1 is an n digited number with all digits equal to 9. Hence sum of its digits is 9n. But it is given as 3798. n= 422. Answer: (C) 11. The image of INMO when reflected in a mirror is (A) Iawo (8) OMNI (c) INWO (D) OWNI Solution: If INMO is written on a transparent paper and held in front of a mirror the reflection reads as OM I. This ts not any one of the options given. On the other hand, if the paper with INMO written is held in front, its reflection in a horizontal mirror below the paper will be ‘INWO” Answer: (A) 12. A transport company’s vans each carry a maximum load of 12 tonnes. 24 sealed boxes each weighing 5 tonnes have to be transported to a factory. The number of van loads needed to do this is (A) 9 (B) 10 (ou (D) 12 Solution: Two sealed boxes weigh 10 tonnes and three sealed boxes weigh 16 tonnes. Thus the van can carry a maximum of two sealed boxes. Hence nurtber of van loads needed to transport 24 sealed boxes is 3 = 12. Answer: (D) 13. The digits of the year 2000 add up to 2. In how many years has this happened since the year 1 till this year 2004? (A) 3 (B) 6 (C)9 (D) 10 10 Problems and Solutions Solution: Number of single digited numbers with sum two = 1 Number of two digited numbers with sum two = 2 Number of three digited numbers with sum two = 3 Number of four digited numbers with sum two = 4 Hence total such numbers with sum two =1+24+3+4=10 ‘These ten numbers are 2, 11, 20, 101, 110, 200, 1001, 1010, 1100 and 2000. Answer: (D) 14, Ram is 7 years younger than Ravi. In four years time, Ram will be half of Ravi's age. The sum of their ages now is (A) 13 (B) 15 (©) 17 (D) 19 Solution: If the present age of Ram is x years then that of Ravi is 2 +7 years, After 4 years their ages will be 2 + 4 and 411. As Ram’s age after 4 years is half of Ravi’s age then, we have 2(¢ +4). = +11. Hence, = 8. So the sum of their present ages is 3+ (3 +7) = 13. Answer: (A) Aliter:' The situation is diagrammatically represented below: Ram Ravi o 4 7 Years B a Ravi a A 4 Years A 7 Years B Four years hence Ram's age is represented by OA’ and Raii’s age is represented by OB'. Sine OA'=half of OB’, OA’ = A‘'B!=Tyears, “ OA=OA'.- AA'=7~—4=3 years and OB =3+7=10 years. Hence sum of their present ages = 3+ 10 = 13, Problems and Solutions 11 15. A circle is added to the equally ie eye spaced grid alongside, The largest rer erie ie. number of dots that the circle can othe ite pass through is oe eee ee ree (A)4 (B)6 (C18 (D)10 Solution: Denoting the dots in the first, second, third, fourth and fifth rows respectively by Aj, Aa, As, Aa, Asi Bi, Ba, Bs, Ba, Bs; Ct, C2, C3, C4, Cs; D1, Da, D3, Da, Ds; and ©), En, Ex, E4, Bs, the circle with centre C3 and radius C3A2 Passes through the eight points Az, As, B1, Bs, D1, Ds, Eo and E,. This is the maximum number of points. None of the options given is 8. Note: Perhaps option (C) viz., 18, is a misprint for 8. With this correction the answer will be (C). 16. A certain number has exactly eight factors including 1 and itself. Two of its factors are 21 and 35. The number is (A) 105 (B) 210 (C) 420 (D) 525 Solution: Since the number has exactly eight factors, it has the representation, pi,ptp2 or pipaps where p’s are primes. (For example, if it is p}p2 then the factors are 1, p1, PIPE Pas Pipa, PyP2 and pip2.) As 21 and 35 are its factors 3,5 and 7 are its factors. Hence the number is 3 x 5 x 7 = 105. Answer: (A) Note: The eight factors are 1,3,5,7,15,21,35 and 105. 17. Ima magic aquare, each row and each column and both main diagonals have thesame total, |_18 ‘The number that should replace x in this 5 15 partially completed magic square is = (A) more information needed (B) 9 (C) 10 (D) r2 12 Problems and Solutions Solution: From the first column, the total of the magic square is 18+ a. Therefore the number in the central cell is 18+ a— (15 +45) = x—2. This implies that the last number in the first row is 18 + s — («+s ~—2) = 20—z, To find the number in the third cell of the third row we can use the third column or the diagonal through it. Hence this number is equal to 18+ ¢ ~ (13 +2 —2) or (18 + 2) — (20-2 +18). Thus 7=Iw-1T of 2=12. Answer: (D) Note: The completed magic square will be as. [13 8 in the adjoining table. 5 }10| 18 wfit}7 18. In the triangle ABC, D is a point on the line segment BC such that AD = BD = CD. The measure of angle BAC is (A) 60° (B) 75° (C) 190° (D) 120° Solution: Since DA = DB = DC, D is the circumcentre of AABC and BC is the diameter of the circumcircle. So, mZBAC =.90°. None of the options is 90°. Note: (C) should have been 90° and not 190°. No angle of a triangle can be greater than or equal to 180°. 19. The product of Hari’s age in years on his last birthday and his age now in complete months is 1800. Hari’s age on his last birth day was (A) 9 (B) 10 (0) 12 (D) 15 Solution: If Hari is x years old on his last birthday and y < 12 be the number of months elapsed after that birthday, we have 2(12z + y) = 1800. 122? < 1800 or 2? < 150. Problems and Solutions 13 The largest integral value of x satisfying this inequality is 12. ‘Thus Hari’s. age on his last birthday =12 years. Answer: (C) 20. One hundred and twenty students take-an examination which is marked out of 100. (with no fractional marks). No three students are awarded the same mark. What is the smallest possible number of pairs of students who are awarded the same mark? (A)9 (B) 10 (0) 19 (D) 20 Solution: If every mark from 0 to 100 is given one each to 101 students, the retraining 19 students will have to, be awarded marks already given. As no three students score the same mark, the minimum possible pairs of students getting equal marks is 19. Answer: (D) 21, If all the diagonals of a regular hexagon are drawn, the number of points of intersection, not counting the corners of, the hexagon is (Aye. (By 13 (C)7 (D) 12 Solution: Let ABCDEF be the regular hexagon. ‘Then there are nine diago- nals - 3 diagonals from each of three alternate vertices: AG, AD, AB, CE,CF,CA, EA,EB and EC. From the figure it is easily seen that the number of points of intersection is 13. Answer: (B) 14, Problems and Solutions Note: The diagonals AD, BE, CF are concurrent at the centre O of the regular hexagon. 22, Three people each think of a number, which is the product of two different primes. The product of the three numbers which are thought of is (A) 120 (B) 12100 (CG), 240 (D) 3000 Solution: The product of the. three numbers is clearly a composite number which can be expressed as the: product of 6 primes of which at least two must be distinct. Using prime factorisation , 120= 23 x 3x 5, 12100 = 2% x 5? x 117; 240 = 24x 8x5, 3000 = 23 x 3 x 5% We find that only 12100 is expressable.as the product of. six primes while the others cannot be expressed as product of six primes. The numbers thought by the three persons are 2%5=10, 6x11=65 and 11x2=22, 23, The area of the shaded region‘in the diagram is (A)9— (B) 8v2 (C)18 — (D) 6V3-3v2 Solution: The shaded area. is the area of the hexagon ABCDEF sinus the area of the square PQRS whose diagonals are both equal to 3 units. Thus the required erea is equal Problems and Solutions 15 to 3(6+3)-$+3(6+3)-3— 88 = 4 _ 2 = 9 square units. Answer: (A) Nots: The hexagon ABCDEFis not a regular hexagon and is symmetric about PR and FSQC. 24, The largest positive integer which cannot be written in the form 5m + 7n where m and n are positive integers is (A) 25 (B) 35 (C) greater than 100 (D) greater than 350 Solution: The number 26 can be written as 5x1+7%3. Hence (A) is not the answer. 33 can also be written as 5x 1+7x 4. Also 34 can be written as 5 x 4+ 7x2, Hence the number required is greater than 34. Every number greater than 35 can always be written as 5m+7n for suitable positive integral values of m and n. For example take the number 103. A multiple of 7 ending in 3 being 63, we can write 103 as 7x 9+45 x 8. Similarly, 245 = 210 + 35 = 7 x 8045 7 or 5x 42+7% 5. For no integers m,n > 0. 35 can be written as 5m -+7n. For m = 1,2,3,4. We find 5m is 5,10,15,20,15 none of which is a multiple of 7. For m = 2, n has to be 0 and not a positive integer. Thus 36 is the largest positive integer of the required form. Answer: (B) 25. The last digit in the finite decimal representation of the 12004 number ($)7™ is (Ay2° (B) 4 (06 (D) 8 Solution: We have 1 \ 2004 1 (3) 0-2 am Now 24 = 16 and 270% = 16 x 16 x ---501 factors. It is easily seen that two numbers ending in 6 when multiplied give another number ending in 6. Thus the last digit in 2° js 6. Hence in 27004 /102004 the last digit of the decimal representation will also be 6. Answer: (C) KAPREKAR CONTEST Sub-junion Level Final Test-2004 1. The greatest common divisor of a and 72 is (a, 72) = 24 and the least common multiple of b and 24 is (, 24] = 72. Find the g.c.d (a,5) and the l.c.m (a, b] given that ais the smallest three digit number having this property; and b is the biggest integer having this property. Solution: The smallest possible value for a is the smallest 3-digit number which is a multiple of 24; hence the smallest possible value for a is 120. The biggest integer 6 with the property (b, 24] = 72 is clearly 72. So for the desired numbers a,b we have g.c.d(a,b) = g.c.d(120, 72) = 24 and L.e.mfa, 5] = L.c.m[120, 72} = 360, 2. Given that’ a? ~ 6? = 105 and a and b are two relatively prime Positive integers ( two positive integers m and n are relatively prime if their g.c.d (m,n) = 1), find all such a and b. After having found all such a and 6, if one draws a triangle ABC with sides having lengths a? — 6”, a? +? and 2ab find the area of all such triangles. Solution: It is given that a? ~ 8 = (a+b)(a—b)=1x3x5x7=105, So, the various possibilities for the ordered pair(a—b,a+6) are (a-b,a +b) = (1,105) (a—b,a +6) = (3,35) (a-ba+b) = (5,21) (a-b,a+) = (7,15) Solving we get (a,b) = (63,62), (a,b) = (19,16), (a,b) = (13,8), (4,5) = (12,4) Problems and Solutions 17 S The triangle with sides a? +, a? — 6, 20b is always a right angled triangle as (a? — 87)? + (2ab)? = (a? + 6)?. The area of such a tiiengle is ab(a? — b*) = 105a. Thus the required areas are 289380, 31920, 10920, 4620. . Ais a set of 2004 positive integers. Show. that there is-a pair of elements in A whose difference is divisible by 2003. Solution: When we divide a number by 2003 there are 2003 possible remainders, namely, 0, 1, 2,3,..., 2001, 2002. Now, since A contains 2004 positive integers, there are at least two. numbers in A which leave the same remainder when divided by 2003. Call them.a; and aj. Then the difference |; — aa| is divisible by 2003. Let ABC be an acute angled triangle with AD, BE, CF as the altitudes (i¢., D is the foot of the perpendicular from A on BC and so on...). If the altitudes meet-at the point O, find the measure of the angles £BOC,ZCOA,ZAOB in terms of the angles ZA, ZB, ZC of the triangle ABC. Solution: From the right triangles BEC and CFB it is clear that ZCBE = 90° — ZC and ZBCF = 90° ~ ZB. Fig. 18 Problems and Solutions f Hence in ABOC we have ZBOC = 180° — (ZCBO + ZBCO) = 180° ~ (90° — 20 + 90° - 2B) = 4B + CO Similarly we find ZCOA and ZAOB. Thus we have ZBOC = ZB + 20; Z0OA=£C+LA; ZAOB=2A+ 2B . Is the statement “If p and p?-+2 are primes then p* +2 is also @ prime” true or false? Give reasons for your answer. Solution: Suppose p and p? +2 are primes. One such prime number is p = 3. If p # 3, then p = 3k+1 end pP+2 = (8k 1)? +2 = 9k? £6+3 which is always @ multiple of 3 and it is always greater than 3 for all integral values of k. Hence for p # 3 we see that p? + 2 is never a prime number. So, p and p? +2 are primes if and only if p = 3. In this case p? +2 = 29 is also a prime. Thus the statement “If p and p* +2 are primes then p? +2 is also a prime” is always true! . Let P denote the product of first n prime numbers(with 2 > 2). For what values of n we have 1. P—1is a perfect square 2. P+1is a perfect square Solution: Since n > 2, we have P as a multiple of 3. So P—1 is of the form 3k-+2 for some positive integer k. But any perfect square is either a multiple of 3 or of the form 8m +1. Thus P — 1 is never 8 perfect square for n > 2. Now, P +1 is an odd integer and hence if P+ 1 = b? then b must be an odd number. Let b= 2m+1. Then P= b?—1= 4m?+4m, This means that 4 divides P=2x3x5x7x 1L(n factors) which is impossible. ‘The answer to the question is that thore is no such n > 2. Probleme and Solutione 19 7. Three counters A,B and C are coloured with three different colours red, blue ond white, Of the following statements only one is true. 1, A is red. 2. Bis not red. 3. C is not blue. What is the colour of each counter? Solution: We use the notation (FR, B, W) to denote that A is coloured red, B is coloured blue and C’is coloured white, With this notation (B, R, W) will mean that A is coloured blue, B is coloured red and C is coloured white. (©) (R,B,W)_| True ‘True (R,W.B) | ‘True | True | False [ (B,R,W) | False | False | True (B,W,R) | False | True | True (W,R,B) | False | False | False (W.B,R) rie True | True From the table it is clear that A is coloured BLUE, B is coloured RED and C is coloured WHITE. 8. The sum and least common multiple of two positive integers z,y.are given as z-+y = 40 and l.c.m [ z,y] = 48. Find the numbers z and y. Solution: Let the g.c.d(z,y) be d. Then x = da and y = db for some integers a,b. We have «+y = d(a+b) and we observe that a and b have no common factors other than 1. Now, Le.mfg, y] = l.c.m{da, db] = dab = 48. 20 Problems and Solutions Again, g.c.d (d(a +b), dab).= d x g.c.d((a+6),ab) = 4 since g.c.d((a + 6),ab) = 1. ‘This means that g.c.d (40,48) = 8 =d. Therefore, we get at+b=5, ab=6 which on solving give a = 3, b= 2ora=2,b=3. On substituting these values for a,b we get n= 24, y= 16 or t= 16, y= 2A. 9. What is the greatest positive integer n which makes n3 + 100 divisible by n + 10? Solution: We have n8 +100 = n3 + 10 — 900 = (n + 10)(n? — 10n + 100) — 900. Therefore n + 10 divides n° + 100 if and only if n+ 10 divides 900. The largest n such that n +10 divides 900 is 890. So, the answer is 890. 10, Find the sum of all three digit numbers that can be written using the digits 1,2; 3,4 (repetitions allowed). Solution: We may have any one of the four digits 1, 2,3, 4 in the hundreds place, tens place or units place. Therefore we have 4.x 4 x 4 = 64 three digit numbers in all. If we consider the units place of these 64 numbers, we see that each of the four digit appears 16 times. The same conclusion is true about tens place and hundreds place. So the sum of the 64 three digit numbers that we can form from the given four digits is Ss 16(1+24+3+4) x 100+16(1+243-+44) x10-+16(1+2+3+4) x1 +S = 16000 + 1600 + 160 = 17760 Problems and Solutions 21 11. Consider the collection C of all isosceles triangles of area 48 sq.units, whose bases and heights are integers. How many triangles are there in.C? How many triangles in C have their equal sides also of integral lengths? Solution: If we use the formula Area of a triangle == base x height we get base x height = b x h = 96. So the various possible values for (b,h) are (1,96), (2,48), (3,32), (4,24), (6, 16), (8,12) (12, 8),. (16,6), (24,4), (32,3), (48,2), (96,1). For each of the above 12 choices for (b, ht) we have an isosceles triangle satisfying our requirements. So, there are 12 such triangles. Of these only two triangles have sides of integral lengths. They are got when b = 12, h=8 and b = 16, h=6 as 6? + 8? = 10? and 8? + 6? = 107. Thus the triangles with integral sides have sides of lengths 12, 10,10 or 16,10, 10. 12. In triangle ABC, we are given that ZA = 90°. Median AM, angle bisector AK and the altitude AH ere drawn. Prove that MAK = ZKAH. Solution: We have AM = C MB = MC since ZA = K 90°. If ZMAK = 2°, then ZBAM = 45° — 2°. From the isosceles triangle AMB we get Al B ZABM = ZABC = 45° — 2°, Therefore ZACB = 90°— ZABC = 90° — (45°—x°) = 45°+2°, From the right angled triangle AHC we get ZCAH = 45° — 2° and hence 2K AH = ZCAK - 2CAH = 45° — (45° —a°) = 2°, Thus 2MAK = 2K AH. 22 Problems and Solutions A 13. Triangle ABC is divided into four regions with areas as shown in the diagram. Find z. Solution: Let us call the point of intersection of BE and CF inside the triangle as D. Set area of AAFD =a and area of AADE = b, Then we have ‘Therefore a@_a+b+6 a _ 6+6 97 9a 797 ao Similarly, b _AAED _AE _AAEB _a+b+9 6” AGED ~ EC ~ ACER 6411 ~ ‘Therefore 67 Sen 6 it (2) From equations (1) and (2) w get 1a -96~54=0 (3) 6a—11b+54=0 (4) Problems and Solutions 23 Solving the system (3),(4) we get = 1080, 918 ~ OT?” 67" Hen ° map 5= 188 r= = 7 14. ABCD is a cyclic quadrilateral (which means that a circle Passes through the vertices A,B,C, D).In- other words the vertices A,B,C,D, in that order, lie on a circle. If the diagonals AC and BD cut at right angles at B, prove that AE? + BE? +.CE? + DE? = 4R? where R is the radius of the circle ABCD. Solution: Let O bé the centre of the circumcircle of the quadrilateral ABCD. It can be easily observed that ZAOD + ZBOC = 2ZABE + 2ZEAB = 2(ZABE + ZEAB) = 180°. This gives ZA'OC = 180° — ZBOC = ZAOD - where BO meets the circle again in A’. Therefore AA'OC = AAOD (SAS property). “ AD=A'C 24 Problems and Solutions AE? + BE? + CE? + DE” = AD? + BC? = AC? + BO? = A'B?(:: ZA'CB = 90°) = 4R?. 15. A= {a,b,¢,d,e} is a set of five integers. We take two out of the numbers in A and add. The following ten sums are obtained 0, 6,11, 12, 17, 20, 23, 26, 32, 37, Find the five integers in the set A. Solution: We may assume that a » | 20029 | 11 times = » 30 to 99 7 times ” » | 100 to-119 | 2 times » » | 120 to 129'| 11 times ” » | 130 to 199 | 7 times * » | 200 to 209 | 11 times sil = 210 to 219 | 11 times » » | 220 to 229 | 21 times ” » | 230 to 249 | 22 times andon page | . 250 1 time ‘Total number of times = 106. Answer: (B) Note: Tuke care of 22, 122, 202, 212, 222, 232, 242. 26 Problems and Solutions 4, In the given addition sum, H,2,O are different = WE digits. The value of the letter O is HE (Ai (B)8 (c) 9 (D) 4 HE HE OH Solution: As 4(HE) = OH,OH can at best be 96, a two digited number which is a multiple of 4. Thus HE < 24. ‘There are three possible values for H viz 0,1 or 2, But H cannot be 1 as OH has to be a multiple of 4 and therefore even. If H =0, then E can only be 5 and OH is 20. ie., O is 2. This option is not givon. So H can only be 2; in this case, OH will be greater than 80 only 92 is the number between 80 and 96 divisible by 4 with 2 in the units place. Hence O=9. E is obviously 3. Answer: (C) A cube of edge 4cms is painted externally. It is then cut into one em cubes. How many of these do not have red point on any face? (A) 4 (B) 8 (C) 66 (D) 16 Solution: Only the central 2 x 2x 2 cube will have no face painted red as it is not exposed to the given 4x 4x 4 cube. Hence only 8 one-cm edged cubes will have none of their faces painted. Answer: (B) Problems and Solutions 27 6. Look at the additions on the left. Here A,B,C are different digits. If AB AB C is placed in the units’ column the Cc Cc total is 45. If C is placed in the 4 65 9 tens’ column the sumo is 99. Then the value of A is (A)4. (B)3 (C)9 (D)3 Solution: Two of the options (B) and (D) are the same one of them must be changed to a number different from 3. Let (D) be N #3,4 or 9. When © is in the tens place, B= 9 obviously. As B+C gives a two digited number with 5 in the units place, B is 6. As A with the carry lof B+C = 16 is 4,4 is 3. Answer: (B) 7. You have five pieces of 6cm rods and 4 pieces of 7cm rods. Using some or all of them, which one of the following lengths you cannot measure? (A) 30 {B) 29 (C) 31 (D) 33 Solution: As 30 = 5x6, 31 = 4x6+1x7 and 33 = 2x6+3x7, these lengths can be measured with the given 5+4=9 rods using a judicious choice of § rods among them. But 29 4 7m-+6n for any positive integral values of m and n. (Because 29 minus any multiple of 7 less then 29 is not divisible by 6 and 29 minus any multiple of 6 less than 29 is not divisible by 7). Hence 29 is the answer. Answer: (B) 8. Some donkeys and some chickens totally 20 in all were seen in @ garden. I counted 64 legs in all. How many donkcys were there? (A) 12 (B) 10 (Cc) 14 (D) 17 26 Problems and Solutions Solution: Let there be d donkeys and’c chickens. Then by the problem d+c= 20 and 4d+-2c=64 or 2d+¢= 32. Hence d= 12 and ¢c=8. Number of donkeys is 12. Answer: (A): 9. Five books and two pencils cost Rs.79. But two books, and five pencils cost Rs.40. What is the total cost of one book and one pencil? (A) Rs.39 (B) Rs.19.50 © (C) Rs.17 ~—(D) Rs.23.80 Solution: Let the cost of a book be Rs.b and of a pencil be Rs. p. Then 55 +2p =79 and 2b-+ 5p = 40. Solving we get b= 15 and p= 2. <. Cost of one book and one pencil is Rs.17. Answer: (C) 10. How many two digit numbers divide 109 with a remainder of ae (A)2 (B)4 (G) 3 (D) None Solution: As remainder is 4, we need two digit divisors of 109-4= 105. As 105 = 5x37. Two digit divisors are 15,21 and. 35. Thus there are 3 such divisors. Answer: (C) 11. The highest oe of 2 that divides the sum of the numbers 4444+ 4date..+ dad 160 fours (a)2 (8) 3 (c) 4 (D) 5 Solution: 4+ 44+44444.---4444..... 0 i 300 foura = 5(9+99+49994...... 99... *23) a §{00- 1) + (10? ~ 1) +--+ a0! ay} Problems and Solutions 29 = 4 ((a6-+ 10? +--- +10") — 100} = ; (a number ending with 0) * : (a number ending with 5) = 8 (a number ending with 5) =2°x an odd number. Hence the highest multiple of 2 in the sum is 3. Answer: (B) 12. @ and 6 are’square numbers; th l.c.m of a and 140 is 560 and the l.c.m of 6 and 140 is 700. Then the Lem of a and 6 is (A) 400 (B) 1600 (@) 2500 (D) 4900 Solution: Clearly a <6. The Lc.m. of a,b and 140 is the Lem. of 560 and 700 viz., 2800. But 2800 = 4?x5?x7, 560 = x57 ond 700=5?x 4x7, Hence a = 4? and b= 8? and Jem. of a and 6 is ab= 400. Answer: (A) 13, a,b,¢,d are natural numbers such that a=bé,b=cd,c=da and d= ab. Then (a+)(o+c)(c+d)(d+e) is equal to (A) @ta+c+d) - (B) (a+b) +{c+d? (©) (a+ 4)? +640)? (D) (a +0)? + (64d)? Solution: Multiplying all the four given relations we get abed = a 67c%d? 80 that. abed = 1. Also atbtet+d = be+ced+da+ab = (a+c)(b+d) Again, a+b = ebt+d),b+ce=d(c+a), e+d = a(b+d) andd+a=b(a+c) 30 Probleme and Solutions abed(b + d)?(a + 6)? abod(a + 6 -+0+¢)? (atb+e+d)?, Answer: (A) & 6 (a+ byb-+ (et d)(d +a) W ‘since obed = 14, If a?+a+1=0 then a? + is a (A) positive integer (B) positive fraction which is not an integer (C) negative integer (D) negative fraction which is not an integer Solution: As a #0 from a?+a+1=0, we have tie and (0+ 2)? =1. 44a. o &+te-1<0. Answer: (C) 7 2 15. ABCD is a square and an equilateral triangle CDE is drawn inward. Then (A) E lies on AB and ZAEB isa straight angle (B) £ lies is the interior of the square and ZAEB = 150° (C) EB lies is the interior of the square and ZABB = 120° (D) E lies outside the square and ZAEB = 150° Solution: 2 cannot be outside the square, because, then CE > BC. & also cannot be on the side AB, because, then in the right angled triangle CBE the hypotenuse CE will be equal to the side BC. Hence (A) and (D) are eliminated. Hence the figure D, fF is as shown. AGEB’ and ADEA are isosceles with ZBOER = ADE = 90° ~ 60° = 30°, «. ZAED = ZBEC = A B 3 (180° ~ 30°) = 75°, Problems and Solutions 31 + ZAEB = 360°~ ZAED~ZDEC- 2BEC = 360° — 75° — 60%"* — 75° = 150° Answer: (B) 16. Which of the following can never be a common factor of 287+ and 378+ where 2 can be any natural number? (A) 26 (B) 13 (0) 91 (D)7 Solution: = can be either odd or even. In any case one of the numbers 287-+2 or 378+ will be odd and hence cannot be divided by 2. As 26 ig an even number it cannot divide both the mimbers 287+ and 378+, for any natural number z. ‘Answer:-(A) 17, How many 4 digit numbers with middle digits 97 are divisible by 45? (A) 0 (B)2 (cy 4 (D)1 Solution: The given four digit number 207y,, say, is divisible by 45 i.e,, it is divisible by 5 and 9. ‘To be divisible by 5, the unit digit y should be either 5 or 0. Case (i) Let y= 5. The four digit number is 2975 aind the sum of digits is 9+7+5+a = 21+2. Ifthe number is divisible by- 9,21+2 is a multiple of 9. So 2 =6. Thus 6875 is divisible by, 45. Case (ii) Let y= 0. Then as in case (i), (r+ 9+7+0) is divisible by 9 exactly. So = 2 and the number is 2970. ‘Thus there are two numbers, 6975 and 2970 divisible by 45. Answer: (B) 18. The product of threo consecutive odd numbers is 357 627. What is their sum? (A) 213 (B) 243 (©) 188 (D) 209 Solution: The sum of any three consecutive odd numbers will always be divisible by 3. (For, if the numbers are taken as In~1,2n+1,2n+8 then their eum is 6n+3 or 3(2n+1)), 32 Problems and Solutions As 209 is not a multiple of 3, (D) is eliminated. Noting that 50°,70° and 80? are respectively 125000, 343000 and 512000 and that the product of the three consentive odd numbers is 357627 the numbers can be guessed to be near 70. The middle numbers in (A),(B),(C) are 71,81 and 51 respectively, Hence the required odd numbers are 71—-2,71 and 71+42. ie, 69,71 and 73 with sum equal to 213. Answer: (A) 19. The number of integers whose square is a factor of 2000 is (A) 3 (B) 6 (C) 10 (D) 12 Solution: As 2000 = 2* x 5° the factors which are square numbers are 2?,2¢, 5?, 2? x 5, 24 x 5?. Trivially 1? = 1 is also a factor. Thus the squares of the six numbers 1,2, 4,5, 10,20 are factors of 2000. Answer: (B) 20. A quiz has 20 questions with seven points awarded for each correct answer, two points deducted for each wrong answer and zero for each question omitted. Ram scores 87 points. How many questions did he omit? (A) 2 (B) 5 (C)7 (D) 9 Solution: This question is same as question (17) of Primary Level screening test. Ram omits 5 questions and answers 13 correctly and 2 wrongly to get the score 13 x7~2x 245 x 0 = 87. Answer: (B) KAPREKAR CONTEST Sub-junion Level Final Test-2005 1. Let N be the greatest integral multiple of 8, such that no two ofits digits are the same, What is the remainder when NV ig divided by 1000? Solution: The required number N must have all the ten digits 0,1,2,-++,9. If it were to be a multiple of 8, the number formed by the right most three digits must be divisible by 8. As N is to be the greatest such number the left most seven digits form the number 9876543. Thus. the remaining three digits form the numbers 012,021, 102,120, 201,210. Of these only 120 is divisible by 8. Hence the required number N is 9876543120. When this NV is divided by 1000 the remainder is 120. 2. Show that it is' impossible to find a positive integer such that the sum of its square and its cube is an integral multiple of the square of the next higher integer. Solution: ° The problem is equivalent to proving that no positive integer « exists such that e4e3=k(2+1)? (1) where k is an integer. Obviously & is positive. As 2+1 0, cancelling of s+1, equation (1) reduces to’ 2? = Ket). As z and z+1, being successive integers, are coprime, x? cannot have a factor r+l. Hence (1) is not solvable for in positive integers. 3. If a 40, 540, ¢#0 andif t+}+ = 0 k+i+55 = 0 bebed = find the value of a+b+c. 34 Problems and Solutions Solution: From 2+} + sh =0, we got # = 4% and from 2414 4, = 0, we get y = —48424) Substituting these values in 5+ 3+} =0, we get ‘ 1 a+b a a(a+ 28) or (a +2b)(a-+ 2c) — (a + b)(a + 2c) — (2 + e)(o + 26) =0 or ~a(at+b+¢)=0. Since a #0,a+b+ce=0. The side AB of the parallelogram ABCD is produced-both ways to P and Q such that PA= AB = BQ. Given that AD = 2AB, find the ,, angle between DQ and CP. Solution: Let CP and DQ intersect at 0. We have to find the angle POQ. As AD = BC = 2AB, both triangles PBC and™DAQ are isosceles. (BP = BC,AQ = AD). Let ZBPG =2° and ZAQD =4°. Then from APOQ, ZPOQ = 180° - 2° -y°. But ZPBC = 180° ~ 22° and ZDAQ = 180° — 2y°. However from parallelogram ABCD, £PBC+ZDAQ = 180°. Thus 180° ~ 22° + 180°. 2y° = 180°. Therefore, 2° + y° = 90°. Hence 2POQ = 180° — 90° = 90° . In other words, PC and QD are perpendicular to one another. Find all positive integers n such that cash of the numbers n+l, n+3, n+7, n+9, n+13 and n+165 isa prime. Solution: Clearly, n cannot be ‘odd; because, then all the six Probleme and Solutions 35 numbers given will be even and cannot be all prime. (The only cvon prime is the number 2). Thus n can only be a number with 0,2,4,6 or 8 in the unit’s place. If n is any number ending with 0 (including 0), n+15 is divisible by 5 and hence cannot be prime. Similarly if n has 2 in the unit place then n+ 3 is divisible by 5 and is not prime. Likewise if n has 6 in the unit place, then n+9 will be a composite number being a multiple of 5. Again when n has 8 in the unit place n+7 will be a multiple of 5 and will not be prime. If 7 has 4 in the unit place and is greater than 10 then n+1 will be divisible by 5 and therefore will be composite. Thus the only choice is that n = 4. In this case, n+1=5,n+3=7,n+7 = lln+9 =13,n+13 =17 and +15 = 19 are all prime numbers. Thus the problem hag the unique solution n = 4. 2 . In how many ways two squares can be selected from a 8 x 8 chessboard so that they are not in the same row or same column? Solution: The first square can be chosen in 64 ways. The next square has to be located in a row and column different from the one in which the first square is. Hence there are 64 ~ 15 = 49 possibilities. Thus the two can be chosen gubject to the given condition in 64 x 49 ways, In these every two specified squares would have been counted twice as the order in which they are chosen is immaterial. Hence the required number of ways is } x 64 x 49 = 1568. x (a) Find the possible remainders when the cube of any integer is divided by seven. Solution: Every integer can be written as 7n +k for some integer n and k = 0,1,2,3,4,5 or 6. (ie., any integer can be 36 Problems and Solutions written congruent & mod 7, where k=0,1,2,--- or 6). (TREK) = (tn)? +3(7n")k + 3(7n)R? + = Tm+k where m is an integer and & =0,1,8, 27, 64, 125 or 216. These values of k® when divided by 7 will leave remainders is 1;1,6,1,1 or 6. Hence when (7n + k)® 4s divided by 7, the remainder will be 0,1 or 6. ; (b) Show that abc(a%b*)(65c)(c3a3) is divisible by 7. Solution: The result is true only if -a,b,c are integers. Suppose one of a,),c is divisible by 7 then obviously the given expression is also divisible by 7. Suppose none of a,b,c is divisible by 7 Then“a*, 83, c3 , by (a) above, will leave remainders 1 or 6 (0 is not possible as a,b,c are not divisible by 7). As there are only two possible remainders two of a?.8*,c> will leave the same remainder. Consequently o® ~ 6%, 5 — or c ~ a? will be divisible by 7. Hence, in any -case, abc(a? — b°)(b? — c)(c* — a) is divisible by 7. Between the digits 4 and 9 several fours and after them the same number of eights were inserted. For example between 4 and 9 we may insert three fours followed by three eights 444888 to get 44448889. Prove that the resulting number is always a perfect square. Solution: Between 4 and 9 let us introduce n number of 4' followed by n number of 8's. ie., let the number be feat 444-4 B8--.89, 7 ” ‘This number then can be written as. $4035 + dd gt 2n42 n+l = Problems and Solutions 37 Aciots? — 1) 4 Zor 41 4x 107 44x10" 4-449} 2x 104141)? {e x10" 4 wy} a (2s) Sum of the digits in 2x 10°41 41 is 3. Hence it is divisible by 3. Thus 241074! — 4 is an integer. ‘This proves that the number formed by insertion of equal number of 4's and 8's between 4 and 9 is a perfect square. lr olPoln 9.. From the set 1,2,3,....2005 in how many ways can one choose two different numbers whose sum is an even number? Solution: The’ total number of odd numbers in the given set is 200$+1 — 1003 and the total number of even numbers in the set is 2005 — 1003 = 1002. ‘The sum of two numbers is even if either both of them are odd or both of them are even, Two numbers from 1003 odd numbers can be selected in (1003 x 1002) or 502503 ways. Similarly the number of ways in which two numbers can be chosen out of 1002 even numbers is $(1002 x 1001) = 501501 ‘ways. Hence the totel numbers of ways in which two numbers from the given set can be chosen such that their sum is even is 502503 + 501501 = 1004004. 10. Can the numbers 4545 and 7-5 both be integers for some integer 7? Solution: M+ is an integer if 14z-+5 is a multiple of 9 or of 3. But as 14245 = 12z-+3+22+2, this is possible only if 22+2 is a multiple of 3. Similarly Maes is an integer if 172 —5 is 38 Problems and Solutions divisible by 3 and 4, This-means that 16a —6+ 22 +1 and hence 22 +1 is a multiple of 3. ‘Thus both the given numbers. 4345 and 16> will be integers (for integral «:) only ifboth (22-+2) and (22+1) are divisible by 3. This is impossible as (27-+2) and (20-1) are consecutive integers. 11. Find all integers x such that. 2c? — x — 36 is the square of prime number. Solution: Suppose 2a? —2—36 = (x-+4)(2¢—9) is the square of a prime number p. The factors of p? being +1,-tp,+p*, there can be six possible values for 2 +4, one of the factors of Pp. ; When 2 +4 =+1,2 = —3 or —5 and the other factor 22 ~ 9 will be —15 or —19. Then p= —15 or 19 and prime number p does not exist, When 2+4 = tp? then 2 = +p?—4 and Qa ~9 = 2p? ~ 17 = +1. No solutions for p exists. When a+4=p,c=p-4 and 2c-9=2p-8—9=2p—17=p or p=17. Hence x = p—4= 17-4 = 13. For this value of z, the given expression 22” ~2 ~ 36 = 17°. Also, when 2+4 = —p,z = —p—4 and 2e—9 = —2p—17 = —p so that p= —17. Then 2 =13 and 2c? —2—36 = 177. Thus only for z = 13 the expression 22° — 2 — 36 is the square of a prime number viz., 17. : 12, When I open my mathematics ‘book, there ara two pages that face me. If the product of the two page numbers is 2756 what are the two page numbers? : Solution: The consecutive say. Then clearly if x is odd a(x +1) = 2756 = 2x2 a? being 2500, we easily: 8 e+1=53. page numbers are x and z+1, ) @-+1 is even and vice versa. As 13 x 53 and n is to be near 50, ee that ©=2x2x 13 = 52 and Problems and Solutions 39 13, A walkers path in a park isas = 4 shown in the diagram. One can walk all along the circumference or along the line segments marked in D the figure. A child walks along the circumference- path starting at A and after going round returns to A. B Her grandfather starts at A and walks through the route AB — BC + CD -— DA.and thus returns to A. Who walked more distance, the child or the grandpa? Solution: If C is the centre of the circular path, then AB+BC+CD+DA<2r+r+r = 6r where r is the radius. The circumference = 2xr > 6r. Hence the distance travelled by the child will be longer than that travelled by the grand father. When C is not the centre there are possibilities for AB+ BC +CD+DA to be greater than 2ar, 14. The number A837B is divisible by 88. Find the digits A and B. Solution: Since 88 = 8 x 11, the number A837B is divisible by both 8 and 11. Divisibility by 8 implies that 37B is a multiple of 8 which in turn ‘implies that 5B is divisible by 8. Hence B = 6. Divisibility of A837B by 11 implies that (A+3+B) and 8+7 differ by a multiple of 11. ie, 94+A and 15 differ by a multiple of 11 or is 0. This shows that 9+ A—15=0 or A=6. Thus the digits A and B are both 6. 15. Suppose for integers x,y we have 2x + 3y divisible by 17; prove that 92+ 5y is also divisible by 17. Solution: Suppose we can write m(2z + 3y) = 17%(rx+ly) + 9x + 5y for integral values of r,! and m with m a coprime of 17 then clearly when 2x + 3y is divisible by 17, 92 + Sy is also divisible by 17. This means we have to solve in integers the equations 2m = 17r+9 and 3n= 1745. 40 Problems and Solutions ‘This is equivalent to getting integral solutions for 3(17r-+ 9) — 2(171 +5) =0. or 17(3r — 21) = —17 or Br — 2h =—1. Many solutions exist. One such is r = | = —1. In this case m = (~17+9)/2 = —4. Thus —4(22 + 3y) = —-17(z+y)+ 9a +5y. Hence 9z + 5y is divisible by 17. When r = 1,1 = 2, we will have m = 13. Even now 13(2z + 3y) = 17(@ + 2y) +92 + 5y and the result follows. 16. (a) Can the sum of 2005 odd numbers be zero? Solution: 2005 being odd, the sum of 2005 odd numbers will be odd and hence cannot be zero. (b) Let a1,a2,43,--- ,@, be any rearrangement of 1,2, 3,-++,n, (For example 2, 4, 3, 1, 6, 5 is a rearrangement of 1, 2, 3, 4, 5, 6). If n is odd’prove that (a, —1)(a2—2)-++(an—n) is an even number. Solution: When n is odd, then there will be r even numbers and r+1 odd numbers in 1,2, »n= 2r+1. Of the numbers (a; — i) where a is odd at least one number will be even corresponding to a; and i both being odd. Hence the product (a1 — 1)(@2 — 2)+++ (aq —n) contains atleast one even number as factor proving that the product is even. 17. Find all possible values of n | such that a rectangular board 9x can be covered with tiles of the shape. Solutio: Each tile is of ry area 3 unit square. Two tiles £ together will form a 3 x 2 at rectangle as shown below: Problems and Solutions 41 Hence in a 9 xn array there must be three such blocks constituting two rows of 9 x 2 rectangular board. When n is even we can split 9 xn into % blocks of 9 x 2 rectangular boards. When n is odd this-is not possible. Hence for all even n, the problem is solvable. 18. A triangle ABC has its circumcentre at O and M is the midpoint of the median through A. If OM is produced to N such that OM = MN, prove that N lies on the altitude through A. Solution: Let D be the mid point of BC so that OD 1 BC and AD is the median through A. Join (A,N) and produce it to meet BC in B. In triangles AMN and DMO, we have NM=OM AM = DM ZAMN = ZDMO. A ». AAMN = ADMO. (SAS property) Hence ZANM = ZDOM. $0, AN || DO or AB || DO Ss «AE 1 BC(: DO 1 BC) Bas. > Cc :. AE ig the altitude from A and it passes through N’. KAPREKAR CONTEST Sub-junion Lovel Screening Test-2006 1. Given the alphametic where each ABCD letter represents a different digit, -BODA the value of C in the least value for Ts1 2 ABCD is HEE (A) (B) 2 (c)6 (D)7 Solution: The first digit from the left is supposed to be nonzero as usuel. So the second number. BCA is more than 1000. Since the difference is more than 1000, we see that the first number ABCD is more than 2000. Suppose that A= 2. Working from right we get successively, D=2+2=4, C=4+1=5, B=5+5=0 with carry 1. . Next let A= 3. We get D=5, C=6, B=1 with carry 1. This is not possibl The ABCD = 3165 and BCDA = 1653. The difference is 1512. Any larger value for A gives only a larger value for ABCD. Thus the answer is C. 2. If 36a4 = 0°, then a? is equal to (A) $oS —(B) Gat’ = (C) 2a? (D) Ga? Solution: 3604 =a =a? = 36; a=6, a® = ax a? = Gu?. So the answer is D. . The weight of a dog is 8 kg plus one third its weight. The weight of the dog is (A) ky. (B) 12 kg. (C) 14 kg. (D) 15 kg, az Problems and Solutions 43 Solution: If the weight of the dog is dkg, then d=8+}d=> jd=8>d=12. The.weight of the dog is 12 kg. The answer is B. 4. Sum of all integers leas than 100 which leave a remainder 1 when divided by 3 and leave a remainder 2 when divided by 4 is (A) 416 (B) 1717 (C) 1250 (D) 1314 Solution: Since the numbers leave.a remainder 1 (2 less than 8) on dividing by 3 and 2 (2 less than 4) on dividing by 4, the numbers are the common multiples of 3 and 4 diminished by 2. Such numbers less than 100 are (4x 3-2) or 12-2 = 10, 24-—2= 22, 36-2 = 34, 46,58, 70, 82,94. That is, there are 8 numbers whose sum is 416. The answer is A. Note: The numbers form an Arithmetic sequence, ie., two consecutive numbers differ by the came constant 12, the 1 cm of 3 and 4: So the sum is $[10 +94] = 4x 104= 416. 5. Let n be a3 digit number such that n =.sum of the squares of the digits of n. The number of such n is (A)o (B)1 (CG) 2 (D) more than two Solution: Let the digits of the three digit number be abc in that order, satisfying the condition of the problem, i.e., 100a+10b+¢ = a? +b*+c7 = a(100—a)+0(10—b)+-e(1—c) = 0 and 00. So. a(100—a) +4(10—b) + ¢(1—c) is never zero or there exist no number satisfying the given condition. The answer is A. 44 Problems and Solutions 6. A three digit number with digits A,B,C in that order is divisible by 9. A is an odd digit and C’ is an even digit. B and C are non zero. The number of such three digit numbers is (A)4 (B) 8 (C) 16 (D) 20 Solution; A+B+C = 9,18 or 27 and A is odd and C is even, B,C non-zero. We shall fix the digit A as 1,3,5,7 and 9 and find the corresponding values of B and C and count the number of numbers ABC ABC ABC ABC ABC 1) 162 5) 342 9) 522 13) 792 17) 972 2) 144 6) 324 10) 594 14) 774 18) 954 3) 126 7) 396 11) 576 15) 756 19) 936 4) 198 8) 378 12) 558 16) 738 20) 918 Thus there are 20 such numbers. The answer is D. 7. The number of prime numbers less than 1 lakh, whose digital sum is 2 (digital sum of a number is the sum of its digits) is, (A) 5 (B) 4 (C)3 (D) none of these Solution: The numbers less than 1 lakh where digital sum is 2 are 2, 11, 101, 1001 and 10001; of these 1001 = 11x13x7 and 10001 = 73 x 137 and hence are not prime numbers. There are just 3 prime numbers less than the lakh, with digital sum 2. The answer is C. 2 . The quotient of 100! and 508 iy (A) 50° (B) 50% (C) 20059 (D) 40050 Solution: 1001 _ (2 x 50) 5080 Boe — = 2 x 505 = 450 x 5950 _ 9960 The answer is C. Problems and Solutions 45 9. A sequence of numbers Tj,7>,T3,+++ :Tn;+* is constructed using the rule that the nth term Ty, has m ‘n's; for example the 25th term Tys = 25252525---25, where 25 is repeated 25 times. The term containing-the same number of 2s as Toa¢ is (A) To22 (B) Taso (C) Tasa (D) Tasa Sohition: The number of 2’s in Tyg = 2 x.226 = 452. It is easy to see that there are 452 twos in Tis2- The answer is D. 10. The least number of numbers to be deleted from the set {1,2,3,-++ ,13,14,15} so that the product of the remaining numbers is a perfect square is (A) (B)2 (C)3 (D) 4 The product of all the numbers from 1 to 15 (Le. 15!) is 1x 2! x 35 x 59 x 7? x 11! x 137 =2x 20 x 38x 52x 5x 7? x 11x 13 =1x 2! x 39 x 5? x 7 x 10x 11x 13 (Taking the product of 2 and 5). When the numbers from 1 to 15 expressed as a product of even powers of prime numbers, the numbers 10,11,13 have odd powers. So these three numbers to be deleted so that the product of other numbers become a perfect square. So the. answer is C. 11. The digit 1 is attached to the right of a 3 digit number making it a 4 digit number which is 7777 more than the given number. The sum of the digits of the number is (A) 23 (B) 18 (17 (D) 16 Solution: Let the.3 digits of the givon number be a,b and c, s0 that abcl may be the 4 digit number. 46 Problems and Solutions The difference between the numbers is (2000a + 100 + 10¢ +1) ~ (1000 + 106+ ¢) = 7777 => 9(100a + 10b-+¢) = 7776 => 100a-+10b +c = 864 (which is the given number) So. the sumn of the digits of the given number is 18. The answer is B. 12. A number is formed by writing the first 10 primes in the increasing order. Half of the digits are now crossed out, so that the number formed by the remaining digits without changing the order, is as large as possible. The second digit from the left of the new number is (A) 2 (B) 3 (C)5 (D)7 Solution: The initial number is 2357111317192329. This has 16 digits, 8 of which are to be deleted, To get the largest number the leading digits should be as large as possible. Starting with 9 there can be only at the most 5 digits. So 9 can neither the first digit nor the second digit. The first two digits can be 7. The largest possible new number is 77192329. ‘The answer is D. 13, Nine numbers are written in ascending order. The middle number is also the average of the nine numbers. The average of the 5 larger numbers is 68 and the average of the 5 smaller numbers is 44. The sum of all the numbers is (A) 540 (B) 450, (©) 504. (D) 501 Solution: Let the middle number be m, so that the average of all the numbers is also m and hence sum of the numbers is 9m. The sum of 5 larger numbers 5 x 68 = 340. Problems and Solutions a7 ‘The sum of 6 smaller numbers 6 x 44 = 220 Since m™ is the middle number, it is included in the larger 5 es. well as smaller 5. Therefore, (340 ~ m) + 220 = 9m or 10m = 560 = m = 56 and 9m = 560 — 56 = 504. The answer is C: 14. In the adjacent figure BA and BC are produced to meet C'D and AD produced in E and F. Then ZAED+ZCFD is (A) 80° (B) 50° (©) 40° (D) 160° if E Solution: In A’ AED and CFD, ZAED+ZCFD = (ZDAB — 90) + ZDCB-90 (ZAED = ZDAB~ZADE and ¢CFD = ZDCB- CDF) = ZDAB+ZDCB — 180 = (360 — 140 — 180) = 40°. The answer is C. 15, Inatriangle ABC, BE is the angle bisector of ZABC , where E lies on AC. EF is the angle bisector of ZBEC , where E lieson AC’. Also EF = EC. Then, (A) AB= BC (B) AC=BC (C) ZABG = ZACB = 72 8 (D) 2BAG = 12" 48 Problems and Solutions Solution: All the angles marked z are equal. ZEGF = ZEFC = 2c (ext 2 of ABEF. In AABC, ZA4 ZB + 20 = 2+ 2x + 2x = 180 => r= 36°. The answer is C. 16. In a box there are some coins and rings which are made of either gold or silver. 60% of the objects are coins, 40% of the rings are gold and 30% of the coins are silver, The percentage of gold articles is (A) 24% (B) 16% (C) 42% (D) 58% Solution: We shall have a tabular colunm and fill it up with the given data and unfilled column can be calculated as given below: ff Coins Rings Total Gold [60-18 = 42% | 40% x 40% = 16% | 58% Silver | 30% x 60% = 18% | (40-16) = 24% | 42% ‘Total 60% (100 = 60) = 40% So 58% of the articles are gold. The answer is D. 17. Ina plane 3 lines and a circle are given. If points of intersection of two lines or that of a line with the circle are counted, the maximum number of points of intersection possible in this is (A122 (B)9 (GD) Solution: 3 lines give a maximum of 3 points of inter- section. Each line intersects the circle in at the most 2 points. So 3 lines give 6 maximum of 3 x 2 = 6 points of intersection with the circle, Probleme and Solutione ae Thus the maximum possible number of points = 3-+-6:=9. The answer is B. 18, In un inowculus auto angled triangle one angle is 50°. I. The other two angles are 66° and 66%, I, Tho other two angles are 50° and 89° LIL. The other two angles can be anything. Thon which of the following statements is true? (A) Tonly (8) Monly (C)Toand I only (D) Ill only Solution: One angle is 50°. If this is one of the equal angles, then the other angle is 180° — 100° = 80°. If this angle is not equal to any of the other two angles then each of the other two ZS = 160 — 65°. Thus, either the three angles are 50°, 50° and 80° or 50°, 65° and 65°. Either Lor If may be true. Thus the answer is C. 19. Let n be the number of integers less than 10,000 which are divisible by all integers from 2 to 10. Then (A) n=0 (B) 1 AB? — AC? = BH? ~ CH? 60? — 80? = 2? — (100 — x)? => 60? — 80? + 100? = 2002 => 200x = 2 x 60? => x = 36, The answer is A. 25. Let the length of a positive integer n be defined as the number of prime factors of 7, counting repetitions. Length of 36 is 4 as 36 =2-2-3-3. Length of 64 is 6 as 64= 2°, The number of numbers less than 100 with maximum length is (A) 5 (B) 4 (c)3 (D) 2° Solution: 64 = 26 =2x2%2x2x2x 2 has length 6. 96 = 25x 3=2%x2x2x2x 2x3 also has length 6, One can easily verify that any other two digit number has & length smaller than 6. The answer is D. Note: Suppose that the length of n is not less than the length of m for any m m2 <2 m 1/3 and -1 CL = V3 and DE = iunit . LM.= AD - (DL + AM) =4-(1+1) = 2unite. :. Area of Trapezium = } x CL x (BC+ AD) =} x V3 x 6 = 3V3sq units ~. Area of shaded portion = (3V3 — $7) sq units. 4JE 1 PgUnE. MIA AAI se ee 5 that zai tyro tz 2006 ~ } Prove thal 2 Pe 2 seen act la Baa} Ptay * A+ 20002 : 1 2 2006 Solution: 7 + 555 + yaoog ~ 1 i given pelos, yb2oy 2420082 | z+ yro 2+ 2006 2 +2006 z+ 2006 z “72006 = + < 1 z+ 2006 J ~ 56 Problems and Solutions If zy, are non zero, then 2 va 2 + 20062 3 Prat Pry Thus it is proved. 5. A 2006 digit number has the property that every two consecutive digits as a two digit number is a multiple of 17 or 23, Find the maximum sum of the digits of such 2006-digit numbers. Solution: If a,b are two successive digits, then the 2” digit number ab is a multiple of 17 or 23. So ad is 17.23. 34, 46, 51,68, 69,85, or 92. Hence, none of the digits can be 0, and 7 cannot be followed by a digit. ~ If8 is a digit other than the last, then it is followed by 5. If 5 lias @ successor, it is 1, and if 1 has a successor it is 7. Thus, we see that none of the digits, except the last four, can be 8,5,1,7. In other words, the fist 2002 digits must be 2,3,4,6 or 9. These 5 digits can’occur any number of times in that order as 2346923--- The first 2000 digits consists of 400 groups of these, so their sum js 400(2+3+4+6+9) = 9600. Consider the possibilities of the last 6 digits . If the first digit is 2. then the last 6 digits are 234692 or 234685. Other possibilities of the last 6 digits are 346923. 346851. 469234, 468517, 692346, 923469, 923468. Hence the possible, values of the sum of the last 6 digits are 26, 28. 27, 27, 28. 31. 30.33, 32 The maximum value of the sum of digits is 9633. Problems and Solutions 57 6. Let p and q be positive integers such that _,-1,1_1 (aid sla gt gat at Bs ais Prove that 83 divides p. Solution: Given eee where p,q are positive integers. Pa(itz+}+ +z +ote+ +3) 7 = (14egegee + +3) = 2a Feet at » F(Ge8) (Bed) (ard) e883 Raine, 89 > Oo ext xe TKR Let N be the produced of all integers from 28 to 55: Multiply both sides of the above equation by N and q. We get pN = 880+ 83) + 83c-+ +++ where a,b,c-++ are integers, ‘The right hand side of this is e multiple of 83. Note that 83 is a prime number, and that NV is not divisible by 83. So p must be divisible by 83. 58 Problems and Solutions 7 Find the sum of 2006! + 20061 +---+ 2006 2006 times +2006! + 200619 +... + 200678 2005 times +2006? + 20061? + «+--+ 20061? +. ++ 2005 times + 20067 4. 20067008 +... + 20067005 2005 times Solution: Given expression reduces to 2006 x 2006! + 2005 x 20061 + 2005 x 20061? + -- +2005 x 20067005, = 20061! + 2005 x 2006! +2005'x 2006*? +. 2005 x 2006209 + + +2005 x 2006202 = (1+ 2005) x 2006! + 2005 x 2006!92+.2005 x 2006103. +2005 x 20962005 = 20061? + 2005 x 20061? +2005 x 200613 +2005 x 200614 + +. #2005 x 2006705 Inductively we get finally 200674 + 2005 x 200674 +. 2005 x 2006205 = 200675 + 2005 x 2006705 = 2006706 .. Expression finally reduces to 20067 , -. 2006" + 2005 x 2006 w (1+ 2005) x 2006* 2006 x 2006" = 2006+! i Problems and Solutions 59 8. Find the integral solution for x and y:a7+615 = 2", Solution: Given 2? +615 = 2 where x,y are integers. As 2? >0, we have 2¥ > 615. But 615 is not a perfect power. of 2. ; 1 Strictly we get 2” > 615 =>y2>10 as 20 = 1024 and 29 = 512. Hence RHS of =? +615 = 2” is even. => 2? is odd =z is odd « As 2U ends with 2 or 4 or 8 or 6 only. ‘We get x should correspondingly end with 7 or 9 or 3 or 1. But no perfect square ends with 7 or 3. .2¥ should not end with 2 or 8. This happens only when y is odd positive integer. .y should not be odd. => y must be even Let y=2k where KEN. Now the equation reduces to x? 4615 = 2%. (or) 615 = (2*)? -— (2)? © 615 = (2 +2)(2* -2) ‘The above equation hai the form of 615 expressed as product of two odd factors of 615 whose sum is an exponent of 2. nt (Qt em) + (2* — 2) = 2.2% = Qht only 615 = 123 x 5 satisfies this, as 12345 = 2", 60 Problems and Solutions Hence 2* +2 = 123, 2*-2=5 and k=6. 2 123-5 =59 2 = and y = 2k=12. . There are two integral solutions in (x,y) for 2”+615 = 2. They are (59,12) and (—59,12) as 59? = (—59)*. 9. The number of smallest semicircles drawn on the diameter AB = 40.12mm is 2". The pattern as shown in the diagram is drawn. Show that the sum of the arc length of all the semicircular arcs is (n+ 1)20.06mm. 4 40.12 mm : S 20.06 mm : € 10 mi Fé Solution: ot Circumference of the largest semicircle = (20.06) mm. Circumference of the 2 smeller semicircle = 27r(2228) mm. Circumference of the 2? semicircles = 2? (2998) mm. etc Circumference of the 2° semicircles = 2"2(2298) mm. , Total circumference of all semicircles = (20.06)r[1 +1+1+4.--+ (n+ 1) terms} = n(n -+1)(20.06) em Probleme and Solutions on 10, Find the largest number that cannot be expreased as a multiple 5, ns a multiple of 8 or as the eum of multiple of 5 and a multiple of 8. [Example: 15 is a multiple of 5, 24 is a multiple of 8, and 26 is the sum of 10 and 16 which are the multiples of 5 and 8.] Solution: First observe that 27 has the desired property. 28 = 20+8, and 29 = 5424. Any integer n > 29 is of the form n = 5k, Sk+1, 5k+2, 6k+3 or 5k+4 where k>6. Now, 5k+1=5(k—3)+416, 6k+2=5( -6)+32, Sk+3 = G(k-1)+8, 5k+4 = 5(b—4)+24. No such z has’ the property . Hence the largest integer with the stated property is 27, 11. ABC is a triangle, with ZABC = ZACB,BD,DE 4 and EF bisect angles LABE,ZBDC end LDEC respectively, EF = EC. Calculate the angles of the triangle ABC. Solution: In AABO, ZB= LC = 22 E In AEFC, BF = EC <. ZBFG = LFCE = 22 CDEC = 2z eum of internal opposite angles of ABDE=a+y. or z= 3 In ABDC, sum of the angles = 2242+ 2y = 32-+2y = 180°. 62 Probleme and Solutions Sum of the angles of ABFC = 2x + 2x + 2 = 180° or 4a + z= 180° Therefore 4+ 2 = de + $4 = 180 = Or + y ='360- Solving 32 +2y = 180 and 9+ y = 360, we get y= 36 and © =72. «Angles of ABC’ are 72°, 72° and (180 — 144) = 36°. 12, ABCD isa rectangle AE = ED = DF = FC = }AC = AM. Show that the eum of the areas of AAED and ADFC is equal to } area of rectangle ABCD. Solution: Let -M be the midpoint of AC. Since ZADC = 90° ; we get MC = AM =MD. Thus AEDM and MDFC are shombuses (since all sides of AEDM and thet of MDFC are equal), Thus, Area of ADCF = Area of AMDC. Area of AAED = Area of AADM. Thus Area of A(AED) + ADFC = Area of AACD = } area of rectangle ABCD. KAPREKAR CONTEST Sub-junion Loved Sereening Test-2007 1 a,b,c, dare reals such that a — 2005 = b + 2006 = c— 2007 = d+ 2008, The greatest among a,b,c,d is (A) a (B) 6 (C)e @) 4 Solution: Let a — 2005 = b+ 2006 = c— 2007 = d+ 2008 = k. Then a = k+2005 And k — 2008 < & — 2006 b = k—2006 e=81 -. highest mark scored = 31+ 48 = 79. Answer:(C) 4. In a triangle if one angle is greater than the sum of the other two angles then the triangle is (A) Acute angled (B) Right angled (C) Obtuse angled (D) Equilateral Solution: Let ABC be a triangle where ZA > ZB + 2C. Adding ZA to both sides of the inequality, we get 2LA > LA+ZB+ LO. ie. 2ZA > 180° (or) ZA > 90°, Hence the triangle must be an obtuse angled triangle. Answer: (C) 5. The largest positive integer n for which 2° < 6300 ig (A) 12 (B) 13 (9) 17 (D) 14 Solution: 1° < 6° = n? < 68 (or) n? < 216. ‘The largest positive integer n satisfying n? < 216 is n = 14, Answer: (D) 6. Three identical rectangles are 200m overlapping as in the diagram. '™ The length end breadth of each réctangle are respectively 2007cm and 10cm. The area of each of the shaded square portions is 16cm? Problems and Solutions 65 ~ ‘The perimeter of the outer boundary of the figure in cm is (A) 10070 (B) 12070 (C) 14070 (D) 11070 Solution: Area of each’ of the shaded square portion = 16cm? (given) = side of each of the shaded square portion = 4em. Also given that the 3 rectangles are identical. Hence the figure will be as follows: 2007em :. The perimeter of the outer boundary of the figure 2(2007cm + 10cm) +.4(2003cm + 6cm) 4034em + 8036em = 12070cm i) Answer: (B) Alternate Method: 3 (Perimeter of the identical rectangle) ~2 (Perimeter of the inner square) = 3{2(2007-+ 10)] ~ 2(16) = 12102 — 32 = 12070 . 20% of 50% is what percent of 25% of 40% ? (A) 80% (B) 60% (C) 65% (D) 100% Solution: Let 20% of 50% = 2% of 25% of 40%. We need to find x’, 66 Problems and Solutions 2080 AO 700 * 100 ~ 100 * 100 ™ 160 => 2x50 = x 25x40 100 z => 1000 = Foo * 1000 =>x2 = 100 Answer: (D) 8. In the adjoining figure A = 60°,C = 50°. ZBDG = 30°, ZGEF = 20°. Then (A) BG =2FG (8) EG> FG A (C) EG=FG D ? c (D) EG< FG Solution: ZABC = 180° — (60° + 50°) = 70°. Now ZGBD = 180° ~ ZABC. . GBD = 180° — 70° = 110°. ZDGB = 180° — (30° + 110°) = 40°. Hence LEGF = 180° — ZDGB = 180° — 40° = 140° => LEFG = 180° — (20° + 140°) = 20°, In AEFG,ZE = ZF = 20°. Hence EFG is an isosceles triangle, => EG=GF. Answer: (C) Problems and Solutions 67 9. A boy on being asked what 3$ of a fraction was made the mistake of dividing the fraction by 18, and got an enswer which exceeded the correct answer by Sf. The. correct answer is (A) & (B) & © 2 (D) None of these Solution: Given Wrong Answer = Correct Answer + ii ie, c+ 49 = }8 of c+ 3s where z is the-indicated fraction > Uz alte Me 4 88 (or) Mp — ee = ; (289-256) ue OH By a (or) z= 4 => Correct Answer = 18 x 4 = Answer: (A) 10. 2,y,z are three sums of money such that y is the simple interest on 2 and z is the simple interest on y for the same time and rate then (A) 22=yz (B) y= 22 (C) 2 =cy (D) None of these Solution: Given y,z are simple interests on x,y respectively at the same rate for the same time. I= PNR/100. Let MB =k. “. y=ki and z= ky for some constant k. ie. y2=22, Answer: (B) 11. How often the hands (hour hand and minute hand) of a clock are in a straight line every day? “ (Ay2 (B)4 (C) 16 (D) 44 Solution: Let us divide the 24 hours in a day into 24 classes, 00:00 to 01:00 AM, 01:00 to 02:00 AM, 02.00 to 03:00 AM, 23:00 PM to 00:00. 68 Problems and Solutions ‘To have the hands of the clock to be in a straight line, they should either coincide (or) point in exeetly opposite directions. SLOT *. Number of times the hands lie on a straight line 11:00 AM - 01:00 PM 3 41:00 PM - 01:00 AM 3 05:00 AM - 07:00 PM 3 05:00 PM - 07:00 PM 3 01:00PM — 02:00PM, 02:00PM ~ 03:00PM, \ other ibr slots each .. Number of times the hands lie on a straight line in a dey =3434+3434+2x 16 = 44, Answer: (D) 12. Tt is given that 2 = $, which one of the following is incorrect (a) = 8 ) HE = 2B (C= 8 ~D) BEY = gy Solution: Given: 2 = { a) Adding 1 to both sides of the equation, we get ety :. Option (A) is true. z 4 zeiectl eo 3st > Q) s2=$ o¥42=$42 (or) HE. Hence option (B) is also true. 2 Aw StH ay Hence option (C) is also true. Problems and Solutions 69 any 4_5 Sipe ee = £28 “5 4 _ 32-25 7 =~ 20 «20 22? 7 9 or ST = a5 but not 3 Hence (D) is incorrect. Answer: (D) 13. If a,b, o,d are positive integers such that a = bed, b = oda,c = i (atb+ce+d dab and d= abc, then the value of aa 8 (A) 4 (B) 2 (1 () 3 Solution. Given a = bed; b= cda; c= dab; d= abe where a,b,c,d are positive integers. Now a+ = bed + cda = cd(a +0) a+b=cd(a+b) > cd=1 as a+b £0. Similarly b+¢ => da=1; c+d = ab and d+a= bc = 1. But ab=cd=da=be=1>a=b=ec=d=1 (a+bt+e+ds — (141414134 (b+be+ed+day? ~ (1+14+1+1)2 4 = ofa. .”. None of the options is correct, Note: If the power in the denominator had been given as 4, the , answer will be option (C). See also Q18 of Bhaskars. Contest page number 37. 70 Problems and Solutions 14. The sum of two numbers is 1215 and their GCD is 81. How many pairs of such numbers are possible? (A)2 (B) 4 (6 * Ws Solution: Let the 2 numbers be 8le, 81b where a,b are coprimes; a 69! = 12 (2) ‘Also wn = 60!-o-b = g = 60!-e- => 60Ieo = 4 8) From (2), we get 1274 = 60 and from (3), we get 60! = 2. s. (tarts) HH" = got! = 2 = 12405} = 2, Answer: (D) 16. The number of digita in 8'%54° (whon written in base 10 form) is (A) 42 (B) 45 (C) 55 (D) 2007 Problems and Solutions 71 Solution: alt x 5 Wl (2x 2x 2) x 5° = x5 = yo x 540 x 2 = 32x10 = 32000...00 .. The number of digit in the expansion of 8" x 5“ will be (2+ 40) = 42. Answer: (A) 17. In a school, there are 5 times as many boys as girls, and 6 times as many girls as teachers. If b,g,t represent the boys, girls and teachers respectively the total number of boys, girls and teachers in the school is (A) 37 8 (B) $b (C) 30 9 (D) 37 9 Solution: Given b = 59; 9=6t (or) t= $= & Hence bib, b btgtt = G+Et a = MES) _ TH a 30 ~ 30 Answer: (B) 18. Consider the equation 2? + y? = 2007. Given = is a real number end y is a natural number, the number of solutions of the equation is (A) 0 (B) 2006 (C) 88 (D) 44 Solution: Given 2? + 4? = 2007 where z is a real number and y is a natural number. ‘We have y? < 2007. Hence y, can take values from 1 to 44 as 45? = 2025 > 2007 > 44. For every value of y, there are 2 72 Problems and Solutions corresponding real values (one positive, other negative) for z. Hence the number of solutions = 44 x 2 = 88. Answer: (C) 19. The number of two digit numbers whose digit sum is divisible by 6 is ‘ (A) 13 (B) 8 (7 (D) 22 Solution. The sum of the digits of such two digit umbers can be 6, 12 or 18, <. The numbers 15,24,33,42,51 give a digit sum 6, the numbers 39,48,57,66,75,84,93 give a digit sum 12 and 99 gives a digit sum 18. o. There are (5+7-+1) = 13 such 2 digit numbers. Answer: (A) 20. A number when divided by 899 gives a remainder 63. What is the remainder when the number is divided by 297 (As (B) 28 (16 (0) 12 Solution: As 899 = 29x 31, when 29 is removed repeatedly from the given number, 63 is one of its end results. Further proceeding, we get N ~29 63 ao 29 : 34 . ~29 29 oo ‘ 63 ‘The required rematuder-is therefore 5. ‘Answer: (A) Note: See also Q.18 of Gauss Contest at page ‘number 9. Problems and Solutions 78 21. A man gets (¢)** of Rs.100 and (2)** of Rs.100 again. He gives away Rs.200. Then the man (A) loses by the transaction (B) will not lose by the transaction (C) loss or gain depends on whether a > bor a < brespectively (D) loss or gain depends on whether a < 6 or a > 6 respectively. 2 Solution. ¢ x 100+ # x 100= eae x 100 (@) 2 For positive a,b, we have 2+ » 9 ab ‘This is because a? + 6? — 2ab > 0 (or) (a — b)? > 0 for reals a,b. +e ab Answer: (B) Hence x 100 > 2 x 100. 22. A six digit number is formed by repeating a three digit number twice (like 245245). Such numbers are always divisible by (A) 1001 (B) 25 (G) 101 (p) 11 Solution. Any 6-digit number of the form ABCABC can be got by multiplying it by 1001 as shown below. ABC x 1001 ABC ABC ABCABC Answer: (A) Note: We can find numbers of this form not divisible by 25, 101 and 111. Find them. 4 Problems and Solutions 23. Twenty four children are seated equally spaced around a circle and numbered from 1 to 24. what is the number of the child who sits diametrically opposite to the child number 10 (A) 21 (B) 23 (0) 22 (D) 20 Solution. If the numbers are arranged from I to 24 in ‘the order in a circle (natural numbers only), then every pair of diametrically opposite number differ by 12. Such as 24—12=12; 18-6=12; 13-1=19; etc. ., The diametrically opposite number to 10 will be 10+12 = 22 so that 22—10= 12. Answer: (C) 24. The lengths of the altitudes of a triangle are in the ratio 1:2:3. Then 5 (A) one angle of the triangle must be 60° (B) the triangle is a right angled triangle (C) the triangle is an obtuse angled triangle (D) such a triangle does not exist Solution: Given: The lengths of the altitudes of the triangle are in the ratio 1: 2:3, .. The side lengths of the triangle will be in the ratio of dibs bie, 6:3:2 Because the area of the triangle = $x base x height, But the side lengths of the triangles cannot be of 6K units, 3K units and 2K unit because (3K + 2K) is not greater than 6K. Answer: (D) Problems and Solutions 75 25. OP, OQ, OR axe rays OS bisects ZROP,ZPOQ = s} @ 62°, ZROQ = 102°. Then SOQ = i (A) 20° (B) 15° 0 (G) 25° (D) 10° Solution: i R 2 ) 102° 164 62° P o Po a w s 2 ° = P o . . Civ> Gi Solving by Frames; we get ZSOQ = 20°. Answer: (A) 26. If z,y,z,0,,c are real and none of these quantities is zero zy =a,zz=b and yz=c then 2?+y?+27= 2 (A) ®@ +P 42 @®) nee, 2 ©) @ or a ©) ors oe +(e) Solution. Given zy =a; 2z=6 and yz=c, Problems and Solutions 76 Let K = xyz. Then ay-az = a-boke Similarly, az-yz = b-c=kz end = yz-ay = c.a=k-y=ky and Kk? =abe .1. wo have k(x? + y? +22) = (ab)? + (bc) +, (ca)? 2 2 = Pa yte se = Et We? +o) eee Answer: (D) 27. The perimeter of a right angled triangle is 144cm and the hypotenuse is 65cm. Its area in square cm is (A) 508 (B) 508 (C) 1440 (D) 504 Solution. Let the legs of the right triangle be of lengths acm, bem respectively. a+b+65 = 144 = a%b=79. (1) 65cm bom Also, by Pythagoras Theorem, we have a? +B == 65? (2) ul a 2 Yb = (a+B)? — (02 + 82) > Dab = 79) — 65? = (79 + 65)(79 ~ 65) => Mba Moos {or) Area of triangle = }ab = 36 x 14 = 504em? Answer: (D) Problems and Solutions 7 28. The number A7389B where A,B are digits is divisible by 72, then (A,B) is (A) (6,3) (8) (3,6) (©) (4,7) (@) (7,5) Solution. A7389B is divisible by 8 as well as 9. +, The last 3 digits ie. 89B should be divisible by 8 ie. 800 + 80 + (10+ B) should be divisible by 8 => B must be 6 as B is a single digit. Now, the Sum of all the digits of the number should be divisible by 9. + A+7+3+84+9+4B should be divisible by 9. Answer: (B) 29. On the sale of 10m cloth, a gain equal to the selling price of 2m cloth was obtained. The gain is (A) 20 % (B)10% (C) 25% (D) 40 % Solution: Gain on 10m cloth = SP of 2m cloth = SP of 10m cloth — CP of 10m cloth => SP of 8m cloth = CP of 10cm cloth Subtracting CP of 8m cloth both sides, we get SP of &cm cloth — CP of 8m cloth = CP of 2m cloth Gain on 8m cloth “GP on 8m cloth CP 2m cloth GP on 8m cloth 2 = BX 100% = 25% © Gain% = Answer: (C) 78 Problems and Solutions 30. Three Circles .C,C2,Cy with redii ri,tz,ts where 71 <12<7g are placed as in the figure. ‘Then 12 =~ (A) vis= (8) Vet () V- ) yan Solution: Let us construct the figure as above. By applying Pythagoras theorem to A’s CCQ, and CaCoR, we get OQ =Vii+n!— Gn)? = Vir = 2am (1) GR= Vrs +1)? = (ra ra)! = Vira = 2/7 (2) By applying Pythagoras theorem to ACLPOs, we get (3+ 2r2 +11)? = (rs ~ ri)? + 4ralVra + 75)? Probleme and Solutions ue) 13+ Ark +1? + Arava + 4rari + Ora =rk+r} — Qrary + Arari + 4rars + BroVrirs ‘This reduces to 4r3 + 4rar1 ~ 8r2/7aT1 =0 => rh -2reJrari + ram =0 => (r2— Vrari) = 0 > r= Vist Answer: (D) KAPREKAR CONTEST Sub-junion Loved Final Test-2007 “L The sum of 2000 numbers is 2007. Find the maximum product of these numbers. [There are more than one set of such numbers] Solution: As real numbers can be classified as positive real numbers, negative real numbers and zero, one can choose both Positive and negative real numbers to obtain 2000 numbers. If we choose two positive real numbers, a1,a2 and 1998 negative real numbers "43, —04,~G5, —Gg,°-- ,—Ai999,—Gapo9 the product will be 91 09°03-04-A5 --« @1999°G2000 Where all a; > 0 for 1 P. Note that the sum is retained as 2007, Similarly for some k>m, k,m € Rt, one can find Q = (a1 + k)(aa +k) 0304+ g5-+- (2000 + k) R= (a, +m): (a2 +m)+a3-a4+ 05+ (@2900 + m) Hence there exists no maximum product. 2. The fourth and fifth term of a sequence are 4 and 5 and the nth term ta is given by tn = 2+ tai —tyg for n> 3, For example Ox998 * (a1g98 + k)> *G1998 * (a1999 + mm) Problems and Solutions on ty =Qts—ty = 2x5-4 6. Find the first term. Also find the sum to 2006 terms (from ty to too06)- Solution: Given ty = 2+ty-1—tn-2 for n23 where tn is the n‘® term => th— tna = tai — tha forall n> 3 => tapos — t2005 = taoos — tao = teooa — t2003 = *** =ty—ty=ty-tg=ty-te= h-hh =) because ts -t4=5-4=1. te-tg=1 > 4-tg3=1 = t3=3 ty-tp=1 => 3-t=1 4 t=2 tf—-t=1l = %h=l >=) Now ty-ta-1=1 + t=t-aitl>h=n as ty =1 Hence first term is 1. Sum of first 2006 terms. = 1+2+3+--- +2005 + 2006 2006 x:2007/2 = 2013021 3. Here is an addition problem with eight digits a, a2 ag a4 by by by by + C2 Oy ce dy dg ds dy +a ee e fi fh fs fa 3.4 6 3 4 5 3 6 What is the sum if each of 6 blocks of 4-digits (aragaga4), (bibababs), (creaesca), (didadsda), (erenesex), 62 Problems and Solutions (fifefafs) ere replaced by 2007 complement, given that each of the above 6 blocks of 4 digit numbers are < 2007. e.g. 2007 complement of any number say 1007 is 2007 ~1007 1000 Solution: Let (aaga3a4) = P (bibabsbs) = Q (creacaca) = R (didadsd4) = S (erezesea) = T (Affsf) = V Given: * 10000 P+Q +10000 R+ 5 +10000 T+V 34634536 Asked to find: 10000(2007 ~ P) + (2007 - Q) + 10000(2007 ~ R) + (2007 — 8) + 10000(2007 ~ T) + (2007 ~ V) "= 10000(2007) x 3+ 3 x 2007 += [000P + Q + 10000R + $ + 100007 + V] = 10000 x 6021 + 6021 ~ 34034536 = 25581485. Alternate Solution: ; Let 2007 complement of (ayaxq3a4) be (AyApAgAy) of (bibabsbs) be (BiB2BsBs), --- and of (fifafsfa) -be (ARB). ayazaaag bibadsba + ArAgAsAa By B2BgB, = 2007 2007 tere2cscy dydad3d4 + CiC2C3C4 Di DgD3D4 = 2007 2007 terereses fifofafat+ EiEn Es, F\FaFsFy = 2007 2007 3463 4536 + R = 6021 6021 Probleme and Solutione 83 . Ro = 60216021 34034536 25581486. 4, Evaluate the following, where addition and subtraction alternate. 2007 2007 2007 2007 [2007 blocks of 2007's] ~ 2007 2007 2007 [2006 blocks of 2007's) + 2006 2006 2006 --- 2006 2006 [2006 blocks of 2006's] ~ 2006° 2006 -+- 2006 +,2006 [2005 blocks of 2006's] + 0999 -+» 0999 0999 [0999 blocks of 0999’s} = 0999 «+ 0999 0999. [0998 blocks of 0998's} + 0002 6002 [2 blocks of 0002] ~ 0002 [1 blocks of 0002] + 0001 [1 blocks of 0001] How many digits sre there in the answer (evaluated result)? Solution: In general LK blocks of K's] f) —) [x - [kK] [(K = 1) blocks of K's] v= [kK] wao wero aero zero where ‘K” is of length ‘FOUR’ with each zero representing the length of K ie. 4 . Given expression reduces to 2007 2006 2005 - - 000300020001 ‘The number of digits in the result = 4444+44-++44 (2007 times) 84 Probleme and Solutions 5. a,b and c are 3 single digit numbers, Consider the two digit numbers ab,bc and ca. If the two digit number ab leaves a remainder 5 on dividing by 7 and the two digit number be Jeaves a remainder 2 on dividing by 7, what is the remainder if the tio digit number ca is divided by 7? Solution: ab leaves remainder 5 when divided by 7. Let the quotient be 2, a natural number. ab =a +b= Te +5 => Wa+ = ld +7+3 or 2a+%—-a=7(2c+1)+3 Qa) bc leaves remainder 2 when divided by 7. Let the quotient be y, 8 natural number. b= 10b+e=7yt+2 => 1006+ 10c = 70y + 14+6 or 98b-+ 2b+ 10c = 70y+144+6 (2) (2) — (1) + 98b-21a + (10e-+a) = 7{10y+2—22-1}+3 7146 — 3a} + (100+ a) = 7{10y — 22-41} +3 Problems and Solutions 85 => (10c-+ 4) leaves remainder 3 when divided by 7 (or) G& leaves remainder 3 when divided by 7. 6, Fifteen concurrent lines are drawn passing through the point O and fifteen triangles are drawn with the common vertex O, 50 that no vertex at the base is common to two triangles find the sum of the base angles of all these 15 triangles (Figures for 3 lines and 5 lines are shown, 4 4 be 4 b 4 Fig Solution: Let the angles formed at vertex O of each of 15 triangles be 21,22,73,04,"-+ 713,214,715. By vertically opposite angles, we get another set of angles 24, 02,03) ay" y 213,414,215 around O. 2. Their sum = 3° = 180°. -. Sum of base angle of 15 triangles = 15 x 180° — 180° = 14 x 180° = 2520°. x . ABC isa triangle. BD is the bisector of ZABC and CD is the bisector of ZACX. DB is extended to Y. BE is the bisector of ZYBC and DE is the bisector of ZBDC. 86 Problems and Solutions 2A= 60°, ZABC = 80°. Calculate the angles of ABDE and ABDC. Refer the given figure. In AABC, 2A+ ZB +, ZC = 180° by angle sum property. ZC = 180° — 60° — 80° 7 A By linear pair, ZAGB + LAX = 180° 40° + ZAOX = 180° - ¢ 1. ZACK = 140? But: OD bisects ZACX. -. ZACD = LDOX = 70° BD bisects ZB = 80° ZBCD = ZBCA+ ZACD = 110°, Now BDC = 180° — 40° - 110° = 30° , The angles of ABDC are 40°,90°, and 110° By linear pair, ZDBO + ZCBY = 180° . 40° + ZCBY = 180° : or LOBY = 140° - But BE bisects ZCBY = 140. * : Problems and Solutions 87 -. ZBBY = ZEBC = 70° ye > ao" e © DE bisects ZBDC = 30° iP J :. ZBDE = 18°. LDBE = ZDBC+ZCBE = 110°. Now ZBED = 180° — 110° ~ 15° = 55°. .. The angles of ABDE are 110°,15° and 55° 8. In the addition problem given below different letters stand for different digits. What docs cach letter stand for and hence find the number represented by MATHI ? Solution: Representing the addition process algebraically 1000 A + 100 M + 10 T + I + 10 M + 10 T + T + 10 T + 1 eee 1000 M + 100 A + 10 T + H 88 Problems and Solutions => 900A + 207 + 47 = 800M +H => H is a multiple of 4 as 900A,207,4/,800M are all multiples of 4. : His a single digit. -. H can be 0 or 4 or 8: . Equation becomes 4 (225A +57 +I — 200M) = or = 225A4+5T+I-200M=Oorlor2 (1) As MATH > AMTI, we have M > A. Let M=A4L. Then equation (1) becomes 225A +5T +I —200(A +L) =0 or lor 2 = %A+5T+I-200L =0 or 1 or2 (2) L must be 1. Otherwise 254+57+1—200L will be negative and cannot equalise* (or) 1 (or) 2as A,T,I,L are single digits. 200 or <. M=A+1 and (2) becomes. 254+57T+I=2 291 or 202 Case I: 254+5T +I = 200 = I should be 0 or 5 where H=0 as 25A,5T,200 are multiples of 5. T cannot be 0 as I= 0 = H = 0 which is invalid because all digits are different, T=5 + %A+5T=195 (or) 5447 =39 A=6,T=9 (or) A=7, T=4 Problems and Solutions 69 A=6,M=7 A=7,M=8, =» T=9,F=5 (or) T=4,I=5 H=0 H=0 () (2) (2) @) Q) @) Gee? 59S 7 8 4 «6 ’ + 7 9 6 + 8 4 5 fea as MO 4 46 + 5 + 5 7 6 9 0 8 7 4 0 .. MATHI can be 76905 (or) 87405 in this case. Case II: 25A+57'+J = 201 es 201 is 1 more than a multiple of 5. => I should be 1 or 6 where H=4 T=135A+T=40 > A=T7, T=5 (or) A=8, T=0 because A,T are single digits. A=7, M=8, A=8,M=9, => T=5,l=1 (or) T=0,J=1 H=4 H=4 Q) @) @) @ @) TotrBu 2.58 1 8. 8 O+1 dT op 8 Be + 9% 0. 2 ie, ea (or) i BetT + 1 + 4 8 7 5 4 9 8 04 90 Problems and Solutions . MATHI can be 87541 (or) 98041 in this case so far. Note: It is not mentioned that TY is a 2-digit number. I=6 +5A+T=39 => A=7, T=4=H Invalid as T,H should be different. (or) A=6, T=9, M=7, 1=6=A Invalidas J, A should be different. Case IIT: 25A+T+J = 202 = Ishould be 2 or 7 where H=8 5 202 is 2 more than a multiple of 5. I=2>5A+T=40 + A=1,T=5 (or) A=8, T=0 because A, T are single digits. A=7,M =8, A=8,M=9, > T=5I (or) T=0,r=2 H=8 H=8 > M =H =8 is invalid (or) A= H =8 is invalid, as each letter stand for different digit T=7>SA+T=39 > A=7,T=4 (or) A=6, T=9 because A, T' are single digits. A=7,M=8, A=6, M=7, = T=4,T=7, (or) T=9, H=8 H=8 = A=I=7 end M =H = 8 is invalid (or) M=I=7 is invalid as each letter stands for different digit. .", There are no solutions in this case. In general, MATHT can be 76905, 87405, 87541 (or) 98041 (any of the four 5-digit numbers) Problems and Solutions 91 9, Find the sum of (2+ 2y — 2007z) -+ (x + 3y — 20062) + (x + 4y — 2005z) +... +(x + 2008y — z) and show that this sum is divisible by 2007 for all integral values of z,y and z. Solution: Sum becomes 2007a + (2+ 3+4+...+2008)y— (142434... +2007)z See on 2007 +[1+2+34+4+... +2007 + 2007 Jy tt —(1+24+34...+4+2007]z 20072 + [= 2008 +2007] y — 2007 z 2008 | " 2007{x + 1005y — 10042} which is divisible by 2007 for all integral x,y aud z. 10. A year on a new planet had 365 days and there are months with no other than 28, 30 or 31 days. Find how many months are there in a year in the planet. Solution: As 13 x 31 > 13 x 30 = 390 > 12 x 2841 x 30 = 366 > 365 the number of months cannot be 13 or more. Similarly 11x28 < 11x30 < 11x31 = 341 < 365, the number of months cannot be 11 or less. ., The only possible number of months should be 12. Now, (13 thmes) 28 + 28 + 28 + 28 + 28 + 28 + 28-+ 28 + 28 28+28+28+28 = 364 8434343+3434+3434342 -28 = 1 (0 times) 92 Problems and Solutions Adding, 31431-+4+31-+31431+31+31+31-431+30-+28 +28 = 365. Hence, the planet could have one month having 30 days, two months with 28 days each, nine months with 31 days each comprising 365 days in the year of 12 months. Note: Alternatively the year can heve 7 months with 31 days, 4 months with 30 days and one month with 28 days. 11. ABC is: an equilateral triangle of side length 22007 units and smaller equilateral triangles of side length 270% , 92005 9200492 ot 90 = 1 are constructed on all the = B three sides (ag shown in the diagram for first 4 triangles). Find the perimeter of the figure thus obtained. (Hint: 142444. A +2 sort 1) Solution: Problems and Solutions 93 AB = BC =CA = 22007 BC, = AC = CAg = BA = BpA = C,B = 26, ‘There are 6 sides in the boundary of length 2° marked with Tine segment as e—}-e. Similarly, the latched line segments ByA1, Czas AsCi, CaBa, BiC5, ByAg are of length 2795 and are 6 in number; likewise, 2° occurs 6 times, 2709 occurs 6 times till 2' occurs 6 times: But 2° = 1 occurs 6 +3 = 9times since the last 34's are not divided further. . Perimeter of thie given figure 6[L +2 +22 +... = 278) 43 6(2°7 1) +3 = 3(2708 _ 1), 4 tt ig 12, ABCD is a square and K and L are points on BC and DC. AM is perpendicular to LK. ZAKM = ZAKB. Find the measure of LAK. Solution: A ZAMK and ZAKM => ZMAK = ZKAB ZABK ZAKB L tt Cc i M q By ASA congruency, al QAKM = AAKB. +, AB = AM. But AD =" Q 1K yo OB _AB as ABCD isa square. AD = AM, AL is common, 4 ZADL = ZAMD = 90° 24 Probleme and Solutions M 2 . a Ae ’ ». QADL = AAML . by RHS congruency », ZDAL=_ZLAM. Hence ZMAK + ZLAM = } x 90° = 45° (why?) <. ZLAK = 45° 13, I added all positive integers from 1 to P and obtained a total of 2007. By mistake, I added a number twice. Find the correct total and the number I added twice. Solution. Let Actual total =1+2+3+...4(P-1)+P and Incorrect total = 14+ 2+3+...¢¢+e¢4 (241) + ... +P = 2007 where z is added twice by mistake. And 0<2 1 and-can be at most P. As 142434...460461462 = 2x8 = 31% 63 = 1953 < 2007 and L4+2434-+6461462463 = axe 63 x 32 = 2016 > 2007, & Probleme and Solutione 95 the value of P must be 62. And correct total = 1953. And the repeated number = 2007 — 1953 = 54 < 62. 14, Evaluate: 1 1,2 1.2 8 2 2 99 i+ (5+5)+(G+342) ++ +(e wt tie) Solution: 142, 142+3 14+2+3+4 14+24+34-:-+99 Bical everge einer Broce pee errr Fem 1,1 /2x3 1. 3x41. 4x *oe ay tis TT 1, 99x 100 100 2 i aed 99 = 5+ tied tet = FU+2+3+. « +99) a. 1, 99x10 ne) 2 = 275. 15. A sequence has first term 2007, after which every term is the sum of squares of the digits of the proceeding term. Thus the first 3 terms are 2007, 63(2? +0?+07+74), 34(57+37). Find the sum to 2007 terms, Solution: Continuing further, wo got 37+ 47 = 25, 2? +57 = 29, Dp 9? ms BS, 82+ 57 = BY, 87407 = 145, 174-4745? = 145, 96 Problems and Solutions Poeesr aan, 42% = 20, P+ = 4, 4 = 16, 22462, = 37, 32477 = 58, 57+8? = 89. And the cycle repeats as : Stace 37 145 he 42 4e_ 20 a“ Hence the first 6 terms are: 2007, 53,34, 25, 29, 85. ‘And the terms there after are 89, 145, 42, 37, 58, 89, 145, 42, 20, 4, 16, 37, 58, 89,... (8 terms) having a cycle of 8 terms. .', Sum of the first 2007 terms will be: (2007 + 53+ 34+ 25 + 29 + 85) + 250 [89 + 145 + 42 + 2044+ 16+ 37+ 58] +89 = 2233 + 102750 + 89 = 105072. Note: 6 terms + 250 (8 terms) + 1 term is equal to 2007, the number of terms of the given sequence. KAPREKAR CONTEST Sub-juuion Level Serocuing Test-2008 1, In the adjoining figure f) and lz are parallel lines. ¢ is a transversal which cuts ;, 2 at A,B respectively. The angles at A,B (refer figure) are trivected. The measures of the angles ACB and ADB are respectively (A) 60°, 120° (B) 120°, 60° (Q) e-y, c+y (D) Ae~y), Ac+y) Solution: ef 4NAB + ZMBA = 180° > (180-32) + (180-3y) = 180 => 3a+y) = 180 > aby = 60° qQ) ZCAB + ZCBA = Ux + y) =2 x 60° = 120° (2) +, ZACB = 180° — 120° = 60° from AACB (3) Similarly ZDAB + ZDBA=2+y= 60° -. ZADB = 180° ~ 60° = 120° (from AADB) (4) ©. Answer (A). 97 98 Problems and Solutions 2. The number of digits when 200* is written in the decimal form ja (A) 2008 (B) 1004 (C) 74 (D).19 Solution 2. 2008 = 28x 1008 = 256 x.100 00 00 00 00 00 00.00 = 256 00 00-00 00 00 00 00 00 So the number digits of 200 written in the decimal form is 19. Answer (D). ABC ig an isosceles triangle with AB = AC = 2008 cm. ADC’ is drawn 08 on equilateral triangle on AC outside AABE. AD is parallel to BC. The bisector of D meets AB in E, say. Then BE 4s equal to (A) 1004em —(B) 2008em == (C)O-—(D) B02cm Solution: A 2008 D A ADC is equilateral; AD||BC. ©. ZACB = ZCAD (alt int Za) = 60° = ZABC, + A= oo 4ABC is also equilateral. So the igure ABCD ia a rhombus. A The diagonal BD bisects 2D and 2B of the rhombus. 1.0, ‘The bisector of 2D meets AB at B or B coincides with E. BE=0. Answer (C). Problems and Solutions 99 4, The units digit of the number 2° is (ayé (B)2 (c)4 (D) 8 Solution: 24 = 2, 2? = 4, 23= 8, 24 = 16 = 6 in units place 2° = 2 i.e,, 2 in units place. Lepr ae eae powers of 2 form a cyclic numbers 2,4,8,6, 2,4,8,6- if the power of 2 is a (multiple of 4) +1, the unit digit is 2; ite is e (multiple of 4) +.2, the unit digit is four and for the power a (multiple of 4) + 3, it is 8 and a multiple of 4 it is 6. Clearly in 27°", 22008 is multiple of 4. . 22°" ends in 6. ie., the unit digit of 2°" is 6. [In fact, 2?” always ends in 6 ifn>y Answer (A). 5. If a,b are natural numbers such that a +b = 2008, then (-1)? + (-1)° is (A)1 (B) -1 (C)2 (D) 2or ~2 Solution: a+b = 2008, a,b are ilatural numbers. So, both @ and 6 are even or both @ and 6 are odd; if both a and b are even (—1)" + (-1)? = 141 = 2; if both are odd (-1)°+(-1)'=-1 ~1 Answer (D). 6, If n = 10" — 1, the number of digits in n° is (A) 30 (B) 28 (C) 32 (D) 27 Solution: (1019 — 1)5 = 10% ~ 3 x 107 4.3 x 1010 — 10% — 1 = 99---9 3 nines 3'x 10° will be a 21 digited numbor and 3x 10! will be a 11 digited number. 100 Problems and Solutions Hence even after simplification the number of digits af. (10% — 1)? will be same os that of 10% — 1. ie, 30. Answer (A). 7. In the adjoining figure ABC, DEF are equilateral triangles. AB = 8&cm und DE-= 3cm, Then the possible value of AB4BD+OF io (A)6.9em (B)71em ()5.2em_=—(D) 8.3em Solution: “ (AE+ED)+BD > 8m (BD+DF)+FC > &m (OF+FE)+AE > &m => %{AE+BD+CF)+9>% + AE+BD+OF > ¥=15 ‘This shows that 8.3em is the only possible value amongst those given. Answer (D). 8. Anu, Babu and Chitra have BY balls altogether. Babu gives 7 bolls to Chitre’and Chitra gives 5 balls to Anu ond Anu gives 4 balls to Babu. If all the three have finally equal number of balls, then the number of balls Anu had at the start is (A) 17 (B) 20 (C) 16 (D) 19 Probleme and Solutions 101 Solution: Let a,b,c be the number of balls, Anu, Babu and Chitra had initially, respectively, Then a+b+ce=51 Balu 4 Anu’ 7 Chitra After the transfers are effected, finally Anu hes a-4+5 =a+1 balls, Babu has 6-~7+4 = b—-3 balls, and Chithra has e-~5+7=c+2 balls. But it is given that'all have equal number of balls natinb-S=e42= Fa17, “ @=16, 6=20, ¢=15 So Anu had 16 balls at the start. + Answer (C). 9, ‘The number-of ordered triples (a,b,c) 1 < a,b,c <9 such that ac=.b?~1 is (A)9 (B)7 (C) 14 (D) 18 Solution: ac = b?-1 => axc = (b~1)(b+1) with this property, we find that any 3 consecutive numbers, satisfy the condition; the ordered triples are (1,2,3), (3,2,1), (2,3,4), (4,3,2), (3,4,5), (5,4,3),, (4,5,6), (6,5,4),, (5,6,7), (7,6,5), (6,7,8), (8,7,6), (7,8,9), (98,7); Again we have (1,3,8), (8,3,1), (3,5,8) and (8,5,3). ” Thus there are 18 ordered triples satisfying the given conditions. Answer (D). 102 Problems and Solutions 10. Let.‘m’ be the number of perfect squares in 1,2,3,.. 1000000. Let ‘n’ be the number of perfect cubes. in 1,2,3, 1000000. Then the value of 2 is (A) 0.01 (B) 0.1 (C) 10 (D) 100 Solution: The number of perfect squares in 1,2,3;---10° is VI08 = 107 ie.,:m = 10%. ‘The number of perfect cubes in 1,2,3,-+-108 = Y109 = 10? ie., n = 10” 2 7 =01 al Blt als Answer (B). 11. The measure of one of the angles of a right triangle is five times that of a second angle. Then the possibility of the second largest angle is (A) 72°. (B) 75° (C) 72° Or 75° (D) none of these Solution: If right angle is 5 times that of another angle, then that angle is % = 18°. So the third angle is 90° — 18° = 72°, Tf one of the acute ¢s of the rt ZdA is 5 times that of the other, then 2+5x = 90° => 62 = 90°, = 15° so the bigger of the acute Zs is 90-15 = 75° So the possibility of the second largest angle is 72° of 75°. Answer (C). 12, The ‘difference of the squares of two consecutive natural numbers is 2008. (A) Only one such pair exists (B) Infinitely many such pairs exist (C), No such pair exists —- (D) Exactly two such pairs exist Problems and Solutions 103 Solution: Squares of two consecutive natural numbers are of different parity i.e. “one of them should be even; the other should be odd, (why?) The difference of these squares cannot be an even number; i.e. cannot be 2008. No such pair exists. Angwer (C). 13. The value of a commodity is increased by 2% first and aguin increased by «%. The total increase is (A) 22% — (B) 42% (©) (20+) % (2) 2% Solution: The final value of the.commodity after the two successive increases is 2042) x (9042) units where the initial value is taken as 1 unit. -:. The increase percent is (200 + x)? _ (200 +z)? — 1007] [ rag? 1) * 100 = 100 (200+ 2)x 100 a? [ae+ Fa] Answer (C). 14, 2 is 1% of 3$ and i is 24" of 2}. ‘Then (A) 2z=y (B)y 248 = plan >n=4, Answer (B). Problems and Solutions 105 18. A rectangular sheet of paper is folded so-that the corners A.B goto A’, BY usin the figure. Then ZZXY is (A) an acute angle (B) an obtuse angle (C) 8 right angle (D) 8 vatiable angle depending on the point X Solution: In the process of folding the sheet as given, AAXZ and AA'XZ are congruent; so also ABXY and AB’XY are congruent. LEXY = LBXY+Z2A'X2Z a 5leexe +2A'X3] = 5 % 180° = 90° Answer (0). 19. The tens place of two three digits numbers is 8 and both the numbers are divisible by 4. Then the difference between the biggest and the smallest such numbers is (A) 888 (B) 808 (C) 708 (D) 788 Solution: The smallest three digit number with 8 in the tens place and divisible by 4 is 180 and the biggest such number is 988. So the differences is 988 — 180 = 808 Answer (B). 106 Problems and Solutions 20. On a line segment AB = 2008 cm a square and a regular hexagon are frawn as shown in the diagram. The distance between theircentres P,Q, in cm is (A) 1004V3(B) 1004(/3+1) (C) 2008/3 (D) 1004(V3 - 1) Solution: 7 AB = 2008. The side AB- of the regular hexagon =2008em and AABQ is éqfilateral “ QR= 2 x 2008em = V3 x 1004cm. RP =} the side of the square = 2008 = 1004cm. So PQ = 1004(V3 + 1)cm. Answer (B). KAPREKAR CONTEST Sub-junion Lovel Final Test-2008 1, A Predator beast weighs 2008kg at the beginning of a year. Daring the first month of the year its weight is increased by 3} % and in the second month-decreased by 25% and in the _ Sind oa ince by 5%, in the fourth month decreased by 334%, in the fifth month increased’ by 163 % and in the sixth month decreased by 213%. This increase and decrease of the weight continued in the 1 next 6 months in the same order with the same percentage. Find the weight of the predator at the end of the year. Solution: Weight of the predator at the end of the year is 2 133} 150 663 1162784 0s | 100 * 100 “ 00 * 00 * 100 27 550]? = 2008 | = 3° al 2008 x 121 5 = Sa = 87 ks 2.¢n’ is a natural number greater than 1: Prove that (n®-+n7+1) is always composite Solution: n is‘a natural number greater than 1. Now, in’+1 = nbtn’+n>—(no—-1) a(n? +n $1) —(n—I)(n + 1)(n? + +1) (PP +n +1)(n8 — (n° +1)(n-1)) As ‘n' is a natural number n+nt1>1 (1) 107 108 Problems and Solutions We have n3 >nn?-1>n-1 2. (n3 + 1)(n3 1) > (+ Y(n— 1) or n®—-1>(n?+1)(n-1) or n&-(n3 +1)(n—-1) >1 (2) >, 18 +n7 +1 is the product of two numbers A=nri4n+1 and B=n5—(n3+1)(n-1), both being greater than 1, where ‘n’ is a natural number. -. (n° +n7+1) is composite. 3. ABCD isa trapezium, euch that ABI|CD. AB = 10 cm; BC = Sem; CD = DA = Som, Find the height of the trapezium and also find AC and BD. Solution: Draw CE parallel to DA to meet AB at BE. Clearly AECD is a parallelogram. Given: AB=10 cm, BC =8cm CD=DA=5em. Now, AE = 65cm and CH'=5 mend EB=6 cm Thus, ABCD is a thombus. Since AECD Js a rhombus, AC 1 DE. Let © be the point of intersection of AC, DE. Clesrly EBCD is a parellelogram, => ED=8cn Probleme and Solutions 109 DO =OF =4cm. In triangle COE, applying Pythagoras Theorem, we get OC =3cm. 1.40 =0C =3cm, In triangle AGB, AB? = AC? + CB?. So, ZACB = 90°. Area of the trapeziiam a Axes of AADC + Area of AACB = }A0-D0+5-AC-BC 1 1. 2 = ($x6x 445 «0%8) on = 36m? Area of the trapezium = }(AB + DC) x h, where h ‘is the height required. 36x 2 6x2 24 “BD N 2 2 1042) +(24) =svivem 5 5 4, Three regular-polygons have one vertex in commion and just fill the whole spacé at that vertex. If the number of sides of the polygons are a, b and c, prove that + }$+2=4. Solution: The regular polygon of a sides, & sides, :c sides cover the space at O as shown in the figure. 110 a Problems and Solutions ‘The sum of the interior angles of a polygon of n sides = (2n — 4)90° . Tt it is regular then all the Interior angles are equal. Each angle = (n=4)90" | The interior angles of the regular polygons are respectively Ga=Hoge, Bh-oq> and PeAgge, ‘The total angle at ‘O’ is 360°. _ A Soge 4 @- Pe Donr a Pe~ Mage = 4x 90 ¢ +, 22-4, 2-4 2-4 ie, + - bad . Find the number of all 5-digit numbers such that their squares have 9 digits. What is the largest of these numbers? {observe: =9; 4? = 16;-.-317 = 961; 327 = 1024] Solution: Observe that the number of digits of a square of an ‘n’ digit number is either (2n~1) or 2n. The squares of the one digit numbers 1,2,3 give the one digit numbers 1,4 and 9 respectively, So, the biggest one digit number whose square is @ one digit number is 3. The square of 4 is 16 which is a two digit number. Problems and Solutions 411 The biggest two digit number whosc square is a three digit number lies between 30 and 40. 30?=900 and 40? = 1600 Now, 317 = 961, 32? = 1024. ‘The biggest three digit number whose square is a five digit number lies between 310 and 320. Now 315? = 99225, 316? = 99856, 317? ='100489 (6 digit number). ‘The biggest 3 digit number whose square has 5 digits is 316. The biggest 4 digit number whose square has 7 digits lies between 3160 and 3170. A little calculation gives 3162? = 9998244, 3163* = 10004569. <. The biggest 4'digit number whose square has 7 digits is 3162, Finally the biggest 5 digit number whose square has 9" digits lies between 31620 and 31630. A quick verification shows that 316227 = 999980884 31623? = 1000044129 ‘Thus the biggest five digit number whose square has 9 digits is 31622. ‘The smallest § digit square number is 100000000 = (1000)? So the number of 5 digit numbers whose squares have 9 digits is 31622 — 9999 = 21623. 6.A certain 8 digit number is divided by 11 and the remainder is tr, 0 Gaenli t aye t Teme mon vl i-m ? nant atit tem” (m=n)n4l)+(n—Dim+n) I-m _ 4 > (mtnjint)) +m mn+im~n?.—In+(nm+n?~Im~In] l—m _ 4 (m+n)(n+1) lam 2mn—n t-m > intayntl) tem eicm_ 2n(l —_m) = Tem (mtnjn+h 4 1 2n t-m|[ e-em 7° < Bither l= m or a — gape =O «. Either 1=m or (m+n)(n +) —2n(i+m) =0 ob=morml—mn-in+n? =0 <.L= mor (n—m)(n=1) =0 le, l=m,n=morn=l 8. A natural number is said to be a “super-star-number” if the number is less than 10 times the product of its digits. Find how many super-star-numbers are there from 10 to 200. Problems and Solutions 115 Solution: Let us consider the 2-digit numbers first, of the form (10A +B) where A is a single non-zero digit, B is a single digit. The super-star two digit numbers are given by WA+B<10AB + 10AB—10A—B>0 => (10A-1)(B-1)>1 => A can be any single non-zero digit and B ‘is a single digit >2. Number of super-star 2 digit numbers = number of possibilities of A x number of possibilities of B =9x8=72 Consider the 3-digit numbers of the form 1AB given by (100 + 104+ B) where A is a single digit and so also is ‘B. ‘The super-star 3 digit numbers are given by 100+10A+B<10-1-AxB 10AB - 104 ~ B > 100 => (10A—1)(B~1) > 101 ‘The possible combinations of A, B are as follows: (3) A=4, B24 (4) A=5, B24 (8) A>6, B23 Number of euper-star 3 digit numbers from 100 to 199 = 34546464 (4% 7) =48 4116 Problems and Solutions Note that 200 is not a superstar number as 200 4 10x2x0x0. -, Number of Superstar numbers from 10 to 200 = 72+ 48 = 120 9, ABCD is a convex pentagon with 2A= 2B = ZD = 9°, ZC = ZE. Sides AB, BC,CD,DE and EBA are extended to K,L,M,N and O respectively. ‘The exterior angles ZOAB, ZKBC, ZLCD, ZMDE and ZNEA are bisected and the bisectors are produced in either. direction to form the pentagon PQAST. Find the angles of this pentagon. Solution: . The dotted lines represent the bisectors. LOBK = 90 => ZCBQ = 45° ZDCB = ZDEA = 135° (2 LDCB = ZDEA = Se" 270° 135° ) ZDOL = 180° — 135° = 45° 45° 1? DOR = LLOR = ZBCQ = > = 2; ZPQR = 180° - ( + a) = ua” Problems and Solutions 117 ZLUPQ = 180° ~ (ZPAB + £PBA) = 180° — (45° + 45°) = 90° ZRST = 180° — (ZSDE+ ZDES) = 180° — (15° +225") = ua} ZSRQ = 180? —(ZRDC + ZROD) = 180° ~ (0 +205") = 112 LSTP = 180° - (ZTEA+ ZTAB) = 180° - (25) +45") =i 10, Among all the rectangles of area\2008 sq. units, what is the smallest possible value of the suntf the lengths of the diagonals? What is the ratio of the product of the diagonals and the area of such a rectangle? Compare this ratio with the ratio of the product of dingonels of the square to that of the area of the square. Solution: Area of the rectangle = 2008sq. units Let the sides of the rectangle be 7 and b. => Ub == 2008 @) we have (J—)? > 0. (+: square of a real number is always non-negative) P+e> 2b VE+8 2 vV2-Vib So. the least value of ViT+07 which is the length of the diagonal is Vivib = V2-V2008 units. 4/251 units Sum of the lengths of the diagonals is 8V361 units. Product of the diagonals is 2b = 2 x 2008 = 4016 aq. units. 118 Problems and Solutions Ratio of the product of diagonals to the area 4016 : 2008 Qt Again in any square the diagonal is 2a units where the side of the square is a units. Product of diagonals:area = 2a? : a’ rs Since in the given problem (the product of the diagonals): (area) is 2:1, the rectangle is a square. (iv.,) of all the rectangles of fixed area the length of the diagonal is least when the rectangle is 8 square. ll. In the adjoining figure, c® AB = 200 units is a diameter of the circle. Points A,B are given the numbers 1,1. The = 4 two semicircles are bisected at C,D. Numbers 2,2 are given to C,D (see figure). Each of the quarter circles is bisected and given the number 3 (see figure). Now, each arc is bisected and the point is given the number awhich is the sum the two numbers at the end points of the respective arc (see figure) This process is continued till the sum of all numbers on the circle is atleast numerically equal to the product of four times the area of the circle and one third the radius. Find the least number of points plotted on the circle. (Take m= 3.14) Solutions. The first three stages of operation are shown above. Sum of the numbers in the first operation =6 = 2x3 Sum of the numbers in the second: operation = 18 =,.2 x 3? Sum of the numbers in the third operation = 54=2 x 3° Sum of the numbers in the nth bisection = 2x 3°, (a Problems and Solutions 119 ¢ < ‘> co): E3 2 34 > de x 100? x 5 % 100 2x 3.14 x 100? x 100 "> = 3" > 3 => 3" > 6280000 Now, 34 = 81 < 100 + 3% < 10000 35 = 243 < 628 33 < 6280000 310 = 243 x 243 = 59049 34 = 4782969 < 6280000 335 = 14348907 < 6280000 ‘Thus the least number of points plotted on the circle should be 15. 12. If a,b,c are reals and abe=1 and a+b+c=h4}41, prove that one of these real numbers is equal to 1. Solution: a-+b+ c= stiigte Since abc = 1 we have a+b+c=ab+bce+ oa sa+b+co-ab—be—ca=0 120 Problems and Solutions => abe — be ~ ca—abt+at+b+ce—abe=0 = abe — bc — ca—ab+at+b+ce-1=0 => (abe — be) — ca+c—ab+b+a~1=0 = be(a — 1) - e(a~ 1) ~ Ka~1)+(a-1) =0 => (a-1[be—c-b+1) =0 > (a-1)(b-1)(e-1) =0 => atleast one of a,b,c is 1. Note : Clearly when a= 6=c=1, abe=1 and atb+e= = 34444. Also if any two of them is 1, then the third is also 1. Suppose a= 1, then b=}, KAPREKAR CONTEST Sub-juuion Level Sereoning Test-2009 1. The number of three digit numbers whose digits are even is (A) 64 (B) 75 (C) 100 (D) 125 Solution:The leading digit has four choices ie. 2,4,6 and 8. The other two digits have 2,4,6,8 and zero. Thus for the hundredth place there are four choices and for the tens and units places there are five choices each. Hence the number of such numbers is 4 x § x 5 = 100. Answer (C). 2. ABCD isasquare and BCE is an equilateral triangle constructed externally. The : measure of ZAED is a t (A) 30° (B) 20° (C) 35° (D) 15° Solution: ZABE = ZABC+ZCBE = 90+60= 150° ‘: - BA = BE and hence ZBAE : = LBEA=8=15° e c and similarly ZOBD = 15° and therefore ZAED = 60°~30° = 30° Answer (A). 3. ‘The number 111 --- 1111 is @ 2009 digit number. It is multiplied by 2009. The third digit from the left of the product is (aya @)2 ()3 ()9 122 Problems and Solutions Solution: The actual multiplication of the 2009 digit number 111 --+ 111 is shown below. 111L --- 111 x 2009 9999 --- 9999 00000 --- 000 000000 --- 00 2222222 --- 2 2232222 --- 21999 ‘The carry over 1 by adding 9+2 starts from the 5** place from the right. The carry over 1 from the 4‘ place from the left is added to the digit 2, and there are no more carry overs. The product is 2232222 21999 Ww all these digits are 2 So the third digit from the left is 3. Answer (C). 4, The average of ten different positive integers is 10. The smallest. is 5. The biggest of these numbers can be (A) 55 (B) 49 (0) 25 (D) 19 Solution: The sum of all the 10 positive integers 10x 10 = 100 The smallest is 5. In order to get the biggest number in the sequence, we should take the first nine different numbers as 5,6,7,8,9,10,11,12 and 13. The sum of there 9 numbers is 81. . The biggest number should be 100-81=19. Answer (D). Probleme and Solutions 123 5. In the figure AB = AC = CD and ZBAC = 32°, then ZBAD is (A) 37° (B) 64° (C) 69° (D) 74° Solution: A Bi a D AB=AC; ZB = zc = 180-82 _ ZACD = 180° —74° = 106° In AACD,AC = CD «.ZCAD = ZCDA= =37 “.ZBAD = ZBAC + ZCAD =32+37 = 69° 74° 180-106 _ 74 Answer (C). 6. The remainder when the number (2x 3x 4 x 2007 x 2008 x 2009) — 2008 is divided by 2009 is (A) 0 (B) 2008 (1 (D) 2007 Solution: 2x3 x 4x 2007 x 2008 x 2009 — 2008 2x 3 x 4 x 2007 x 2008 x 2009 — 2009 -+1 20092 x 3 x 4 x 2007 x 2008 — 1] +1 2009 divides 2009 [2 x 3 x 4 x 2007 x 2008 — 1]. Hence when the given number is divided by 2009. the remainder is 1. Answer (C). 124 Problems and Solutions 7. In a certain month in the new academic year, Sundays are on even dates in that month. Which day occurs on three even dates of the following month (A) Thursday or Friday (B) Wednesday or Thursday (C) Tuesday or Wednesday (D) Monday or Sunday Solution: To have Sunday occurring on three even dates, Sunday should occur on 2) 9 [16} 23 [30 If this month has 30 days then the first of the following month is Monday and 2" is Tuesday and Tuesdays occur on [2] 9 [26] 23 [30 Thus Tuesdays occur on 3 even days. If the month on which 3 Sundays occur on even days have 31 days, the following month starts on Tuesday and 2"4 of this month is Wednesday and Wednesdays occur on 2] 9 [16] 23 [30 and hence either Tuesday or Wednesday occur on 3 even days 2, 16 and 30". Answer (C). (Note: From the question we note that the month mentioned cannot be January and no day can appear on 3 even days in the month of February ] 8. The digital sum of a number is the sum of the digits of the number. A three digit number divisible by 9 has the following property The digital sum of the quotient of this number divided by 9 is 9 less then the digital sum of the original number. The number of such three digit numbers is (A) 2 (B) 3 (C)4 (D)5 Problems and Solutions 425 Solution: Any 3 digit number divisible by 9 has the digital sum 9,18 and 27. 999 is the only number divisible by 9 with the digital sum 27; The quotient on dividing 999 by 9 is 111 and its digital sum is 3. The difference is 27-3 =24; 999 is not the number with the required property. If the digital sum of the number is 9, the digital sum of the quotient should be zero which is not possible. So we should consider the three digit numbers with digital sum 18 They are 990 981 [972] 963 954 945 936 927 918 909 891 882 873 864 855 846 837 828 819 792 783 774 765 756 747 738 |729 693 684 675 666 657 |648) 639 594 585 576 |567| 558 549 495 [486] 477 468 459 396 387 378 = 369 297 288 279 198 189 It can be easily see that numbers shown on the boxes alone have the required property and there are 5 such numbers. Answer (D). 9. In the adjoining addition each letter represents a AA digit (identical letters represent the same digit, +AA different letters represent different digits) The CAB value of A is (A) 6 (B)7 (c)8 (D) 9 Solution: In this addition A+ A should give a carry over 1 (and C =1)A+ A = 2A is an even number and A should be 5,6,7,8 or 9. The 10° place addition together with the carry 126 Problems and Solutions over from units place should be = x “ A=9 A+At1 ees [The carry over ‘one’ from tens place to hundreds place is 10 tens =100 = one hundred] Thus A=9,B=8 and C Verifying 99 +99 198 Answer (D). 10. A square and a triangle overlap as shown here. The shaded area is two thirds the area of the square and 40% of the area of the triangle. The ratio of the area of the square to the area of the triangle is (A) 5:3 (B) 3:5 (GC) 4:15 (D) 715, Solution: Let the area of the square be s and that of the triangle be t. Area of the shaded portion is 3° Also area of the shaded portion is 40% of t = 24. 23, ; Sto *, The ratio of the area of the square to that of the area of the triangle is as follows: 0 I L est no o I ents Answer (B). Problems and Solutions 127 1[2|* 11. The natural numbers are ala }4 filled as per the table shown SaleciilG where * represents a square not filled. We find in the vanenES middle squares the numbers 2, 9 | + |10 *,5,8,%, * [11] 12 13) 14} * ‘The number found in the 2009th middle square is Solution: TREES 3i*)]4 Observing the numbers in the middle maletlte squares, there is a pattern for numbers 71sls and the empty mark x. The mark x appears in 24, 5H, gt... squares. 9 | * [20 * | 11) 12 These positions are one less than a alule multiple of 3 i.e, 3k — 1. Now 2009 is 2010-1 = 3x670—1. Thus 2009" square 15] * | 16 in the middle contains the empty mark *. : : Note: Of the 2009 middle squares, 670 are marked by empty symbol *. ; So the remaining 2009 — 670 = 1339 squares have mumber sequence 2,5, 8, 11,-*- 2008t square is the 1339" square after deleting the empty squares. The number sequence 2,5,8,11+++ is a sequence of the numbers which are 1 less than a multiple of 3, The first number is 1 x 3—1, the second is 2x 3—1 the third 128 Problems and Solutions is 3x 3—1--- . So the 1339'* number in the sequence is 3 x 1339 — 1 = 4017 — 1 = 4018. i.e., The 2008 square has the number 4018 and 2010" square has the number 4021! Find the patterns of the position of * in 1% and 3° squares of each row. Also find a pattern for the numbers in these squares. Ay Ay Ay Aa Ay Me Ap dy Ay As Aus Ad LJ tt 1 Im a straight road distances in kilometers are marked from a tree, Two of the sign boards show 75 km, and = km. Which 1 position of the sign shows é km from the tree ? Solution: The distance between the two signs is 1 13. ~ 78 = 99k" Z 5 ‘The distance is divided into 13 equal intervals. So the length of each interval is = 55 +18= G5 km from the tree. It is required to find the letter corresponding to zm from the tree. Distance between ; km mark and a Jan is 1 1 2 1 1 fl e-ap=amt pkm As ; 1 6 ~ 18 ~ Te ~ gk™ AS gg km represents 1 interval 5 km represents 10 intervals, The end point of the 10¢4 interval Ato represents = km. 13. Observe the above diagram formed by unit squares. Not including the middle square hole, the number of unit squares Problems and Solutions 129 needed to build the tenth pattern is Solution: Observing the diagrams, the pattern for number of unit squares, deleting the square hole in the middle is 5x342x3-12, 6x442x4—-2%, 7x54+2x5—3? or 7x3-1?, 8x4-27, 9x5—3? So the number of unit squares (deleting the middle hole) required to construct the 10‘ figure is 16 x 12— 10” = 192 — 100 = 92 So the number of unit squares required is 92. 14. Three digit numbers are written with different digits. The sum of the biggest and the smallest of such numbers is Solution: The biggest 3 digit number, with different digits is 987. The smallest 3 digit number with different digits is 102. Their seem is 987 +102 = 1089. 15. A certain number n is divisible by 21, 28 and 49. The smallest possible value of n is Solution: ‘n’ is divisible by 21, 28 and 49. So n is divisible by the prime numbers 2,3, and 7. the greatest power of these numbers in the divisors of N are 2,1 and 2 respectively. So the least value of N is 2? x 3! x 7? = 588 130 Problems and Solutions 16. In the diagram ZAPB and ZDPC have a common vertex P. XPY is the common bisector of the angles ZAPB and ZDPC. If ZAPB = 150°, ZCPD = 40° then ZBPD is — Solution: ZCPB = 180°—[ZXPB+ZCPY] 180° — [75° + 20°] 85° (Why?) ZBPD = ZBPC+ZCPD =85° + 40° = 125° 17. There are 2009 equispaced points on a circle. The number of isosceles triangles with two of their vertices at these points and the third vertex at the centre of the circle is Solution: Since the vertex of . the isosceles triangles is the centre, we should find, the number ways of choosing the fe other two vertices, from among 2009 equispaced points. A Fixing one vertex say A, there are 2008 ways of choosing the second vertex among these 2009 points, This is true for each of the 2009 vertices. When we count from each vertex A, A2,--- we count each vertex pair twice as Problems and Solutions 131 Ay An, Av, AsAz, A7As ~~. Thus there are } x 2009 x 2008 = 2009 x 1004 = 2017036 triangles 18. Two circles of different radii touch externally. APQB is (or the line of diameters as shown 4, B in the diagram. AD,BC and LS DRC are tangents so that the area of the rectangle ABCD ww is 16cm”. The area of the p R c triangle PQR is ——cm?. Solution: AB r+r+R+R =2r+R)=CD PQ = (r+R) wits QR =R Units Area of the rectangle is Rx (r+ R)sq units = 2(r + R)R = 16 sqem 1 g(Rtr) xR AR+r) xR 4 Area of the triangle POR , 16 Peat 4 sq. cm. 19. Two of the altitudes of the scalene triangle ABC have lengths 4 and 12. If the length of the third altitude is also an integer, then its greatest value can be 132 Problems and Solutions Solution: Let h be the fe length of the attitude v from the 3° vertex B. 20 24 24 ee Mi ow alr ala O°} *t' 37a ws sh <6 h Integer wh =5 20. Rite reads 41 pages every day. She started reading a book containing 2010 pages. The number of pages she reads on the last day to finish the book is ——— Solution: The number of pages in the book is 2010. The number of pages read each day is 41. 2010 = 41 x 49+1 She finishes reading the book in 49 + 1 = 50 days and on the fiftieth day she reads just 1 page. KAPREKAR CONTEST Sub-junion Loved Final Test-2009 1, Find the number of distinct remainders when 2009 is divided by natural numbers. Note that the remainder r is such that 0 2009, the remainder is 2009. If 2009 is divided by integer 2009 to 1005 in decreasing order, the remainders are 0, 1,2,-+-1004. If we divide 2009 by numbers from 1004 --- down to 1, the remainders are definitely less than the divisors and hence the remainders are already a part of {0,1,2,--- ,1004}. Thus the distinct remainders less than 2009 are 0,1,2,--- ,1004. 2.Find the sum of all the positive integers less than 2009 and relatively prime to 2009. Solution: 2009 = 7 x7 x 41 The positive integers < 2009 and not relatively prime to 2009 are the multiples of 7 or 41. Multiples of 7(< 2009) = 7 x 1,--+7 x 287 Multiples of 41 < 2009 are 41 x 1,41 x 2,-+- 41 x 49, The numbers counted in both are AL x 7,41 x7 2,41 X 7X 3,9 AL X 7X7 .. Sum of the nos not relatively prime to 2009 and < 2009 TL + 2-4 ++ + 287) +41( +++ + 49) 41 x TL+2++0°+7) 7 x 287 x 144 + 41 x 49 x 25 — 41 x 7(7 x 4) 2009(144 + 25 — 4) = 2009 x 165 134 Problems and Solutions , Sum of the nos < 2009 and relatively prime to 2009 = 142+-+-+2009 — 2009 x 165 = 2009 x mis — 2009 x 165 = 2009(1005 — 165) = 2009 x 840 = 1687560 3.There are two neighboring townships A and B. In A there are 500 children. When there was spread of an epidemic, in township B, 40% more students were free from the attack, compared to A. If the number of students free from the disease is the same in both the townships, find the student population in township B. Solution: Student population is A = 500 Let the student population in B be x. Let the % of student not affected by the disease be y in town A. The % of students not affected by the disease in B is y +40. No.of students not affected by the disease in _ 500 xy ~~ 100 =5y, is an integer <. y is an integer. No. of students not affected by the disease in x x (y+ 40) Bae 100 a 500y _ (y +40) It that —— = is given that 100 500y 7 +40 isan integer and y is also an integer. ©. (y+40) divides 500y and y+40< 100. It is easy to see that y=10 > 2=100 and y=40 = «= 250 y=60 > x=300 Problems and Solutions 135 Student population is B may be 100 when 10% of people in A and 50% in B are not affected (or) student population in B may be 250 and 40% of students in A and 80% of students in B are unaffected. (or) 60% of A’s students population is unaffected and 100% of B’s student population of (300) is unaffected. In case 1, 50 students are unaffected and in case 2, 200 students are unaffected and in case 3, 300 students are unaffected. 4. if a,b,n are natural numbers and n # 1, arrange the following in ascending order of magnitude. na (n+1)b’ na (n—1)a (m—1)b nb Solution: Let na (n+1)d na a hie” 7b a. In all the four numbers 3 8a common factor ‘ hb ite n n+1 n n-1 ©. It is enough to write >, order. 1 oa n 1 = 1+-, n z 1 1 <5 and <5 1 1 -— <14+-<14+—. “4 ntl + a —T n n+l n < ome 136 Problems and Solutions 5.In the diagram given below ABEF, BCDE are squares of side a units and EG (the extension of EF) is b units. In the figure, the rectangles are successively dissected into pairs of squares and a rectangle. The first four stages of this process is shown in the figure. The dimensions of the rectangles in these stages are (a + b,a), (a,b — a), (6 — a,3a — 26) and (3a — 2b,5b — 7a) respectively. The ratio of the length to the breadth in each case are respectively. a+b a b-a 3a — 2b a’ b-a’ 3a — 2b’ 5b—Ta Find the ratio of the length to breath for 5 more stages and show that all the nine ratios are each equal to 2+° a (Note that 6? = 2a?). Solution: In the first four stages the length and breadth of these rectangles are clearly GD a+b GH_ a JL SB SF Oa IH, a m, , OL ~ JL—JO™~ (6—a)—2%(8a—25) ~ 5b—7a Observe the ratios a+b a b-a 3a @ b-a 30-2 ©= 5b —-Ta Numerator represents the length and the denominator represents the width of the rectangles. Problems and Solutions 137 a+b a a | ———__ = ____. From Weett op ta ba? a b-a From 5° we get =—5 3a — 2b _ 38a—26 b-a—2(3a—26) 5b- 7a" Continuing the process we get the length and breadth of the rectangles in the 5¢ to ot® stages as From -—— 5b-7a 17a—12b 29b—4la 99a — 70b 17a — 12b’ 29b— 41a’ 99a—70b’ 1696 — 239a ang 1696 = 2394 77a — 4080 b is the length of the diagonal of the square of side a. (ie) For the second rectangle ratio is Ba b+a a The third ratio a+b The fourth ratio is also = ——. a(b+a)_ a(b+a) b? = 20? © az from I (b—a)(3a + 2b) “Qa? = 452 (b—a)(3a +26) It is proved below: 138 Problems and Solutions 3a—2 _ (3a —2b)(5b-+ 7a) 5b— 7a 250? — 49a? ab — 108 + 21a? -3 @+ab_atbd a a (Using 1) 6.In the adjoining figure ABCD is a square of unit length. The side AB,BC,CD and DA are produced on both the directions so that the extended segments also have lengths 1 unit. Show that the point triples (L,B,I), (K,A,F), (J,C,@) and (E,D,H) are collinear and PQRS is a square. Solution: We have to prove that LI,JG,HE and FK pass through the vertices B,C,D and A respectively. In ALAB, AB = AL and ZLAB = 90° “ ZLBA = 45° QQ) Similarly ZOBI = 45° (2) {from right ACBI,CB = CI }. Also ZABC = 90° «. ZLBA + ZABC + ZCBI = 45° + 90° + 45° = 180° Problems and Solutions 139 L,B,I are collinear. Similarly J,C,G; H,D,E; and F,A, K ; are collinear. Consider ARAB ZRAB = ZRBA=45° —-, ZARB = 90° Similarly ZBSC = ZCPD = ZDQA=90° PQRS is a rectangle. Also RA= RB = $B =SC = PC =PD=QD=QA. «.PQ=QR=RS=SP. «. PQRS isa square. 7.a,b,¢ are distinct digits. Find all (a,b,c) such that the three digit numbers abe and cba are both divisible by 7. Solution: If abe is 3 digit number with a,b,c as the digits then abe = 100a + 10b + ¢ The number cba obtained by reversing the digits is cha = 100c + 106+ a Both are divisible by 7. ‘Their difference is divisible by 7 abc — cha = 99a — 99c is divisible by 7. But 99(a—c) is a multiple of 7 and 7 does not divide 99. .. 7/(a—). Since a,b,c are distinct digits a—c=7 or —7 Since a and c are leading digits in 3 digit numbers a,c #0. lga, c<9 a 9 c=2} a=8 c= ‘The numbers are form 962,861, or 269 168. 7 divides 962 or 209 if b=5. The numbers are 952,259. 7 divides 168 or 8b1 if b=6. The numbers are 861,168. ‘Thus there are 4 numbers. 168,861,259,952. 140 Problems and Solutions 8. 2,y,z2 are positive numbers such that 2y 22 2x Tay) Tae 14a x = Prove that z=y=2z Solution: Let a y > z is not possible =y=z. KAPREKAR CONTEST Sub-juuion Loved Screening Test-2010 1, Given a sequence of two digit numbers grouped in brackets as follows: (10), (11, 20), (12, 21, 30), (13, 23, 31, 40). --- (89, 98), (99). The digital sum of the numbers in the bracket having maximum numbers is (A) 9 (B) 10 (C) 9 or 10 (D) 18 Solution: Observing the number of numbers in the brackets and the digital sum of each number in it, it is found that there is one number with digital eum 1, two numbers with digital sum 2, 3 numbers with digital sum three, in the brackets 1,2,3,+-- etc. In the 9th and 10*® brackets there are 9 numbers each, the digital sums being 9 and 10 respectively. From the 11th bracket onwards the number of numbers decreases by one and inthe 18th bracket there is only one number, with digital sum 18, Therefore the maximum number of numbers are found in brackets 9 and 10, with digital sum 9 and 10 respectively. Answer : (C) 2. Using the digits 2 and 7, am and addition or subtraction operations only, the number 2010 is written. The maximum number of 7 that can be used, so that the total numbers used is a minimum is, (A) 284 (B) 286 (C) 288 (D) 290 ‘Solution: 7 x 287+1, gives TATHT + ATH ) 287 times: 2010 2010 i “Mt 142 Probleme and Solutions ‘The number of digits used being 287+ 1; but only the digits Tand 2 are to be used. 7X 2B6+4x2 THTATH JTF 24 24242 (b) 2e6 times 4 times. ~. 2010. W The total numbers used is 290. Again 2010 = 7x 288-3x2 THT+ TH T-2- 2-2 288 sevens 3 twos © ‘Total numbers uséd have is 288-+3=291>290 _ (c) In: (b), we-use 286 sevens end the total number of digits used is 290 which is the minimum. ‘Answer : (B). 3, In the adjoining ranigoli design each of the four sided figures is a rhombus and the distance between any two dots is 1 unit. The total area of the design is (A) 36v3 (B) 9Vv3 (C) 24v3 (D) 18v3 Solution: (4,B),(4,C),(B,D),(G,D) and (B,C) are the adjacent dots. But A and D are not adjacent. , Joining BC, we get triangle ABC Lic and DBC both of which are equilateral. Thus each rhombus \_\ LT gives rise to two equilateral triangles AN of side 1 cm. There are 18 ()-) such rhombuses, there by giving 36 / _/ \\ equilateral triangles. Problems and Solutions 143 S . In the addition problem shown, different ‘The total area of these 36 equilateral triangles is 36x gts xIxt sq. cm = 9V3 sq.cm. Answer : (B). letters represent different digits. If the carry over from adding the units digit is 2, then (A+Z) cannot be (A) 2 (B) 4 (C)7 (D) 9 Solution: I+ A ofthe tens place cannot be greater than 17 and together with the carry over 2 from units place, it can not be more than 19. Hence the carry over from 10’s place to hundreds place can be 1, and for same argument the carry over from 100’s place should be 1 and hence M = 1. If A+ =9,A+J+ corry over 2 from units place = 11 and T should.be 1. But M and T can not both be 1, so A+J cannot be 9. Answer : (D). . The percentage of natural numbers form 10 to 99 both inclusive which are the product of consecutive natural numbers is (A) 9§. ®) 7% (C) 10 (@)9 Solution: The required two digit natural numbers are (1) 3x4=12 (2)4x5=20 (8) 5x 6=30 (4)6x7=42 (5)7x8=56 (6) 8x9=72 (7) 9x 10 =90 (2x3 and 10 x 11 are not two digit numbers} Hence there are exactly 7 two digit numbers out of (99-9) = two digit numbers. ‘The percentage of these 7 numbers is gf x 100 = 2 = 7%, Answer : (B). 144 Probleme and Solutions 6. In the adjoining figure, ABCD is a square of side 4 units. Semicircles are drawn outside the squares with diameter 2 units as shown. The area of the shaded portion in square units is (A)8 (B) 16 (CG) 16—27 (D) 8-7 Solution: The diameter of the inscribed circle is equal to the x4 4 side of the square = units. Its area is sq. units = 4 sq. units. Area of the 4 small semi circles = 4x 3X2 =2n sq. units. ‘Therefore, Area of the shaded portion = Area of the square — Area of the inscribed circle + Area of the 4 semi circles = (16~ 49 +2n) sq. units = 16—2n sq, units Answer : (C). 7 nm=1t11+111+---+1111111111. The digital sum of n is (A) 39 (B) 38 (C) 87 (D) 36 Solution: n= 1 be lt 1211111111 1234567900 Problems and Solutions 145 The digitel eum of n = 14+24+3+44+54+64+74+9+0+0 7x8 = t9s37 Answer : (C). 8.1 A+B=C0;, B+C=D, D+A=E then A+B+C is (A) EB (B)D+EB (C)E-D (D):B-D+E Solution: A= B-D B= D-C c=c A+B+C = E Answer : (A). i 9. A three digit number ab7 = 0° +8°+7°. Then ais (A) 6 (B) 4 (C)7 (D) 8 Solution: The number ab7 = 100a+106+7 1000-4 100+7 = +847 = a2 +08 4343 or 100a+105 = 336+0°+0° " a can not be less than 3, otherwise 2, 100a+10b+7 < 297 and 297 < 343. for 4 ‘Taking a= 3, We got 307+ 106 = 370+ or 10 — 6? = 63 or 10b = 63 + 0% 10d is always less than 63 + b* (b= 9 = 00< 63+ 729) [verify] 146 Problems and Solutions So @ can not be 3. Let-us-take a to be 4. “Then 400-4 105+7 = 644074343 - 1b = B Either 6 “= 0 or = 10 e 10 not possible. b= 0. Therefore b = 0,|a==-4| satisfy the relation ab7 = a +09 4.79 or 407 = 4? + 0° +75 For other values of a = 5;6,7,8 or, 9 we. cannot find the corresponding value }. Hence there exists only one three digit number with unit digit 7 satisfying thé given relation having a=4 and the answer is B. Answer : (B). 10. Among the participants of this screening ‘test, some of them together got the correct answers for all the problems, not all of them got more than 8 problems correct. The maximum and minimum number of problems solved by the three together are (A) 25,25 (B) 24, 26 (C) 25,27 ~~ (D) 25, 28 Solution: This screening test has 25 problems. Since three of them together got correct answer for all the problems, the minimum problems solved together by the three should be 25. (I may be 8, 25 and 25 6r 8,8,9 where, of the three participants no two of them solved the same’ problem ete. For maximum, let us consider that one of them solved:8 problems and each of the-others solved all‘the 25 problems, Thus the-maximum that could be solved.may be 58. Note: In the problem printed in the question paper ‘some information was omitted and the choice(25,58) was mistakenly printed as (25,28). Problems and Solutions 147 i. 2+$=%y where a,b are natural numbers. = 26 (2) a= 19, b= 13x14 (3) a= 14, = 13x14. Of these statements the correct statements are (A) (1) and (2) (B) (1) and (8) (C) (2) and (8) (D) (1)(2) and (3) : Toil tion: = += = — Solution: ata 3 (2) a=b=-26 gives ca : 6 “26 = 36 = 7g 18 8 correct statement or true statement (b) a= 13,b= 13x 14 Vlad oa _ at eh ab" ist Ixia Catan EEE «. It is false (c) a= 14,6=13x 14 aa 1 _ 41 4 4 ato~W* sx xi xi >is a true statement. So, (1) and (3) are the correct statements. Answer : (B). 12, a3 +08 =p: x pa, a® ~ b* = ps, where pi,p2,p3 are’ prime numbers, a> ,2<0,b<5. ‘The following statements are also given: (1) pi+peo+pa is a prime number. (2) p3—(pi+pa) is prime number. (3) i ~ p2 is a prime number. Then the valués of a and b are respectively. (A) G4) (B) (43) (©) (3,2) (D) (2,3) 148 Problems and Solutions Solution: a? +b? = (a+ 6)(a” — ab +8") = pi X Pa Q) a3 — b= (a — b)(a? + ab +6?) = ps (2). From (2), a—b=1 or a=b+1 and a*+-ab+ b? ='pg. Since a > bya > 2,5<5 and a=b+.1 give us the choices for @ and 6 as follows. an ene oe aahonea and b=1,a=2 gives w= 7 and 1 Xpyp = a+b =9=3x3 Pitpatp = 3+3+7=13 Pi~P2 = 0. So the third statement is not satisfied. b=20=3,py = 19 Pi = 5,p2=7( orp, =7,p2 =5) pytp2+Ps, = 31.is a prime number. Ps= (pi +P) = 19 (12) =7 is also a prime number. Bip. = 7-5 =2isa prime number For a=4, b=3, ps =a—? =55 is not a prime number. Fora=6,b=5, a8 -b8 = 69-53 =91--13x7 is nota prime. : Fora=5,b=4, a9 +68 ='59 443 = 189 = 9 x 21 is not a prime. : Problems and Solutions 149 ‘Therefore there exists exactly one pair (a,b) = (3,2) satisfying all the conditions. Answer : (C). 13. p is a prime number greater than 3. When p* is divided by 12 the remainder is (A) always an odd number greater than 2. (B) always 1. (C) Lor 11 (D) always an even number. Solution: _p is a prime number > 3 and hence is an odd number and a non multiple of 3. That is p is of the form 3 x-.even number +1=3 x 2n+1=6n+1(Why?) p? = (6n + 1)? = 36n? + 12n +1 = 12[8n? + 1] +2 p? leaves a remainder 1 when divided by 12. Another method: p must be in one of the forms 12n + 1,12n+5,12n+7,12n +11 p= (12n+1)? 122 42x 12n4+1 = 24n(6n+n) +1 ee multiple of 12 pr=(12+5)? = 12?n? +20 x 12n +25 = ln? +10x 12x nt+2441 multiple of 12 12m? +2 x 12n x 74+49 12?n? +2 x 12n x 744841 multiple of 12 p= (12n+7)* p= (12n411)? = 129m? +2x lan x 11 +121 = 12m? 42x 12n x 11412041 multiple of 12 Inall the above four cases, 7 is a multiple of 12 plus one, which means that all prime numbers greater than 3, when squared and divided by 12, leave a remainder ‘1. 150 Problems and Solutions 14. ‘Two regular polygons have the number-of sides in the ratio 3:2 and the interior‘dngles in the ratio 10:9 in that order. The number of sides of the polygons are-respectively (A)6and4 (B)Qand6° (C)12end8 (D) 15 and 10 Solution: ‘Let the number of sides of the polygons be 3n and 2n respectively. ‘ : ‘The measure of each interior angle of the polygons are 2xBn-4 right angles, and.?420=4 right angles and 6n—4 . in ML 2 in, 3 Hof . Sn = 12, a8. ‘The sides of the polygons should be 12 and 8. Answer : (C). . : / 15, @ is a real number such that a’ +4a—8 = 0. Then the value of a? +64a? ig (A) 128 (8) 164. (C) 256 (D) 180 Solution: a+4a-8 = (isthe same as —4(a — 2) is the same’ as af = (a)? = (-&(a—2)) = 16(a—2)? = 16{a? — 4a +4 a’. = 16a° — 64a? + 64a; This is tho same as a” +64a? = 16a" + 64a = 16(a + 4a) = 16 x 8 = 128 Answer : (A). Problems and Solutions 451 16. ABC is aright angled triangle with B = 90° BDEF isa F square. BE is perpendicular to AC. ‘The measure of 2DEC is -. B Solution: BDEF is a square. BE is a diagonal .. 2BED = 45° ZBEQ 90° [BE is perpentiicular to AC] ..ZDEC = ZBEC—ZBED = 90°— 45° = 45° i Answer ; 45° 17. p is prime number and p = a? ~ 1. The number of divisors of a+p is____.. Solution: p=a?-1=(a+))(@-1)=at1 a—1=1 implies a=2 and hence p=a+1=2+1=3 pta=3+2=5, also a prime number. So the number of divisors of p+a=5 is 2. Answer : 2. 18. a=14+3+54+7+--++-2009 b= 24+4464+84-+: +2010 then the value of (a — 6)? 2009 = 2010 -1= 2% 1005-1 Therefore, a is the sum of the first 1005 odd numbers = (1005)? b = ALF 24344-4--'+ 1005) 2x 4008 X 1008 005 x 1006 152 Problems and Solutions 2.a~b = 10057-1005 x 1006 1005[1005 ~ 1006) = ~1005 (a—)? = (-1005)? = 10057 Aliter a-b=-1+ -14 -14+ -14-+-+ -1 and there are 1005 terms. 4 a-b (ao)? 1005-x -1 = -1005 (—1005)? = 1005? i) 19, 1} +29 +3$+---n=3;, on complete simplification has the denominator —_______. Solution: i foe né+2n atgtet a+T The common denominator ‘of the fractions is lem of the denominators. On simplification the denominator will be a “divisor of the lom of the denominator. oe 3 a Example: 5 +5 3: 6 is thelom of 3 and 6, and the fractions when wudbed a2 and simplified give the denominator 2 wh which Ba progr divisor, of the lem 6, In the case of ae the denominator remains to be the lcm, & fact 8 is itself a divisor of 8). Note: If the problem is modified as n nti +2 n+l ae n+1 13 x 2x 8E Ke xn n(n +2) 8x5, Ce aeeetle es Problems and Solutions 153 (eaKSuen Kn) eax dx bx (nti) x e+) 2x3x4x-+-nx (n+1) ‘The denominator which is a.product of natural numbers from 2 to (n +1), is also a factor of the numerator. On cancelling the factors the denominator becomes 1. The answer in this case is 1. Answer : 1 20. The biggest value of ;19%(a € N) is never greater than Solution: 10a _ 10 Wa 44 0 on dividing numerator and denominator by @ which is a natural number. However great a might be the denominator can. not be less than 1 as a is e natural number ((i.e.,) a positive integer). Therefore the fraction can not be more than .10. {when a becomes very large, the fraction is very near 10]. So the biggest value is never greater than 10. 21. From a point } within an equilateral triangle perpendiculars are drawn to the three sides and are 5,7 and gcms in length. The perimeter of the triangle is _—-. cm. Solution: Let.the side of the equilateral triangle be a cm. oO is the given point. . Area of the triangle ABC = {| Sum of the areas of the triangles B ‘. OBC,OCA,OAB 154 Problems and Solutions 7 Peqem = 5{6a+ 70+ Qo] sq.cm = 4 sq.m = Basa v8, 2 2 4° 7 3 2 Perimeter of the triangle = 3acm= 126 om for 42% /3 cm] 22, The number of terms in the expansion (a + b + c)® is Solution: (atb+o8 = (a+b+o)*(a+b+c) = (+P +e + 2ab + Bbc + 2ca)(a+b+c) =, a3 +ab? + ac*+-207b + 2abe +-+2c07 tba? +B + bc + Dab! + 2B%e + Sabo tea? + ob? +8 + 2abe + 2b? + Aca Simplifying (4 +b +c) = a3 + 0° + & + 30% + 346? + 3a? + 3a + 304¢-+ 3bc + Gabe and there are 10 terms. 93. The-value of 2 satisfying the equation ——— = } is “aT Problems and Solutions 155 Solution: The L.H.S. and 2a+ Ba4+2 =3 => 824+4=92+6>¢=-2 Answer : -2. 24, ABCD is a rectangle rotated = 7. clockwise about A by 90° as shown, Therotation takes B to BY, C to C',.D to D!. AB = 6cm, BC' = 10cm. > c The' breadth. of the rectangle i ABCD sb —______. a Solution: Join BC’. C’D'B ie a tight angled triangle; Let the breadth AD = AD! = b,AB = 6 cm, BO’ = 10cm and C'D! = AB = 6cm. D'B=6+b 156 Problems and Solutions Applying Pythagoras theorem, CB? = o'D? +D'B? or 10? = 674 D'B? DB? = 100-36 =64=8 DB = 8=D'A+AB = DA+6 DA = Im The breadth of the rectangle is 2 cm. 25. AB is a line segment 2000cm long. The following design of semicircles is drawn on AB, with AP = Sem and repeating the designs. The area enclosed by the semicircular designs from A a egek P, Solution: AP =5 cms. P. uw Y Sem The number of designs repeated = Ea = 400 Problems and Solutions 157 ‘The area for one design is area of the semi circle on AP, as the two smaller. semi circular regions are equal and one of the semi circular regions is included in the bigger semi circular region the other is omitted. .'. The area of all the 400 semi circular regions 1 25 = Bm * 7 * 400-80 cm. = 12507 aq.cem Answer : 12507 sq.cm. KAPREKAR CONTEST Sub-juuion Louel Fival Tost-2010 1. @ and b are positive integers with a > b. Find all pairs of a,b such that the sum of their sum, difference, product and quotient is 36. Solution: From the given condition we get (a+b)+(a—b) +ab+¢ = 36 2a+ab+ ; = 36 2ab+ ab? +a = 36d a(?+2b+1) = 36b 360 a= (®) (6+1)? Since b and b +1 are relatively prime to each other for any positive integer b, (b +1)? divides 36. = b+1=2,b41=3,b4+1=6 => b=1,b=2,b=5. + a=9,a=8,a=5 (using (*)) Since a > b,a=5,b=5 is not admissible. , There are two pairs (a,b) = (9, 1), (8,2). 2. The three digit numbers ABC,BCA,CAB are added and a four digit number with the four digits different, results. If A,B,C are distinct, find all ABC and the sum of all such three digit numbers Solution: Since the sum of three digit number ABC, BCA and CAB isa 4 digit number with different digits A+ B-+C should give a carry over 1 or 2. Problems and Solutions 159 If it is two then the sum of the digits A+ B+C should be from 20 to 27. Since A,B and C are distinct 27,26,25 are not possible. For the sum of the digits from 20 to 24 sum of the three numbers ABC,BCA,CAB are 2664, 2553, 2442, 2331, 2220 respectively and the digits of these sums are not distinct. Similarly A+B+C = 11 to 18 also give the sum to be a 4 digit number with repeated digits [Ex. A+ B+C = 11 gives 1221,---A+B+C =18 gives 1998]. So the only choice left is A+ B+C = 19 which gives the sum of the numbers ABC + BCA+CAB = 2109, where all the four digits are different, such that the number A, B,C can be 289,379,478,469,568. There are exactly 5 distinct 3 digit number satisfying the given conditions. Their sum is 289 + 379 + 478 + 469 + 568 = 2183(289 —+ 892 — 928ete). Again all the digits of their sum are also different. 2 Using all the digits 1,2,3,4,5 one can form the 5-digit number 14352. It is observed that no three successive digits are in ascending order or in descending order. Find twenty such five digit numbers including the biggest and the smallest. Solution: Let us fix the start and end digits first and look for the middle digits later. Suppose the start digit is 1 and end digit is 2. the number looks The second digit can not be 3, because if the second digit is 3, then the third digit will be 4 or 5 which makes the first 3 digits in ascending order like [1.3 4 ] 52 or [13 5] 42. These numbers are not acceptable. Again taking the numbers 1...... 2 suppose the second digit 160 Problems and Solutions = is 4, the number becomes 14-2. We are left with 3 and 5. We can form only one number 14352 as 1 not acceptable. Suppose the second digit is 5, the number becomes 15 — -2. We are left with digits 3 and 4. We can form only one number 15342 as 1[5| 43] 2 _|is not acceptable. We can workout with different combinations of start and end digits and tabulate to find the numbers as follows. Start digit End digit Number 1 2 14352, 15342 2 1 3 14253, 15243, 2 1 4 13254 1 1 5 14235 1 2 3 24153, 25143 2 2 4 21534, 21354 2 2 5 21435 1 3 4 31524, 32514 2 3 5 31425, 32415, 2 4 5 41325, 42315, 2 There are 17 numbers in the tabular column with start digit. less than the end digit. If you reverse 17 numbers, you get another 17 numbers satisfying the condition with the start digit greater than the end digit. There are [34] such numbers. Of which 53412 is the biggest and 13254 is the smallest. . In triangle ABC, ZACB = 120° and ZCAB = 40°. AC is extended to P such that AP = AC +2CB. Find the measure of ZABP. Probleme and Solutions 161 Solution: Given: AP = AC+2CB (ie) AC+CP = AC+2CB =CP = 2CB Let Qbe the mid point of CP >. CQ=QP=CB BGQ = 60° and the triangle QC'B ‘is isocircles. => AQCB is equilateral. Since QP = QC = QB, QBP = QPB 2. Ext BQC = 60° = 2QBP =QBP = 30° ~ ABP = 20° + 60° +.30° = 110° 5. A faulty odometer of a car proceeds from digit 8 to the digit 0 skipping the digit 9 regardless of the place value of the position where 8 occurs, For example travelling 1km, the odometer changed from 000038 to 000040. If the odometer now reads 002010, find the actual distance travelled by the car. Solution: In the normal odometer 10 symbols 0,1,2,3, 4,5,6,7,8 and 9 are used in each place value. The place value 162 Problems and Solutions for numerical system is as follows: Thousands | hundreds | tens | units | or 1000 | 100 | 10 | 1 In the faulty odometer only 9 symbols 0,1,2,3,4,5,6,7 and 8 are used in each place value. Seven hundred | eighty ones | nines | units and twenty nine or 9x9x9/9x9}9]}1 <. The number of readings (scrolled) from 000000 to 002010 in the faulty odometer is given by 9x9x9/9x9/9]1 2 0 14,0 = 729 x 24+81x0+9x 141% 0= 1467 <. The actual distance covered is 1467. 6. ABC isatriangle. BA is produced e to P such the BA = AP,AC is produced to Q such that AC = CQ and CB is produced to R such Ts that CB = BR. The area of the triangle PQR. is 2010 square units. g- a Find the area of the triangle ABC. 0 Solution: Let the area of the triangle ABC be a. Consider the two triangles ABC and ARB. Problems and Solutions 163 Since BC = RB and they have the same altitude, we have AABC = AARB =a Consider the triangles RBA and RAP. By the same reason ARAB = ARAP Thus ARBP = 2a. By similar arguments we get ARCQ = 2a and AQAP = 2a APQR = 2a+2a+2a+a=7a = 2010 AABC = 2A savare units 2 A square piece of cloth of side 4m is cut into thin strips of width lcm parallel to a side. These cut strips are glued together to form a long strip (ignore the length shortened by gluing the smaller strips). This long strip is made to enclose a square region along the boundary. What is the area of the region? If the long strip is made to enclose a rectangle one of side 4m along the boundary find the area of the rectangular region. Solution: AB = 4m =400em= BC = AD=CD. The width of the strips =1 cm 400 cm lem No. of strips cut out = = 400 Length each strip = 4m. So the length of the long tied up strip = 400 x 4 = 1600 m [Since loss of length on tying is ignored]. When this long strip is made to form a square enclosure each side is —— = 400m. Area of the enclosure = 400 x 400m? =1,60,000 m?. If one side of rectangular enclosure is 4m and the other 164 Problems and Solutions side is 1, then 2[4+1] = 1600m 4+1 = 800m 1 = 796m <. The area of the rectangular enclosure = 4x 796 = 3184 m? 8. How many two-digit natural numbers are increased by 11 when the order of the digits is reversed? Solution: Let (10a +6) be the two digit number given 1Wb+a = 1+10a+d => 10(b-a)+a-b = Il > = 11@-a)-(6-a)+(@-b) = 1 > U(b-a)+2a-b) = 1 i] Since 11(b— a) and 11 are divisible by 11, 2(a — b) must be divisible by 11. => (a—b) must be divisible by.11 which cannot be as a,b < 10. The only possibility is a—b=0 > a=b. By reversing the digits we get the same no. So it is not possible. Thus there are no two digit numbers which satisfy the given condition. Aliter: 10b+a 11+10a+b > 9a = 9-11 1 a=b Since a,b are digits, the equation has no solution for a,b in digits. Thus there are no two digits numbers which satisfy the given condition. KAPREKAR CONTEST Sub-jurion Level Serecuing Test-201l PART-A 1. Given 63.63 = m(21+ ;45),m,n positive integers. with n< 100. The value of (m+n) is (A) 21 (B) 24 (C) 104 (D) 101 Solution: 63.63 = m(21 + ) multiplying both side by 100, we get 6363 = m(2100-+n) q) 6363 3 x 2121 ™ = 300+n 2100+ n* Since m is an integer, 2100-+n should divide 3 x 2121, where nis a positive integer less than 100. n = 21 gives Bx 2121 3x 2121 _ ™=7100+n ‘2121 Thus n = 21, m = 3 satisfies the equation (1) and the given conditions. .", m+n = 24. Answer : (B)- 2. m is a natural number and (n +2)(n+4) is odd. Then the biggest power of 2 that divides (n +1)(n +3) for any n is (A) 1 (B) 2 (C)3 (D) 4 Solution: Since the product (n-+2)(n +4) is odd n should be odd say (2k +1) (n+ 1) = (2k +141) = (2h +2) = 2(k +1) (n+ 3) = (2k +1 +3) = (2k +4) = 2(k +2) (n+ 1)(n +3) = 2k +1) x 2(k + 2) = 4(k +1)(k + 2) 168 166 Problems and Solutions (k+1) and (k+2) are consecutive integers and hence one of them should be even. ©. (n+1)(n +3) =8 xc for some integer ¢ =Bxe. Therefore biggest power of 2 that divides (n + 1)(n +3) for any 1 is 3. Answer : (C). Note: Taking n = 1,3,5,7,---. We get (n+1)(n+8) to be 2x4=8=2, 4xG6=8x3=3x 2, 6x8=3x 2, 8x10=24x5, 10x 12=2%x 15 and soon. But 2° divides (n+1)(n +3) for all odd values, But 24 and 2° or still higher powers do not always divide the product and hence the answer is 3. 3.Two squares 17emx17em overlap to form a rectangle 17cm x 30cm. The area of the over lapping region is (A) 289 (B) 68 (©) 510 (D) 85 6 o 4 c GS area et eae agie Solution: AB + FE = 17cm + 17cm = 34cm. AF = AE+EB+BF=30cm B = AE +EB; EF = FB + BE (in the overlapping rectangle) AB+FE = AE+EB+EB+FB=34em (1) ButAF = AB+BF = 30cm AE +EB+FB=30cm (2) (1) - (2) gives BB =4cm. Problems and Solutions 167 “ ‘The overlapping rectangle EBCH has EB =4cmand BC = 17cm. +, The area of EBCH the overlapping is 68 cm?, Answer : (B). Note: If the squares are just touching, but not overlapping the rectangle is of size 17 x 34. But the overlapping one is of size 17 x 30. ©. The overlapping region is 17 x 4 = 68em?. |. Aruna and Baskar wrote the three digit number 888 and Aruna changed two of its digits and wrote the biggest three digit number divisible by 8. Bhaskar too changed two of the digits of 888 and wrote the smallest three digit number divisible by 8. The difference of the new numbers obtained is (A) 848 (B) 856. = (C) 864.—(D) 872 Solution: In the new numbers one digit is 8. To get the biggest number we should use the digit 9 in the 100th place. Then 984 is the biggest number with one digit 8 retained and divisible by 8 [998 is not divisible by 8). To get the smallest number the 100th place should be 1 and let 8 occupy the units place. Then 1818 is divisible by 8, only when the place holder & takes the value 2. Then the smallest value satisfying the condition is 128. The difference is 984 — 128 = 856. Answer : (B). 7 is a negative integer. The expression having the greatest value is, (A) -2n?+2n (B) -2n?—2n (C) 2n?+2n (D) 2n?-2n 168 Problems and Solutions Solution: As n is a negative integer, 2n is negative, —2n is positive and n? is always positive for any n. —2n2-+2n is less than zero as both —2n? and 2n are negative. —2n? negative and —2n is positive —2n? —2n > —2n? +2n. ‘Again 2n?-+2n < —2n?—2n because both 2n? and —2n are positive and their sum is greater than the sum in any of A, B and C, as they have at least one negative valued term. So, 2n? — 2n has the greatest value. Answer : (D). 2 . Five positive integers are written around a circle so that no two or three adjacent numbers have a sum divisible by 3. In this collection the number of numbers divisible by 3 is (A) 0 (B)1 (C)2 (D)3 Solution: Let the five numbers arranged around the circle be P,%2,Y,2. p+q is not divisible by 3, neither is p+q+z. If p,q and are of form 3k-+1, then p+q+2 is divisible by 3. With out loss of generality take p of the form 3k; +1. Taking q of the form 3k, +2 makes p+q divisibly by 3, ie., (3ky + 1) + (Bkz +2) = 3(ky + ke +1). Therefore, P let pbe = 3k z q andq = 3k +1, gc = 3ke4+1 y (x can not be of the form 3k2 +2— why?) now neither (p+) nor (q+) nor p+q+< are divisible by 3. Problems and Solutions 169 ‘The same argument shows that y can not be of the form 3k-+1 or 3k +2. So y should be of the form 3k. Now if y = 3k3, then qtr =3(ky + ke) + 23g +2 +y = 3(ki + ko + ks) +2, so the condition is satisfied. Now y and p are multiples of 3, q and x are 1 more than a multiple of 3. So z should also be of the form of 3k +1. So two of the five members are divisibly by 3. Note: Instead of making the numbers q,z and z as 1 more than a multiple of 3, you may take them as 2 more than a multiple 3. Verify. Answer : (C). 7.A number is called a palindrome if it reads the same forward or backward. For example 13531 is a palindrome. The difference between the biggest 10 digit palindrome and the smallest 9 digit palindrome is (A) 976666666 (B) 9888888888 (C) 9899999998 (D) 9777777777 Solution: The biggest 10 digit palindrome number is 999---99 The smallest 9 digit palindrome number is 1000-:Q1 7 times 1998 7 times The difference is 989 Answer : (C). Note: The difference is not a palindrome. 170 Problems and Solutions 8. In the figure, PQRS is a rectangle of area 2011 square units. K,L,M,N are the mid points of the respectiv sides. O is the midpoint o MN. x The area of the triangle OKL is equal to (in square units) @) 2 et ° 2011 3(2011) (Ep) em) Solution: Join NK and LM. Since K,L,M,N are mid points of the rectangle, the triangles SKN, PLK,QML and $ N R and NK of KLMN are equal and hence therefore : KLMN isarhombus. Area P t a of the rhombus = 1/2 area of the rectangle PQRS.. Also KLIMN. RMN are congruent (by 2 sides and included angle). So, - a the four sides KL,LM,MN x | Area of AKLO = 3 [area of the rhombus KLMN] =} (} area of the rectangle PQRS] 201 4 +. [KLMN and KLO are between the same two parallel lines KL and MN] Answer : (C). Problems and Solutions aa 9. A computer is printing a list of the seventh powers of all natural numbers, that is the sequence 17,27,37,---. The number of terms (or numbers) between 5%! and 48 are (A) 12 (B) 130 (CG) 14 (D) 150 Solution: Bh = (58) = (125)7 gs = (44)? = (256)7 and in the sequence 17,27,37,47,--. ,1247,1257 1967,--. ,2557, 2567. There are 256 — 126 = 130. [Here the terms are between 1257 and 2567, excluding 125 and 256]. Answer : (B). 10.A natural number n has exactly two divisors and (n +1) has three divisors. The number of divisors of (n +2) is (A)2 (B)3 (C)4__(D) depends on the value of n Solution: As n has exactly 2 divisors, it is a prime. Also n+1 has exactly 3 divisors and thus it is the square of a prime. If n = 2, n+1=3 has only 2 divisors. -. n > 2, is an odd prime. Also if n > 3, then n +1 cannot be the square of a prime number. .°. n #2, n =3 is the only solution. Thus n+2=5 has exactly 2 divisors. ‘The answer is 2. Answer : (A). 11. B44, =r where p,q.7 are positive integers and p=q. The biggest value of r less than 100 is (A) 19 (B) 57 (C) 76 (D) 95 4172 Problems and Solutions Solution: z + 4 =r multiplying both sides by 90 10p+9q = 90r or 19p = 19q = 90r (since p=q) 90r + 90 and 19 are relatively prime. Hence r should be a multiple of 19 clearly the biggest value of a multiple of 19 less than 100 Pp is 95. = 95. Answer : (D). 12, In the grid shown in the diagram A and B erase 4 numbers each from the table. The 13] 14] 15 sum of the numbers erased by them are in 20 | 21 | 22 | the ratio 10:11. After erasing the numbers, 27] 28} 29, the number left out on the table is (A)21 (B) 14. (0) 22, (D) 13 Solution: The sum of all the numbers in the table is 189. Let the sum of the numbers erased by A be 10¢ and B lz. ‘The sum of the 8 numbers erased by both is 10x + Ma = Qe Let the number not erased by y. y = 189 — 21a, or 2le < 180% <9. y = 189 — 2x < 29 , Qe > 160. 28. As a<9 a must be = .. The number that remains on the table is 189 — 168 — 21. Answer : (A). Problems and Solutions 173 13. The positive integers p,q. p—q and p+q are all prime numbers. The sum of all these numbers is (A) divisible by 3. (B) divisible by 5 (C) divisible by 7. (D) prime Solution: p—q and p+q are both odd or even. They are primes = they are both odd. .. One of p,q is odd and the other is even 2 is the only even prime. “G2. Hence, p—2, p, p+2 are consecutive odd primes. Hence they are 3,5,7. 5, 2, 5-2, 542 are alll prime. Sum is 24+345+7=17 which is a prime. Answer : (D). 14. a,b,c are positive reals such that a(b-+c) = 32, b(c+a) = 65, and c(a+6)=77. Then abc = (A) 100 (B) 110 (C) 220 (D) 130 Solution: a(b +0) = 32 @ b(c-+@) = 65 (2) c(a+b)=77 (3) adding 2(ab + be + ca) = 174 ab + be + ca = 87 (4) From (1) and (4) be = 87 — 32 = 55 (2) > be-+ab = 65 55+ab=65 ab=10 174 Probleme and Golutione (3) Sac+be=77 ac ab-bevca = 55 x 10 x 22 BX UX 5x 2x 1x 2= (2x5 x L)* 2x5x1l 110. Answer : (B). Note: Here the — ve sign is ignored because a,b, ¢ are positive reals. 15. 100th term of the sequence 1,3,3,3,5,5,5,5,5,7,7,7,7,77,7 «+ is (A) 15 (B) 13 (O17 (D) 19 Solution: The sequence is 1,3,3,3, 5,5,5,5,5, 7,7,7,7,7,7,7, Let us divide them into blocks of equal numbers Block No. of Numbers Total 1 1 1 2 3 4 3 5 9 4 7 16 5 9 25, 6 i 36 7 13 49 8 15 64 9 7 81 10 19 100. 100th term is 19. Answer : (B). Problems and Solutions “76 PART -B 1, When a barrel is 40% empty it contains 80 litres more than when it is 20% full. The full capacity of the barrel (in litres) is Solution: Let the full capacity of the barrel be « litres. 40% empty means 60% full. 60% of the capacity of the barrel is 8 x litres 20% of the capacity of the barrel is 29, x litres 00 60 20 Too" = i907 +89 oF 40 80 x 100 F To = 9 = Fp = 200 litres ‘The full capacity is 200 liters. Note: Without using x we can caleulate the full capacity. 60% capacity = 20% capacity + 80 liters. ., 40% capacity = 80 liters. Full capacity = 19? x 80 = 200 liters. Answer : 200 Liters. 2. a,b,¢ are the digits of a nine digit number abcabcabc. The quotient when this number is divided by 1001001 is i Solution: The nine digit number abcabeabe in standard notation is 108a + 107 + 10% + 10%a + 104 + 10%c + 10?a + 10b+¢ = 10°(102a + 106+ c) + 103(10%a + 10b +c) + 1(107a + 106+) = (10a + 10b + ¢)(10° + 10° + 1) = (10a + 10b + ¢) - (1001001) i . The quotient when the 9 digit number is divided by 1001001 is abe. 176 Problems and Solutions The answer is abc or in standard notation 100a + 10b+c. Answer : 100a + 10b+c. 3.The first term of a sequence is (3). Each new term is calculated using the formula (152) where is the preceding term. The sum of the first 2011 terms is 5 Solution: Let us call the terms of the sequence ti, ta, t3,-++ , toon - = tao = 5/9. Their sum = 1006 x 3 + 1005 x § (since there are 1006 odd positions in 1 to 2011 and 1005 even positions in 1 to 2011 Similarly tp = t4 = ty = ts = 53283 (17761 16 = 63 = ar = 845 Qy- 53283, 17761 16 Answer : 6 " a aS . ‘The number of integers n for which j= is the square of an integer is » Solution: Let g2; = m?. Since RHS is positive, LHS must be positive =n<20. If n=10, then p25 = =m? >m=1. If n=11,12,-++ , 15,17, az is not a perfect square. If n=16, then gS = 8 =4=m? a m=2. If n=18, then gig = B= When n is 10, 16 or 18, m2; is a perfect square. =m>m=3,. Answer : Number of integers is 3. Problems and Solutions 477 5. Alll sides of the convex pentagon ABCDE are equal in length. A= ZB =90°. Then ZE is equal to —. Solution: As per the data given, > the pentagon with equal sides is as shown in the figure. Ez c ADGE is equilateral ZDCE = 60° and ZAEC = 90° :. LE= ZDCE + ZAEC = 60+ 90 = 150°. A Ba Answer: ZE = 150°. = |. The units digit of the number 37° js ——. Solution:The units digit of powers of 3 are as shown below: 333 #49 B47 341 333 Thus we have blocks of 3,9,7,1 as the units digit. Since 2011 = 2008 +3 =4 x 502 +3, the units digit of 3°91 is 7. Answer : 7. 7. The number of digits used to write all page numbers in a: book is 192. The total number of pages in the book is : Solution: The number of pages of the book with page number a single digit is 9 (i.e., 1 to 9). 0) ‘The number of pages of the book with page numbers 10 to 99 (ie., each page has 2 digit number) = 90. (2) (1) and (2) add up to 99. 178 Problems and Solutions Using 9 single digits and 90 two digits. We have 99 pages and the number of digits used to write (99) page numbers is 9+ (90 x 2) =9 + 180 = 189 3 digits (192 — 189) are yet to be used for some more pages. The next page number is 100 and it uses exactly 3 digits. So the book has 100 pages. 8. A bar code is formed using 25 black and certain white bars. White and black bars alternate. The first and the last are black bars. Some of the black bars are thin and others are wide. The number of white bars is 15 more than the thin black bars. The number of thick black bars is Solution: Number of black bars = 25. The first and last ones are black. +. Number of white bars = 25 = 24. The number of thin black bars = 24—15 = 9. (white bars are 15 more than thin black bars) .. The number of thick black bars = 25-9 = 16. Answer : 16. 9. By drawing 10 lines of which 4 are horizontal and 6 are vertical crossing each other as in the figure, one can get 15 cells. With the same 10 lines of which 3 are vertical and 7 horizontal we get 12 cells. The maximum number of cells that could be got by drawing 20 lines (some horizontal and some vertical) is ——. Problems and Solutions 4179 Solution: The number 20 can be written as the sum of two non zero numbers as 10+10,9+11,8+12,---,2+18,1+19. Thus the number of vertical lines and horizontal lines may be any one of the above pairs for 1 vertical line and 19 horizontal lines or 1 horizontal line and 19 vertical lines, the number of cells will be zero. The number of cells is the product of the gaps between the horizontal lines and vertical lines, which are 9 x 9,8 x 10,7 x | 11,-+- ,0x 18 and 81 is the maximum value of these products, so the maximum number of cells is 81. Answer : 81 10. If a? — 6? = 2011 where a,b are integers, then the most negative value of (a +6) is . Solution: a = 2011 (a+6)(a—b) = 2011 x1or —2011 x -1 (a+b) 2011 or — 2011 Most negative value of a+b is —2011. 11. and y are real numbers such that zy = 2-+y = 2(0 # 0,y 40). Then the numerical value of (x —y) is Solution: i zy=— > wy-2=0 y => a(y’-1)=0 c40 => y=ly=tl sy=aty > y=sy—2=a2(y-1) or z= Fy. ", y= 1 can not be taken. “.y = 1 and hence x = }. So the values of z,y are } and —1 180 Problems and Solutions az-yis }-(-1)=3. Answer : 3. 12. When « = 2011, the value of 7 is i = Ay +—y l+= z Solution: 7 ba T 2 +z 14+$—- 1+= 1+oo a 1 cree reg fa Tee a0 +m 1 * a3 2012 4023 Answer : Tae 13. After the school final exams are over, all students in a class exchange their photographs. Each student gives his photograph to each of the remaining students and gets the photograph of all his friends. There are totally 870 exchanges of photos. The number of students in the class is Solution: Let the number of students be «. Number of photos exchanged by any student = (x — 1). Number of photos exchanged by « students = a(x —1). By data a(z-1) = 870 30(30-1) = 870 . @ = 30 No. of students in the class 30, Problems and Solutions 14. The angles of a polygon are in the ratio 2:4:5: 181 ‘7. The difference between the greatest and least angle of the polygon is Solution: Let each angle be Qe: de : Sax: Br : 6x: Sum of angles = 30c = (2n-4)90 = 302 Putting n = 6, (12-4)90 = 302 8x90 = 30c Tr a= 24, Difference between greatest and least’ = 52 = 5 x 24 = 120° Answer : 120°. 15. The perimeter of a right angled triangle is 132. The sum of the squares of all its sides is 6050. The sum of the legs of the triangle is Solution: By dataa+b+e = C+P+e = 3 +2 = c= c 132 6050 6050 3025 55 => a+b=132—55=77 units. KAPREKAR CONTEST Subjuuion Level Fiual Test-2011 1. Find all three digit and four digit natural numbers such that the product of the digits is a prime number. Find the sum of all such three digit numbers and the sum of all such four digit numbers. Find the biggest prime factor of each sum. Solution: The products of the digits of a number is prime if one of the digits is either 2,3,5 or 7 and the others are all 1. Three digit numbers. Four digit numbers 112,121,211, 1112, 1121, 1211, 2111, 113,131,311, 1113, 1131, 1311, 3111, 115,151,511, 1115, 1151, 1511, 5111, 117,171,711. 1117, 1171, 1711, 7111. There are 12 3 digit There are 16 4 digit numbers where product numbers, whose product of the digits isa prime —_of digits is a prime number. Sum of these number. Sum of the 16 12 numbers numbers = 2775 = 32219 = 3x5?x 37. = 11x 2929 = 11x 29x 101 ‘The biggest prime factor The biggest prime is 37. factor is 101. 2. Let a sequence of numbers be denoted as t1,ta\tg,+++, where ty =1 and tp = ta-1+n. (n is a natural number), Find ta, t3, t4, tro, tao « Problems and Solutions 183 Solution: t=1, 2=14+2=3, tg =6+4=10, to =1+24+34+44+---410 2011 x 2012 too = ——3——— = 2011 x 1006 = 2023066, This sequence can be seen to be ty = Lte=14+2,t3 =1424+3,t=1424344,--, too =14+2+3+--- +2011 = 20llx2012 — 9993066. ‘The terms of this sequence are triangular numbers. 3. When written out completely 167°! has m digits and 6257011 has n digits. Find the value of (m+n). Solution: 161 = (24201 (46 = at) = 980 625 = (582011 G95 — 54) 8044 1. 1601 x 6257011 = (16 x 625)2011 = 28044 x 8044 = (10) No of digits of 1621 x 62520! — 1984 is go45. Observe the following patterns: 1. 2!x 5! = 10! has 2 digits =(1+1), where 2 and 5 are single digit numbers 2. 22 = 4, a single digit number, 5? = 25 is a two digit number and 2° x 5? = 100 is a three digit number =(1+2). 3. 23 x 53 = 103 = 1000, a 4 digit number. 23 = 8 is a single digit number and 5° = 125 is a3 digit number and 4=1+3. 4, 24 x 54 = 104 = 10,000 is a five digit number. 24 = 16 is a 2 digit number, 54 = 625 a three digit number and 5 = 2+3. 184 Problems and Solutions 210 — 1024 is a four digit number 5'° = 625 x 625 x 25 is a seven digit number. No of digits of 2!° x 5!° = 10!9 = 11 digits =4+7. In general an m digit number multiplied by an ‘n’ digit number give a product of (m-+mn—1) digit or (m+n) digits. But it has been observed that multiplying powers of 2 and the same powers of 5 give a product having number of digits to be the sum of the digits of the power of 2 and the same power of 5. So, 1671 — 2844 has m. digits. And 625711 = 584 has n digits. 7, 28044 5 58044 — 108044 — 199000 ---00 (8044 zeros) and hence has 8045 digits : “m+n = 8045. 4, Four digit numbers are formed by four different digits a,b,c,d (none of them is zero) without any repetition of digits. Prove that when the sum of all such numbers when divided by the sum of the digits a,b,c,d, the quotient is 6666. Solution: Let [abcd] be a four digit number with different digits. Then [abcd] = a(10)° + 6(10)? + c(10)! + 4. ‘The numbers in which d is the unit digit are abet There are 6 numbers in achd which the unit digit is bead d. bacd cabd cbad™ Ina similar way, there are 6 numbers with d in the tenth place, in the hundreds place and 6 more with d in the thousands place. The same is the case for the other three digits a,b,c. Problems and Solutions 488 ‘Thus adding them we get, 6(a +b +c+d)(10*) + 6(a +6 +c +d)(10") +6(a+6+¢+d)(10)) +6(a+b+c+d) =6(a+b+e+d)[10+ 10? + 10 + 1] When it is divided by (a +b+c+d) the quotient is 6(10° + 107+ 10+1) = 6(1000+100+10+1) 6(1111) = 6666. Note: Write down all the 24 numbers using a,b,c,d and verify! 5. ABCD is a parallelogram. Through C a straight line is drawn outside the parallelogram. AP,BQ,DR are drawn perpendicular to this line from A,B and D. Prove that AP = BQ+DR. Solution: Draw DSLAP. Clearly DSPR is a rectangle (since 3 angles are 90°) =>SP=DR QQ) In ADSA and ACQB AD = BC ZDSA = ZCQB = 90° 186 Probleme and Solutions Since AP || BQ and AD || BC ZDAS = ZCBQ .. The two As are congruent. = AS =BQ (2) Adding (1) and (2) we get AS +SP=BQ+DR => AP = BQ+DR. 2 . Falguni puts 12 plastic bags inside another plastic bag. Each of these 12 bags is either empty or contains 12 other plastic bags. All together if 12 bags were non-empty, find the total number of bags. Solution: Falguni puts 12 plastic bags inside another bag (perhaps the biggest). Altogether 12 bags are non empty. So the biggest bag which contains 12 bags is nonempty. As it is given that 12 bags are nonempty, 11 of the bags inside must be nonempty, each containing 12 bags. Therefore the total number of bags = 1412411 x 12=1+412+132 = 145 bags. [one bag outside - the biggest bag it contains 12 bags inside it and out of these 12 bags inside the big bag, 11 bags contain 12 bags each]. 7. A,B and C are the digits of a three digit number ABC(A,C #0). The number got by reversing the digits (also a three digit number) is added to ABC and the sum is found to be a square number. Find all such three digit numbers. Solution: Let the number in standard notation be [ABC] = 100A +10B+C. 0) The number got by reversing the digits 100C + 10B + A--- (2) Problems and Solutions 187 Adding (1) and (2), $= 101(A+C) + 20B We should find the values of A,B and C, so that S is be a square number. Since A,B and C are digits of a number A+C cannot be greater than 18. 2 < A+C < 18. The unit place digit is contributed by 101(A+C), as 20B ends in 0. Case: (1) (A+ C) = 18, 17,13,12,8,7,3,2, then S = (101) x (A+C)+20B ends in a digit which is invalid for a square number. Hence A+C cannot have these as values. Case: (2) A+C=16. S=101 x 16 + 20B = 1616 + 20B 1616 < S < 1796. There is no square number in the interval with 6 in the unit place. The only square numbers are 1681, 1764, which do not end in 6. Case: (3) A+0=15. $= 1515 +20B ends in 5. 1515 < S$ < 1695. The square numbers in this interval are 1521 and 1681 only. They do not end in 5. So no solution with A+C=15 also. Case: (4) A+C=14. S = 1414+ 20B ends in 4. 1414 < S < 1594. The square numbers in this interval are 1444, 1521. If § =1444, then 208 =30 B=3/2. Invalid solution. Again no solution with A+C = 14, 188 Problems and Solutions Case: (5) A+C=11. S$ =1111+20B ends in 1. 1111 < S < 1291. Square numbers in this interval are 1156 and 1225. They do not end in 1. Again no solution with A+C=ll. Case: (6) A+C=10. S=1010+20B ends in 0. For S to be a square number, S must end in 2 zeros ic., 20B = 90 or 190. Both are not possible. Hence again no solution. Case: (7) A+C=9. 5 = 909+ 20B ends in 9. 909 < S < 1089. The only square numbers in this interval are 961, 1024 and 1089. If $ = 1089, then B = 9. Possible numbers are 198, 297, 396, 495, 594, 693, 792, 891. Case: (8) A+C=6. S = 606 + 20B ends in 6. 606 < S < 786. Square numbers in this interval are 625, 676, 729, 784. Valid square value for S is 676 as it ends in 6. This gives 20B = 170, B = 17/2, an invalid solution. So no solution for A +C = 6 also. Case: (9) A+C=5. Problems and Solutions 189 S = 505+ 20B ends in 5. We must have the tens place of Sas 2 for a square number. 505 < S < 685. An acceptable square number ending in 5 is S = 625 = 505+ 120. B=6. <, 20B = 120. Numbers are 164, 263, 362, 461. Case: (10) A+C=4. S = 404+ 20B ends in 4. 404 < S < 584. The square numbers in this interval are 441, 484, 529, 576 of these only 484 is acceptable leading to B= 4. ‘Thus numbers with A+C =4 are 143, 242, 341. Solutions are: 143, 242, 341, 164, 263, 362, 461, 198, 297, 396, 495, 594, 693, 792, 891. 8. Nine squares are arranged to form a rectangle ABCD. The smallest square P has an area 4 sq.units. Find the areas of Q and R. ® ) ® ©) @) ®) 190 Problems and Solutions Solution: Let the side of B be x units. Then, side of A = side of B+ side of P = 2+2 (l) side of D = side of B— side of P = 2-2 (2) side of E = side of A+ side of P = c+24+2=244 (3) side of E = side of Q+ side of D — side of P r+4 = side of Q+a—2-2 (from (2) and (3)) side of Q = 8 side of C = side of B+ side of D = tt2-2=2-2 (4) ABCD is a rectangle side of R+ side of Q side of R+8 sideof R = side of F = side of R From (5) and (7) 3a — 12 2a x “, side of R=2+20= 36. Area of R= 1296 sq.units. side of C+ side of D Qn -2+"-2 3a — 4. 30-12 (5) side of E+side of Q ct+4+8=2412 (6) side of F + side of Q E+124+8=2420 © (7) c+20 32 16 KAPREKAR CONTEST Sub-junion Level Sorecning Test-2012 PART - A 2 Sea 1.p is an odd prime. Then (=) - (=) is (A) a fraction less than p . (B) a fraction greater than p. (C) a natural number not equal to p. (D) a natural number equal to p. Solution: 2 ) 4 4 p-1\* (p-1? _ P= 2p41 2 wre 4 Answer: (D) 2. PQRS is a parallelogram. MP and NP divide ZSPQ into three equal parts (4MPQ > ZNPQ) and MQ and NQ divide ZRQP into 3 equal parts (CMQP > NQP). If k(ZPNQ) = (ZPMQ) then k = (A) 3 (B)1 (©) $ (0) } Solution: By hypothesis ZNPQ = ZMPN = ZSPM =z (say). 191 192 Problems and Solutions Similarly ZNQP = ZMQN = ZRQM = y (cay). Now 32 +3y = 180? + r+y=60° 4 . on ZPNQ = 180°-(c+y)=120° , . ZPMQ = 180° — (2x + 2y) = 180° — 2(x + y) = 180°—2-60 = 60° Therefore k(ZPNQ) = ZPMQ implies k =}. Answer: (A) 3. In a foot ball league, a particular team played 50 games in a season. The team never lost three games consecutively and never won four games consecutively, in that season. If N is the number of games the team won in that season, then NV satisfies (A) 25 a=c c+td=d+e > c=e b+e=ct+d > b=d at+b=b+c=c+d=dte= 2012 at+b+c+d+e= 5024 => atb+atb+a=5024 => 3a+2b = 5024, or 2(a +b) +a = 5024 => 2(2012) +a = 5024 = a= 1000 b=2012—a=1012, d=b=1012 d—a= 1012-1000 = 12. Answer: (B). 5. The last two digits of 372, when represented in decimal notation, will be (A) 81 (B) 01 (C) 41 (D) 21 Solution: 81 81 81 61 41 21 ol 8 8 8 8 8 61 41 21 ol 81 From these observations, we can establish the following properties: 194 Probleme and Solutions Property A Numending Mult. with a Num results ina Num with digits ending with ending with digits digits (a) 81 81 61 (b) 61 81 Al (&) 4 81 a (a) 2 81 OL ) o1 81 al ~. We have 34,324, 344, 354, +, ending with digits 81. 38,378, 348... | ends with digits 61. 312, 382, 382... | ends with digits 41. 336, 386, 355... | ends with digits 21. 320, 340, 360... ends with digits 01. 32012 — 312410020) ends with digits 41. Answer: (C). 6. There exists positive integers «,y such that both the expressions (3¢-+2y) and (4a —3y) are exactly divisible by (A) 11 (B)7 (C) 23 (D) 17 Solution: 4(32+2y)—3(4e—3y) = 122+8y—122+9y = 17y. Also 3(32 + 2y) + 2(4e — 3y) = 9x + 6y + 8x — 6y = 172. ~. If 17 divides one among (3x + 2y), (4x — 3y) it divides the other too. Hence these exists positive integers «,y such that both, (32 + 2y), (4a — 3y) are exactly divisible by 17. Answer: (D). 7. The years of 20‘ century and 21% century are of 4 digits. ‘The number of years which are divisible by the product of the four digits of the year is (A)7 (B)8 (c)9 (D) none of these Problems and Solutions 195 Solution: For the 20 century the years are from 1900 to 1999 and hence the product of the digits of the years have 9 as a factor. So it is enough if we consider only 12% numbers. Of these 1900 to 1910 have zero as one of the digit. So it is enough to consider numbers from (1911 — 1919)(1921 — 1929) --- (1991 — 1999). 1917, 1926, 1935, 1944, 1953, 1962, 1971, 1989, 1998 are the numbers divisible by 9. (1) 1917 =9 x 218=9x3x71=27x 71 Product of the digits is 1 x 9x 1x 7 = 63. 1917 is not divisible by 7. (2) 1926 =9 x 214 =9 x 2x 107 Product of the digits is 1 x 9x 2x 6 = 27x 4 and the number 1926 is not divisible by 4 (neither by 27). (3) 1935 = 9 x 215 =9 x 5x 43. Product of the digits is 1x9x3x5=27%5. 1935 is not divisible by 27. (4) 1944 =9 x 216=9x8x27. Product of the digits is 1x 9x 4x4=9x 16. 16 does not divide 1944. (5) 1953=9 x 217=9x 7x31. The product of digits is 1x 9x5x3=27x5. 1953 is not. divisible by 5. (6) 1962=9x 218 =9x2x 109. Product of the digits is 1x9x3x4. 1962 is not divisible by 4. 196 Problems and Solutions (7) 1971 =9 x 219 = 27 x 73 and the product of the digits is 1x 9x 7x1. The year 1971 is not divisible by 7. (8) 1989=9 x 221 =9 x 13x 17. The product of digits 1x9x8x 9. The year 1989 is not divisible by product of digits. (9) 1998 = 9 x 222 =9 x 6x37. The product of digits 1x 9x 9x8. The year 1998 is not divisible by 1x 9x 9x 8. For 21% century all the year numbers from 2000 to 2099 contain one zero and divisibility by zero is not to be considered and hence no year number is divisible by the product of digits of the year number. (Zero does not divide any number but every non zero number divides zero!). 8. If +2 =-1, then the value of 23 — 3 is (A) 0 (B) 1 (C) -1 (D)2 Solution: r+i=-1 given > 2? +1 = g?=-2-1=-2+(r+}) => o=1 and s3-3=0. Answer: (A). 9. The number of natural numbers a(< 100) such that (a3 — a”) is a square of a natural number is (A)7 (B)8 (C)9 (D) 10 2 Solution: a3 — a? = a(a—1). For a—1 to be a square number, a should be = 2,5, 10,17, 26,37, 65,82. There are 9 such numbers. Answer (C). Problems and Solutions nen 10. Let n be the smallest nonprime integer greater than 1 with no prime factor less than 10. Then (A) l00 factors are 10,11,12,13, ---. Least prime factor is 11. Possible value of n are 110, 120, 121, 144, Correct number non prime is 121. i.e. 120 < 121 < 130. Answer: (C). 11. A point P inside a rectangle ABCD is joined to the angular points. Then (A) Sum of the areas of two of the triangles so formed is equal to the sum of thé other two. (B) The ‘sum of the areas of the triangles so formed is a whole number whatever may be the dimensions of the rectangle. (C) The sum of the areas of a pair of opposite triangles is greater than half the area of the rectangle. (D) None of these Solution: Consider triangles ; APB and CPD. A B Sum of areas of AAPB and ACPD b we sang} bane = jeut} 1 = hale+y) i D c = xab. 2 196 Priédeme 20d Sberione Consider triangles APD and BPC. Sum of areas of AAPD and ABPC 1 1. = gle) + 5Ha) = zo2+b) = pra = dob, Answer: (A). 12. In the adjoining figure ABCD and BGFE are thombuses. AB = 10cm, GF = 3cm. GE meets / DC at H. ZA=60°. i The perimeter of A ABEHD is (in cm) . e (A) 47 (B) 40 (C) 39 Os Solution: BGFE isa thombus and ZEBG = 60°. BE = GF = 3cm. So EC = BC~BE 10—3=7em. ABCD is a rhombus implies 24 60°. Noting that ABGE is equilateral, ZHEC = &* (vertically opposite angle). -. AHCE is equilateral and we have CH = HE = CE = Tem. The required perimeter AB +BE+EH+HD+DA=10 Answer: (D). 13, If 3041 = 2b-1 (3a — b+ 5c — 9d) is (A) 0 (B)1 (c)2 (D)3 5e+3 = 7d+1 = 15, then the value of Problems and Solutions 199 Solution: 8a+1=15 > 3a=14 2-1=15 > 2%=16,b=8 Se+3=15 > 5c=12 Td+1=15 => 7Td=14,d=2 3a —b+5c—9d = 14-8+12-18=0. Answer: (A) 14. The perimeter of an isosceles right angled triangle is 2012. Its area is (A) 2012(8- v2) (B) (1006)°(3 - v2) (C) (2012)? (D) (1006) Solution: v2 ; Perimeter of isosceles Ye ieee right angled triangle is Given a 2a+V2a = 2012 2012 a= a = 2012/(2+ V2) = mo1a(a = v2) = 1006(2 — V2) a? 1006?(2 — V2)? 1064 — 4/2 +2) Area 5 = gD _ 10067(6— 42) _ 1006? x 2 x (3 ~ 2v2) on 2 oH 2 Answer: (B). 15. Five two digit numbers (none of the digits is zero) add up to 100. If each digit is replaced by its 9 complement, then the sum of these five new numbers is (A) 295 (B) 195 (C) 380 (D) 395 Solution: Let the five numbers be a1b,,a2by,a3b3,ayby_ and asbs. Where a;s are the 10" place and js are units place 200 then we get, Problems and Solutions (10a3+b1)+(10a2+b2)-+(10a3-+b3)+(10a4+b4)+(10a5+b5) = 100 10(a1 + a2 + a3 + a4 + a5) + br + bo + bs + b4 +b5 = 100 (1) Writing the above numbers changing each of digits to its 9 complement we get, [10(9—a1)+(9—by)]-+[10(9—a2)+(9-b2)]+[10(9—as)+(9—bs)] +{10(9 — a4) + (9 — b4)] + [10(9 — a5) + (9 — bs)] W 10[45 — (a; + a2 + a3 + a4 + a5)] +45 — (by + bo + bg + ba + bs) = 450 +45 — [10(a1 + a2 + a3 + a4 + a5) +(bi + bo + bg + bg + b5)] 495 — 100 (using (1)) 395. Answer: (D). Note: Let the numbers be 18,19,20,21,22 whose sum is 100. If each digit is replace by its 9 compliment 18 19 a 20 21 22 100 + + oF + iy 81 80 79 78 77 395 The sum of all 9 compliment numbers is 395. Note: This example does not go with the hypothesis since one of the numbers (20) has a digit zero. Problems and Solutions 201 PART -B 1. The sum of two natural numbers is 484. Their HCF is 11. The number of such possible pairs is —— Solution: Since the HCF of two natural numbers is 11, both the numbers are multiples of 11. Let the numbers be lla and 11b where a,b are co-prime. ©. Mat 11b=484 + 11(a+b)=11x 44 a+b=44 where HCF of a,b is 1. The possible pairs of (a,b) are, (1,43) (3,41) (5,39) (7,37) (9,35) (13,31) (15,29) (17,27) (19,25) and (21,23). .. The number of possible such pairs is 10. Answer: 10. 2. ABCD is a trapezium with AB and CD parallel. If AB = 16cm, BC = 17cm, CD = 8em, DA = 15cm then the area of the trapezium (in cm?) is Solution: D 8 a Draw CE||AD, as shown. We get AECD, 1 ur a parallelogram. :, AE=OD=8cm. BE = AB AB = (16—8)cm BE =8cm Consider ABEC we have BE? + EC? BC? (8? +15) = 17 .. BEC is aright angled A 90° at E. 202 Problems and Solutions > 8 eS .. Area of trapezium = Area of rectangle 1s ss ua AECD+ Area of triangle 4 ¢ - 8 8 BEC = (15 x 8+ 3 x 15 x 8) = 180cm?. 3. The number of multiples of 9 less than 2012 and having sum of the digits as 18 is — Solution: The only 2 digit number with sum of the digits 18 is 99. Let us tabulate 3 digit numbers with sum of the digits 18 as follows: End digit | Numbers How many? 9 189, 279, 369, --», 909 9 198,288,378, 297, 387,477, 396, 486,576, 495,585,675, «++ , 945 594,684,774, «++, 954 693 ,783,873, --- , 963 792 ,882,972 891 ,981 990 fe }releolalalalalo Blreleolalalalalaolo 54 Note that the last 2 digits of these 54 3-digit numbers are different from each other. To get 4-cigit numbers < 2012 with the sum of the digits 18, we can replace the starting digits of each 3-digit number (in the 54 numbers we got) as follows: Problems and Solutions Starting digit Replace by 1 10 2 uu 3 12 4 13 5 4 6 15 7 16 8 7 9 18 203 ‘There is no four digit number from 2000 to 2011 whose sum of the digits is 18. <. The number of multiples of 9, less than 2012 and having sum of the digits as 18 is 1+ 54+ 54 = 109. 4, AB,CD,EF,GH and IJ are five 2-digit numbers such that each letter stands for a different digit. The largest possible sum of these five numbers is Solution: To get the largest sum the tens placed digits must be of higher values ic., A,C,B,G,I should be as large as possible. But they are different. +. The values of A, C, £,G,I should be 9,8,7,6,5 in some order. Here are few possibilities: AB +CD +EF +GH +15 The largest possible sum of these five numbers is 360, a) 93. +84 +71 +60 +52 360 2) 84 +73 +62 +91 - +50 360 (3) 61 +54 +93 +80 +72 360 204 Problems and Solutions 5. Two consecutive natural numbers are respectively divisible by 4 and 7. The sum of their respective quotient is 8. Then the sum between the numbers is — Solution: Let the numbers be n,n +1 2 4 ntl = nee ab. @ and “7 4a => 4a+1=7) 4a—7b= (@) Also a+b=8 => 4a+4b=32 (2) Solving (1) and (2) 11b = 33 b 3 > n 20 n+1 = 21 .. Sum of numbers = 41. 6. p is the difference between a real number and its reciprocal. q is the difference between the square of the same real number and the square of the reciprocal. Then the value of p*+q?+4p” ‘i. Solution: Answer: 2g?. 1 = Aat+ a) -4 1 = et + 3-2) = 297. Problems and Solutions 205 7. A square is inscribed in another square each of whose four vertices lies on each side of the square. The area of the smaller square is 2 times the area of the bigger one. Then the ratio with which each vertex of the smaller square divides the side of the bigger square is — Solution: Let «+ y= a. By data, 2 vty? = Ret vy 49x? + 49y? = 25x? + 25y? + 50ry 2de? + dy? = 50cy <1 > x Zz. 12a? — 25ay+12y? = 0 2 x By -Setae ao 7 y y z _ + VOw— 5G y 2.12 x y _ WAT _ 32 24 2 Justification for +. Since x cannot be equal to y. We can assume x > y. 4 Answer: 3- 8. a,b,c are three positive numbers. The second number is greater than the first by the amount the third number is greater than the second. The product of the two smaller numbers is 85 and that of the two bigger numbers is 115. Then the value of (2012a — 1006c) is —— Solution: b-a=¢c e-b =f b-a -b 2 = ate b= ate a,b,c are in AP. 206 Problems and Solutions ab = 85 and be = 115 b(a+c)=200 + b=10. We have a=8.5, b=10, c= 11.5. 2012a —1006e = 1006(2(8.5) — 11.5) = 1006(17 - 11.5) = 1006 5.5 = 5533. Answer: 5533. a. eu 9. Ihae= 25 and y the value of o(y +2) += + 2 when a = 2012 is —— Solution: «= 74, y= %? I Answer: 2012. 10. PSR is an isosceles triangle in which PS = PR. SP is produced to O such that PO = SP. Then ZSRO is equal to Problems and Solutions 207 Solution: PS=PR=> ZS=ZR. In APSR External 2SPR = 25 In AOPR ZPRO=ZPOR= ZPRO=90-S ZSRO = ZSRP+ZPRO = $+90-$ = 90° ZSRO = 90°. Answer: 90°. sD) (Nr Aliter: With the hypothesis with P as centre draw a circle of radius PS. The circle will pass through O and R and OPS is the diameter. We know that angle in a semicircle is 90°. Hence ZSRO = 90°. 11. a,b,o,d are five integers such that a+b=b+ce=c+d= d+e=2012 and a+b+e+d+e=5024. Then the value of (d-a) = — Solution: Refer Question 4 of Part-A. 12. A class contains three girls and four boys. Every Saturday, five students go on a picnic, a different group is sent each week. During the picnic, each person (boy or girl) is given a Cake by the accompanying teacher. After all possible groups of five have gone once; the total number of cakes received by the girls during the picnic is —— Solution: Assume A, B,C are girls. D,E,F,G are boys. Let us tabulate as follows: 208 Problems and Solutions No. of | Noof | Possibilities | Possibilities CASE | Girls in | Boys in | — of Girls of Boys 5 persons | 5 persons 1 3 2 ‘ABC DE or DF or DG or EF or EG or FG 2 2 3 ‘AB or AC_| DEF or DEG or or BC DFG or EFG 3 1 4 Aor BorC DEFG The number of times A (girl) went to picnic 6 +8 Ho = 15 Case 1 Case Il Case III Each girl would have gone 15 times for the picnic. . To total number of cakes received by girls together =3x15=45. Answer: 45. 13. CAB is an angle whose measure is 70°. ACFG and ABDE are squares drawn outside the angle. The diagonal FA meets BE at H. Then the measure of the angle EAH is — Solution: ZCAB = 70° ZCAF = 45° c FAH is straight line G ZBAH = 180°-ZCAB-CAF 180° — 70° — 45° Ve A B = 65° ZBAE = 90° ZEAH = 90° - BAH = 90°-65°=25° . e 7 Answer: 25°. Problems and Solutions 209 14. The number of prime numbers p for which p+2 and p+5 are also prime numbers is Solution: p+5 is a prime which has to be odd => pcannotbeodd > p=2. But for p=2, p+2=4 is not a prime. .. Number of primes for which p+ 2,p +5 are also primes is zero. 15. The number of integer pairs (m,n) which satisfies m(n?+1) = 48 is — Solution: m n?+1 n? (nis an integer) “1 48 47 not possible 2 24 23 not possible 3 16 15 not possible 4 12 11 not possible 6 8 8 7 not possible 6 5 not possible 12 4 3 not possible 16 3 2 not possible 24 2 1 possible value 48 1 0 __ possible value Integer pairs are (24,1) (48,0). KAPREKAR CONTEST Sub-junion Lovel Final Test-2012 1. Find the number of numbers coprime to and less than 2012. Find their sum. Find also the quotient when this sum is divided by 2012. (Information: 503 is a prime). Solution: 2012 = 4 x 503 All even numbers are non coprime to 2012 besides 503 and 1509. ‘The number of even numbers less than 2010 = 221° = 1005. The odd numbers having a common factor with 2012 are 503 and 1509. «. The number of numbers which are non co-prime to 2012 and less than 2012 is 1005 + 2 = 1007. The number of numbers less than 2012 is 2011. .. The number of numbers co-prime to 2012 = 2011 — 1007 = 1004 Sum of these 1004 co-prime number is sum of all these 1004 odd numbers or sum of all the odd numbers less than 2012 — (503 + 1509) 2011 = (2 x 1006 — 1). That is 2011 is 1006" odd number and the sum to 1006 odd numbers is 1006?. Sum of the co-prime numbers to 2012 is 1006? — 2012 = 1006[1004] = 1005? — 1 = 1010025 -1. The quotient is 502. v . Composite twins are defined below: (a) Odd composite twin: let a and a+2 be two odd composite numbers. If (a—2) and (a+4) are primes then (a,a+2) is called an “odd composite twin”. 210 Probleme and Solutions -_ (b) Even composite twin: Let b and 6+2 two even numbers (b> 2). If (6-1) and (b+3) are primes then (b,b+2) is called an even composite twin. List all composite twins less than or equal to 100. Solution: Odd composite twins upto 100 (1) (4) (25,27) (2) (33,35) (3) (49,51) (4) (55,57) (5) (63,65) (6) (75,77) (7) (85,87). Even composite twins upto 100 (1) (4,6) (2) (8,10) (3) (38,40) (4) (44,46) (5) (68,70) (6) (98,100) (ID) In the case of odd composite twins the four numbers are of the form (a —2,a,a+2,a+4). Their sum is 4a+4 = 4(a+1) and in the case of even composite twins the four numbers are of the form (b— 1), b, (6+2), (+3). Their sum is 4b +4 =4(b+1). Thus the four number in either case is always a multiple of 4. 3. ABCD is arectangle. The sides are extended and the external angles aro bisected and the bisectors are produced in both ways to form a quadrilateral. Prove that the quadrilateral is a square. Solution: 212 Problems and Solutions ZDAX = 90° [ZDAB=90°, DAB +ZDAX = 180°] ZDAP = 52DAX = 45° Similarly ZYDC = 90°, ZYDS =45°ZPDA «. In AAPD, ZPAD = ZADP = 45° ZAPD = 180° — (45° + 45°) = 90° Similarly ZAQB = ZBRC = ZCSD = 90° . PQRS is a quadrilateral with all the angles equal, equal to 90°. In AAPD; ZPAD = ZPDA = 45° A is isosceles and right 2d. » AP=PD= 5% Similarly, BR=CR= +, also AQ = BQ=CS=DS= 4 ZPAD + ZDAB + ZBAQ = 45° +.90° + 45° = 180° P,A,Q is a straight line. ae tL _ (bd: PQ = PA+PQ= 4+ y= (HR) Similarly, QR = RS = SP = (4) : . The quadrilateral PQRS has all its angles and all its sides equal. 2 The area of PQRS = (By = sa) = Baer eth | 4. (a) A single digit natural number is increased by 10. The obtained number is now increased by the same percentage as in the first increase, The result’ is 72. Find the original single digit number. Problems and Solutions 213 (b) After two price reductions by one and the same percent the price of an article is reduced from Rs. 250 to Rs. 160. By how much percent was the price reduced each time. Write detailed steps. Solution: (a) Let the single digit number be x. After adding 10 it becomes (10+ 2). ‘The percentage of increase = 12 x 100 = 2%. Given: The increased number is again increased by 12 %. ive. (@ +10) + HOHO00 | This number is given to be 72. (+10) + (x+10)10 =72 z > (+1) E29) =? (+10)? = 722 => 27-52r+100=0 => (e—2)(z—50)=0 > 2=20r50 The single digit number is 2. (b) Let x % be the price reduction. ‘The first reduction = 250 x 725 = & Rs. The reduction price after the first reduction = (250 — S)Rs. ‘The second reduction = (250 — 5); Rs. ‘The price after the second reduction = 250 — § — (250 - 52) 335 Rs. This is given to be 160 Rs. (250 — 52) — (250 - 5£) 335 = 160 214 Problems and Solutions a ef — fy 10 (500 — 5a)[1 — 3%] = 2 x 160. => (500 — 5z)(100 ~ x) = 200 x 160 => 5x? — 1000x + 18000 = 0 => 2x? —200x + 3600 =0 ie., (@ — 20)(x — 180) =0 iu -» © = 20 or 180. ‘The percentage of reduction is 20 % . If a finite straight line segment is divided into two parts so that the rectangle contained by the whole and first part is equal to the square on the other part, prove that the square described on one of the diagonals of the rectangle contained by the whole and the first part is three times the square on the other part. Solution: (a) Let AB =a be the finite line segment. P divides it. Let AP=2,PB=y. Py 7 o Then c+y=a (1) a Given: ax = y? (2) Let R s PQRS be the rectangle contained by the whole and the first part SQ? =a? +2? To prove that a? + 2? = 3y? Problems and Solutions QQ) > az=y?. Using (1) we get x(a +y) = => wtay=y? Now, a@+a? = 215 (3) (e@+y)? +2? a+ 2Qny+y? +7 2a? + 2ey + y? 2a? + ay) +y? 2y?)+y? [Use (3)] 3y? 6. abcde is a five digit number. Show that abede is divisible by 7 if and only if the number abcd — Solution: Let ¢ = Wyte = ly + 2le = > Tre Ty - (2 x e) is divisible by 7. abede abed —2 xe 1abed — 20e + € abcd 0 +e — 20e ax — 20e. 7. If 23 + by? + 0223 = p®, ax? = by? = cz? and d+hels find /a+vb+ Ve only in terms of p. Solution: Let az? = by? = cz? =k => a=4,b=4,c=4 k 216 Probleme and Solutions But 2x5 + by + c2z3 = pd 2 2 2 = Beth Pred -e(bese3) - lf -a “ Va+vbo+ Ve= ve. 8. Take any natural number. Multiply it with the next two natural numbers. Take another natural number different from the first and do the same as before. Subtract one result from the other to get a positive difference and divide the difference obtained by the positive difference of the original numbers. Add to the quotient the product of the original numbers. Prove that the final result is the product of some number by the number next above it. Solution: Let (x —1) be the natural number. The next two natural numbers are z and +1. Product = (a —1)e(x +1). Let (y—1) be another natural number z # y. The next two natural numbers are y and y+1. Product = (y— 1)y(y+1). Problems and Solutions ene: Difference of the products = 2(z?—-1)-y(y?-1) = a-y'- (2-9) (2 -y)(a? + 2y +4") - (e@-9) = (c-y)(2? +2y+y?-1). Dividing this by the difference of the original numbers («—y) , we get x? +ay+y?—1 Let it be z. z=a+ayty—-1 Add to this the product of the original numbers. We get 2? + cy +y? -—1+(r-1)(y—-1) = a tayty?-ltay—2—-ytl = 420y+y?-2-y = (e+y)?-(e+y)=(et+yet+y-1) = (rt+y-1(e+) = (some number) (the next number above it).

Potrebbero piacerti anche